Behavioral Health - ROSH & SmartyPance EOR Practice Q's

Ace your homework & exams now with Quizwiz!

A 23-year-old woman with no significant prior medical history presents to the clinic reporting a chronic history of sleep issues. She states that in the evenings and during attempted sleep, she often feels a persistent "crawling and tingling" feeling in the legs that is relieved by movement, making falling asleep difficult. Physical exam is benign. Which of the following is the most likely next step in diagnosis? A. Iron studies B. Magnetic resonance imaging of the brain C. Polysomnography D. Urine drug screen

A. Iron studies Restless legs syndrome is a common sleep-related movement disorder characterized by an uncomfortable or unpleasant sensation in the limbs relieved by movement. Symptoms typically occur in the evenings or at night, during periods of inactivity. Most patients with restless legs syndrome demonstrate periodic limb movements during sleep, which may or may not cause arousal from sleep. Restless legs syndrome is common, occurring in up to 15% of the population. There is a strong genetic component. Restless legs syndrome may occur as a primary, idiopathic disorder or secondary to Parkinson disease, pregnancy, iron deficiency anemia, or diabetic or uremic peripheral neuropathy. While the pathophysiology of restless legs syndrome is not completely understood, there is a strong association with reduced iron stores. Additionally, abnormalities in dopaminergic systems, circadian physiology, thalamic functions, and other neurotransmitter irregularities, including glutamate and gamma-aminobutyric acid (GABA), may be seen. The hallmark symptom of restless legs syndrome is an uncomfortable urge to move the legs, and occasionally the arms, to relieve a feeling of deep creeping, crawling, tingling, or electric sensation in the limbs. In addition to movement, patients may also report that stretching, rubbing, or hot or cold water may relieve symptoms. Restless legs syndrome may result in sleep-onset insomnia, nocturnal awakenings, depression, and anxiety. Diagnosis of restless legs syndrome is often made clinically, and the only testing that is always indicated is iron studies due to the strong association with decreased ferritin stores. The International Restless Legs Syndrome Study Group has developed diagnostic criteria for restless legs syndrome that consists of patients having all five of the following: an urge to move the legs that may or may not be accompanied by an uncomfortable sensation in the legs, the urge to move or uncomfortable sensations occur during periods of rest or inactivity, the urge to move or unpleasant sensations are partially or totally relieved by movement as long as the activity continues, the urge to move or unpleasant sensations are worse in the evening or at night than during the day, and symptoms are not solely accounted for by another medical or behavioral condition, such as leg cramps or habitual foot tapping. Polysomnography is not required for the diagnosis of restless leg syndrome, but may be helpful in cases of severe movements causing frequent sleep arousals and in ruling out other disorders, such as obstructive sleep apnea. Treatment for restless legs syndrome consists of oral iron therapy in all patients with iron deficiency or in those with low-normal ferritin levels (ferritin < 75 mcg/L). It is also important that, if possible, patients avoid aggravating drugs, such as caffeine, most antidepressants, antipsychotics, dopamine-blocking antiemetics, and centrally-acting antihistamines, and participate in daily exercise to lessen symptoms. In patients with persistent symptoms despite the aforementioned treatments, a nonergot dopamine agonist (pramipexole, ropinirole, rotigotine) or an alpha-2-delta calcium channel ligand (gabapentin, pregabalin) may be indicated.

A 23-year-old man presents to the clinic with concerns that he is unable to form relationships with others. He notes that he wants to make friends with his co-workers, but he is always scared to attend social gatherings and happy hours after work. When you ask why the patient is hesitant to attend these events, he reports feeling like he may not be liked by others. Which of the following is the most appropriate intervention for this patient? A. Alprazolam as needed B. Cognitive behavior therapy C. Daily escitalopram D. Propranolol as needed

B. Cognitive behavior therapy The patient in the above vignette has avoidant personality disorder. These individuals want to develop a relationship with others but avoid them due to feeling inadequate, inferior, or having fear of rejection or humiliation. These individuals are usually awkward in public situations. According to the fifth edition of the Diagnostic and Statistical Manual of Mental Disorders (DSM-5), an individual must meet four (or more) of the following criteria to be diagnosed with avoidant personality disorder: 1) avoids activities that involve direct person-to-person contact due to fear of being humiliated, rejected, or disapproved of; 2) unwilling to interact with others unless they know for sure they will be liked; 3) shows restraint within intimate relations due to the same fear; 4) is noted to be lost in thought with being rejected during any social interaction; 5) is shy during new social interactions due to being scared of not being liked; 6) views themselves as inadequate or inferior to others; and 7) is usually hesitant to take risks or engage in a new activity due to fear of being humiliated. The best form of management for patients with avoidant personality disorder is psychotherapy, which includes both cognitive behavior therapy and social training. Pharmacological therapy can be used in adjunct to therapy. This therapy can include a beta-blocker as needed for anxiety or a selective serotonin reuptake inhibitor for associated depression. Alprazolam as needed (A) is the correct treatment for acute panic attacks. These individuals do not avoid social situations, rather their panic attacks occur at random and occur abruptly. Daily escitalopram (C) and propranolol as needed (D) are both appropriate treatments only as adjunctive therapy for those with avoidant personality disorder if they have underlying depression or anxiety. Overall, medication therapy is not the first-line treatment for any personality disorder.

A 23-year-old woman presents to the clinic due to anxiety attacks. During these attacks, her heart races, she sweats and feels like she is going to die, and she gets tightness in her chest, nausea, and dizziness. Several visits to the emergency department in the past have determined no physical cause for these symptoms. The attacks started 1 year ago and have occurred only three or four times since then, but to avoid another attack, the patient now stays at home nearly all day, every day and is unable to work. Which of the following is the most effective nonpharmacologic clinical intervention for this patient? A. Acceptance-based exposure therapy B. Cognitive behavioral therapy C. Interpersonal psychotherapy D. Relaxation therapy

B. Cognitive behavioral therapy Acceptance-based exposure therapy (A) is a method of psychotherapy whereby a patient is subjected to triggers of anxiety or panic in an attempt to lessen fear of these situations. However, in this method, breathing techniques, muscle relaxation, and other tools for anxiety control are not introduced first. Interpersonal psychotherapy (C) involves addressing interpersonal conflicts that result in anxiety. Interpersonal psychotherapy is inferior to cognitive behavioral therapy in the treatment of panic disorder. Relaxation therapy (D) is a component of the cognitive behavioral approach, but when it is used alone, it is inferior to cognitive behavioral therapy in the treatment of panic disorder. Panic disorder is characterized by recurrent unexpected panic attacks with at least one of the attacks being followed by at least 1 month of either persistent worry about additional attacks or their consequences or a significant maladaptive change in behavior related to the attacks. A panic attack is a discrete episode of intense fear accompanied by four or more of the following symptoms: palpitations, diaphoresis, shaking, shortness of breath, chest pain, nausea, feelings of choking, dizziness, chills, paresthesias, derealization, fear of losing control, and fear of dying. These symptoms tend to peak within 10 minutes and then gradually subside. Panic disorder is more common in women, has a familial component, and generally begins before 25 years of age. Other psychiatric or physiologic causes of symptoms of panic attack should be ruled out before a diagnosis of panic disorder is made. Treatment for panic disorder can involve nonpharmacologic and pharmacologic therapy. The nonpharmacologic intervention with the most promising clinical data is cognitive behavioral therapy. In cognitive behavioral therapy for panic disorder, the patient is taught about the nature of the disorder and is encouraged to keep a mood diary as well as a record of any panic attacks. Next, the patient is taught breathing techniques and muscle relaxation techniques. Incorrect or maladaptive thoughts are addressed. Finally, patients are exposed repeatedly and systematically in a graduated manner to the situations that trigger panic attacks. Pharmacologic therapy for panic disorder may include selective serotonin reuptake inhibitors, serotonin and norepinephrine reuptake inhibitors, tricyclic antidepressants, or mirtazapine.

A 43-year-old man presents with anhedonia, insomnia, weight loss, intense fatigue, and feelings of worthlessness for the past 3 weeks. In addition to psychotherapy, you decide to prescribe the most suitable atypical antidepressant based on his symptoms. Which of the following side effects for this medication should you educate the patient about? A. Diarrhea B. Drowsiness C. Orthostatic hypotension D. Psychomotor agitation

B. Drowsiness Major depressive disorder is a widely prevalent psychiatric condition that is diagnosed based on criteria established in the fifth edition of the Diagnostic and Statistical Manual of Mental Disorders. Patients must exhibit depressed mood or anhedonia (i.e., lack of interest) with four or more additional symptoms, including insomnia or hypersomnia; unintentional weight loss or gain; psychomotor retardation or agitation; fatigue or low energy; decreased ability to concentrate, think, or make decisions; thoughts of worthlessness or inappropriate guilt; or recurrent thoughts of death or suicidal ideation with or without suicidal intent. Symptoms must occur nearly every day for at least 2 consecutive weeks. The most effective treatment strategy for major depressive disorder involves psychotherapy in conjunction with pharmacotherapy. Second-generation antidepressants used in the treatment of major depressive disorder include selective serotonin reuptake inhibitors (sertraline, citalopram), serotonin and norepinephrine reuptake inhibitors (duloxetine, venlafaxine), atypical antidepressants (mirtazapine, bupropion, agomelatine), and serotonin modulators (nefazodone, trazodone). Tricyclic antidepressants (amitriptyline, nortriptyline) and monoamine oxidase inhibitors (phenelzine, isocarboxazid) are older antidepressants that are more commonly used as second-line medications. Selective serotonin reuptake inhibitors are a common initial treatment but may be limited by side effects, such as sexual dysfunction and weight gain. Atypical antidepressants may be used as an alternative to selective serotonin reuptake inhibitors or serotonin and norepinephrine reuptake inhibitors. These include mirtazapine and bupropion, with agomelatine available outside the United States. Bupropion may cause insomnia or psychomotor agitation, while mirtazapine is most likely to result in drowsiness and weight gain. Another class of antidepressant medication, serotonin modulators, includes nefazodone, trazodone, vilazodone, and vortioxetine. Side effects for nefazodone most commonly include drowsiness and gastrointestinal toxicity, while trazodone may result in orthostatic hypotension, QTc prolongation, gastrointestinal toxicity, and severe drowsiness or somnolence. Diarrhea (A) and other gastrointestinal toxicity symptoms (e.g., nausea, vomiting) are most commonly caused by selective serotonin reuptake inhibitors. Orthostatic hypotension (C) is a known side effect of many tricyclic antidepressant medications (imipramine, amitriptyline) as well as trazodone. Psychomotor agitation (D) and insomnia are not associated with mirtazapine but can be caused by monoamine oxidase inhibitors (isocarboxazid, phenelzine) and certain selective serotonin reuptake inhibitors (sertraline, fluoxetine).

An 18-year-old woman with no prior psychiatric history presents to the emergency department by ambulance after having a witnessed seizure at home. The patient's height is 5 foot 8 inches and weight is 105 pounds (BMI 16 kg/m2). Physical exam reveals lanugo on bilateral upper extremities, brittle nails, and cyanosis of the hands and feet. Laboratory workup reveals hyponatremia with hypokalemic alkalosis. In addition to seizures, which of the following complications is consistent with the suspected diagnosis? A. Hypertension B. Osteoporosis C. Polymenorrhea D. Tachycardia

B. Osteoporosis Anorexia nervosa is an eating disorder characterized by low body weight due to restriction of energy intake relative to needs with a disturbed body image and fear of gaining weight. Patients may restrict intake overall or participate in cycles of binge eating and purging. Medical complications account for approximately half of all deaths in anorexia nervosa, which has one of the highest mortality rates of any mental illness. The general complications of anorexia nervosa are a direct result of weight loss and malnutrition. Starvation induces protein and fat catabolism, which leads to loss of cellular volume and atrophy of the heart, liver, brain, intestines, kidneys, and muscles. Common complications of the cardiovascular system include decreased cardiac mass, reduced cardiac chamber volumes, valvular disorders, bradycardia, hypotension, and QT prolongation. Gynecological complications include infertility and secondary amenorrhea due to anovulation, which often resolve after proper weight gain. Patients with anorexia nervosa can experience profound bone loss due to malnutrition causing increased resorption and decreased formation of bone and are therefore at increased risk of developing osteoporosis. Gastrointestinal complications, including gastroparesis and constipation, are commonly seen. However, patients may also experience diarrhea. Patients may also experience reduced glomerular filtration rate and problems concentrating urine, leading to electrolyte abnormalities, dehydration, and seizures. Neurological disorders, such as Wernicke encephalopathy, Korsakoff syndrome, and brain atrophy, may also be seen and can also contribute to seizures. Cytopenias, including anemia, are often seen. Pulmonary function is often decreased due to wasting of respiratory muscles, resulting in dyspnea and reduced aerobic capacity. Treatment of anorexia nervosa includes refeeding, which may require hospitalization and supervision in severe cases to prevent refeeding syndrome, cognitive behavioral therapy, and treatment of complications as indicated. Psychiatric medications, including selective serotonin reuptake inhibitors (SSRIs) and antipsychotics, play a limited role but may be indicated to treat comorbid illness.

What is the only antidepressant that is not known to aggravate restless legs syndrome?

Bupropion

A 7-year-old boy presents to the clinic with his mother who is concerned because he has been craving ice for the past 2 months. She reports that he consumes about 10 cups of ice per day. Which of the following nutritional deficiencies is the history most concerning for? A. Cyanocobalamin deficiency B. Folic acid deficiency C. Iron deficiency D. Magnesium deficiency

C. Iron deficiency Pica is a psychological disorder defined by an intense craving for nonfood items. The onset of symptoms typically begins during childhood but may also begin during adolescence or adulthood. The diagnosis is based on clinical criteria as defined in the fifth edition of the Diagnostic and Statistical Manual of Mental Disorders. The clinical criteria include repeated eating of nonfood substances that are not nutritional for at least 1 month. Further, the eating behavior must be inappropriate for the patient's expected developmental stage. Pagophagia is a specific type of pica defined as a craving for ice. Pagophagia is specific to iron deficiency, although the mechanism behind this association is unclear. Pagophagia improves rapidly with iron supplementation.

A 45-year-old man with a history of heroin dependence presents to the clinic requesting medical treatment for his addiction. He has tried both cold turkey and 12-step programs without success. He is specifically interested in trying methadone. Which of the following is a potentially life-threatening adverse effect of methadone? A. Atrial fibrillation B. Hypertensive emergency C. QT prolongation D. ST depression

C. QT prolongation Atrial fibrillation (A) is not associated with methadone use. Methadone causes respiratory and central nervous system depression. It does not cause an increase in blood pressure, thus hypertensive emergency (B) is not a side effect that requires monitoring. Other cardiac side effects may occur with the use of methadone. ST elevation, angina pectoris, bradycardia, ventricular tachycardia, myocardial infarction, or cardiac arrest may occur. ST depression (D) is not a finding seen on ECG in patients taking methadone. Opioids in the medical setting are used for pain relief. They also have the potential to cause euphoria. This effect is what leads to opioid misuse and the development of opioid use disorder. Opioid use disorder can occur with both pharmaceutical opioids, such as oxycodone or morphine, or illegal opioids, such as heroin. It is a chronic disorder, typically with multiple episodes of relapse and a high rate of both morbidity and mortality. Opioid use disorder is diagnosed using the fifth edition of the Diagnostic and Statistical Manual of Mental Disorders (DSM-5) criteria, including taking opioids in larger amounts or over a longer period of time than intended; unsuccessful attempts to cut down or quit; cravings; interference with obligations at home, school, or work; use in situations that are physically hazardous; development of tolerance; and the presence of withdrawal symptoms. Medication-assisted treatment with an opioid agonist or antagonist, together with psychosocial interventions is considered first-line treatment for individuals with opioid use disorder. Methadone is a long-acting opioid agonist, which acts on the mu-opioid receptors, resulting in the prevention of withdrawal symptoms, a decrease in cravings, and a reduction in the euphoric effects of other opioids if used while taking methadone. When prescribed for opioid use disorder, methadone is regulated and must be dispensed at a methadone clinic. There are a number of side effects and many drug interactions associated with methadone. Constipation, sweating, drowsiness, and peripheral edema are common side effects, but the cardiac side effects are the most dangerous, and patients on methadone need to be monitored closely to prevent them from occurring. Methadone use has been associated with QT prolongation, which can lead to cardiac dysrhythmias. There are a number of risk factors that increase the likelihood of developing QT prolongation while taking methadone, including age > 65 years, female sex, liver and kidney disease, hemodialysis, anorexia nervosa, and cardiovascular disease. Prior to starting a methadone program, patients should be advised of the risk of dysrhythmia; screened for a history of structural heart disease, dysrhythmia, or syncope; and assessed for other risk factors associated with QT prolongation.

Which childhood condition often precedes antisocial personality disorder?

Conduct disorder

Your patient, who experienced a collision while driving one month ago, is diagnosed with adjustment disorder. Which of the following is most likely reported during history taking? A. Circumscribed, intense periods of fear related to the collision B. Distressing, intrusive thoughts about the collision C. Flashbacks of the collision D. Maladaptive behaviors related to the distress caused by the collision

D. Maladaptive behaviors related to the distress caused by the collision Adjustment disorder is a behavioral disorder characterized by an emotional or behavioral response to an identifiable stressor. The fifth edition of the Diagnostic and Statistical Manual of Mental Disorders also stipulates the behavioral change must occur within 3 months of onset of the stressor and cannot exceed 6 months in duration after the stressor is resolved. Patients should not be diagnosed with adjustment disorder in instances of bereavement. Subtypes of adjustment disorder include with depressed mood, with anxiety, with mixed anxiety and depressed mood, with disturbance of conduct, with mixed disturbance of emotions and conduct, and unspecified. Those diagnosed with an illness, specifically cancer, are more likely to develop adjustment disorder. This diagnosis may be more prevalent in women and is only second to mood disorders in incidence. Patients who engage in self-harm are also commonly diagnosed with adjustment disorder after psychiatric investigation. Risk factors for emotional distress include a history of psychiatric disorders, history of physical or sexual abuse, cognitive impairment, communication barriers, uncontrolled symptoms of cancer, severe comorbid general medical illness, conflicts with caregiver or family, living alone, and financial problems. Somatic symptoms may be the primary concern with multiple, unrelated concerns in different organ systems. Further evaluation reveals underlying mood or conduct disturbances. The essential component in diagnosing adjustment disorder is the presence of a new stressor within 3 months of symptom onset. Adjustment disorder is diagnosed only when the response to the stressor causes marked impairment in the patient's life and does not adhere to societal or cultural norms of response. Maladaptive behaviors related to the distress can include substance use, such as alcohol or illicit drug use, to relieve anxiety or depressive symptoms related to adjustment disorder, and careful evaluation is warranted to rule out substance use disorders. Psychotherapy is the first-line treatment strategy for managing adjustment disorder. Therapy focuses on enabling the patient to remove the stressor from their life, teaching the patient to adapt to stressors that cannot be removed, and altering the response to the stressor with symptom reduction and behavioral modification. Medication therapy may be used as adjunct management for those whose condition does not improve with therapy alone. Antidepressants (e.g., fluoxetine, sertraline, venlafaxine) can be used to combat depression, while benzodiazepines (e.g., lorazepam, diazepam) may alleviate symptoms of anxiety or insomnia.

A 23-year-old woman presents to the clinic reporting sudden-onset sharp chest pain, dyspnea, racing heartbeat, and a choking sensation. The symptoms lasted about 10 minutes. She has a past medical history including major depressive disorder and states she has never experienced these symptoms before. Current medications include bupropion and oral contraceptives. She does not report illicit drug use, alcohol use, or tobacco use. Vital signs include HR 113 bpm, RR 19 breaths per minute, BP 123/91 mm Hg, T 98.8°F, and SpO2 99% on room air. Preliminary ECG readings are shown above, and cardiac enzymes and d-dimer are within normal limits. Which of the following is the most likely diagnosis? A. Factitious disorder B. Illness anxiety disorder C. Myocardial infarction D. Panic attack

D. Panic attack A panic attack is a discrete period of intense fear that is experienced in conjunction with specific cognitive and somatic symptoms. Cognitive symptoms frequently include racing thoughts, preoccupation with health concerns, and catastrophic misinterpretation of somatic symptoms, among others. Patients often report somatic symptoms, such as a choking sensation, racing heartbeat, sweating, nausea, shaking, chest pain, and numbness. Panic attacks are often self-limited and can be categorized as unexpected, situationally bound, or situationally predisposed. Common comorbid anxiety disorders include panic disorder, social phobia, post-traumatic stress disorder, and obsessive-compulsive disorder. Identifying specific triggers for panic attacks can help to uncover these underlying comorbid disorders and further direct specific treatment. Patients who experience recurrent, uncued panic attacks should be treated for panic disorder with primary treatment involving antidepressants, such as fluoxetine, paroxetine, and sertraline. Acute panic attacks can be treated with benzodiazepines (e.g., diazepam, lorazepam), but long-term use of these agents is not recommended due to dependency and side effects (e.g., disinhibition, ataxia, dysarthria, nystagmus). Overdose of benzodiazepines can result in respiratory depression, hypotension, shock syndrome, coma, and death. Factitious disorder (A) is a behavioral disorder in which an individual deceptively presents himself or herself as ill, impaired, or injured even in the absence of an external reward. The patient in the vignette above does not have obvious deceptive behavior and, despite normal preliminary examination and test results, is not overtly falsifying her symptoms. Illness anxiety disorder (B) is characterized by a preoccupation with having or acquiring a serious illness and does not present with discrete episodes of intense fear, such as that demonstrated in the vignette above. Additionally, somatic symptoms are often minimal or even absent in patients with illness anxiety disorder. Myocardial infarction (C) should be thoroughly ruled out in any patient with signs and symptoms suggestive of underlying ischemia. In the vignette above, myocardial infarction is not as likely to be the cause of the patient's symptoms as her preliminary laboratory and ECG results are within normal limits.

A woman presents to the clinic 1 hour late for her scheduled appointment. When questioned as to why she is late, she states she had to walk from home because she is afraid of driving. She states the last time she drove, she had a choking sensation, rapid heart rate, sweating, and nausea. Which of the following is the most likely diagnosis? A. Agoraphobia B. Panic disorder C. Social phobia D. Specific phobia

D. Specific phobia Specific phobia is an anxiety disorder in which patients experience intense fear or anxiety about a particular object or circumscribed situation, other than a social situation. These fears commonly involve animals (e.g., spiders, dogs), the natural environment (e.g., storms), blood-injection-injury (e.g., seeing blood, getting an injection), or situations (e.g., enclosed spaces, heights). Young women with low socioeconomic status are at increased risk for developing specific phobias. Phobias tend to appear in middle childhood or early adolescence and persist throughout the patient's lifetime. Impairment related to the specific phobia can range from mild to severe, with many patients initially presenting for an alternative reason, such as a comorbid mood disorder or result of extreme tactics to avoid the phobic trigger. The fifth edition of the Diagnostic and Statistical Manual of Mental Disorders provides criteria for diagnosing specific phobia. These criteria specify that a patient must exhibit a marked fear or anxiety of a specific object or situation, immediate fear and anxiety when confronted with the phobic object or situation, and an avoidance behavior of the phobic situation or object. If the phobic object or situation cannot be avoided, patients express intense fear or anxiety to endure the phobia. The fear or anxiety must be out of proportion to the actual danger posed by the phobic stimuli, present for 6 months or more, and result in impairment or significant distress. First-line treatment for specific phobia is psychotherapy, which should involve exposure to the feared stimulus. If cognitive behavioral therapy is not accessible to the patient or the phobic stimulus is infrequent or rare, benzodiazepines (e.g., diazepam, lorazepam) can be used to treat an anticipated phobic stimuli encounter. For example, a patient can be instructed to take these agents prior to flying if their phobia involves riding in an airplane. Agoraphobia (A) is the fear of being in a situation where help may be difficult or impossible to obtain if panic-like symptoms or other incapacitating or embarrassing symptoms emerge. Patients with this condition often avoid public spaces and crowds. The patient in the vignette has a fear specific to driving motor vehicles and does not express panic or fear in other public situations. Panic disorder (B) presents with similar symptoms to those being described in the vignette, but the panic attacks associated with panic disorder are unprovoked and have no known trigger. In contrast, panic attacks related to specific phobia can be traced to a single triggering object or situation (e.g., driving). Social phobia (C), also known as social anxiety disorder, is defined by excessive fear of scrutiny, embarrassment, and humiliation in social or performance situations. The patient in the vignette is concerned with driving the motorized vehicle.

A 74-year-old man with a history of a psychiatric condition marked by delusions and hallucinations presents to the psychiatric clinic with an insidious onset of involuntary protruding tongue movements. He reports that he has been taking quetiapine for 20 years. Which of the following is the suspected diagnosis? A. Agranulocytosis B. Akathisia C. Neuroleptic malignant syndrome D. Tardive dyskinesia

D. Tardive dyskinesia Schizophrenia is a severe psychiatric condition marked by recurrent episodes of psychosis. Manifestations include hallucinations, delusions, disorganized thoughts, disorganized behavior, and negative symptoms, which are an absence or reduction in normal processes. They include diminished expression, decreased motivation or interest in typically appearing activities, lack of energy, and social isolation. Schizophrenia is diagnosed according to criteria listed in the fifth edition of the Diagnostic and Statistical Manual of Mental Disorders, which include a symptom duration of at least 6 months and the presence of two of the following findings: delusions, hallucinations, disorganized speech, disorganized behavior, and negative symptoms. Patients with schizophrenia are treated with antipsychotic medications, such as quetiapine. Tardive dyskinesia is a medication-induced hyperkinetic movement disorder that is caused by exposure to dopamine-blocking pharmacologic agents. Oral, facial, and lingual dyskinesias (abnormal movements) are the most common manifestations. These movements may manifest as tongue protrusions, smacking the lips, retracting the corners of the mouth, bulging cheeks, or chewing movements. However, patients may also experience dyskinesias in the extremities or trunk or other symptoms, including dystonias (sustained or repetitive muscle contraction), chorea, or tics. The diagnosis of tardive dyskinesia is made clinically. In most cases, the patient will have oral, facial, or lingual dyskinesias and a history of exposure to an antidopaminergic medication, such as an antipsychotic or metoclopramide. Patients who are taking one of these medications should be closely monitored for tardive dyskinesia. The treatment of tardive dyskinesia varies according to the type of presentation. The causative medication should be discontinued if possible, but in many cases, the antipsychotic medication is necessary to treat a psychiatric condition. Patients with minimal symptoms of tardive dyskinesia do not require treatment. Patients with anxiety due to mild symptoms can be treated with a benzodiazepine, such as clonazepam. Those with localized and severe symptoms can be treated with botulinum toxin injections, and patients with disturbing diffuse symptoms can be treated with vesicular monoamine transporter 2 inhibitors, such as valbenazine or tetrabenazine. Anticholinergic drugs, such as benztropine, may exacerbate dyskinesias but can be helpful in patients who primarily have dystonias. Interestingly, some patients who have severe and refractory tardive dyskinesia and no longer require antipsychotic medications are put back on antipsychotic medications because it can help suppress symptoms of tardive dyskinesia. Agranulocytosis (A) refers to a reduced number of granulocyte white blood cells, which includes neutrophils, eosinophils, and basophils. It is a classic side effect associated with the second-generation antipsychotic clozapine. Akathisia (B) is a common extrapyramidal symptom that may occur as an adverse effect of antipsychotic medications. The typical presentation is a motor restlessness described as a compelling urge to move or an inability to sit still. In mild cases, the restlessness is a subjective sensation that cannot be observed. The treatment options for akathisia include beta-blockers, benztropine, and benzodiazepines. Neuroleptic malignant syndrome (C) is a life-threatening neurologic emergency that is associated with antipsychotic medication use. The common manifestations are mental status change (often an agitated delirium), rigidity, fever, and dysautonomia (e.g., tachycardia, hypertension, and tachypnea). Symptoms often occur during the first 2 weeks of therapy with antipsychotics. Treatment consists of stopping the inciting agent and providing supportive care.

Which of the following is the most commonly abused substance by schizophrenic patients? A. Alcohol B. Cannabis C. Cocaine D. Tobacco

D. Tobacco Substance abuse can present with a variety of manifestations, depending on the substance being abused. The most commonly used substances include caffeine, alcohol, and tobacco or nicotine products. The DSM-V provides the diagnosis substance use disorder to broadly classify those displaying addictive behavior. Those at a higher risk of substance abuse include comorbidities of depressive, anxiety, or bipolar disorders, posttraumatic stress disorder, eating disorder, attention deficit hyperactivity disorder, or schizophrenia. In an epidemiologic study, researchers found 90% of schizophrenic patients use nicotine. Screening for substance use disorder should be performed at regular intervals with further assessment as needed. Treatments for tobacco cessation include medications, such as bupropion or varenicline, or nicotine replacement therapy, either alone or in combination with behavioral psychotherapy.

What are the three clinical findings that comprise the triad associated with Wernicke encephalopathy?

Encephalopathy, oculomotor dysfunction, and gait ataxia

Which antidepressant requires discontinuation for 5 weeks prior to initiating a monoamine oxidase inhibitor?

Fluoxetine

What are the established adverse effects of selective serotonin reuptake inhibitors?

Sexual dysfunction, drowsiness, insomnia, weight gain, headache, and dizziness.

How is factitious disorder distinguished from malingering?

The fabrication of symptoms in malingering is motivated by a clear external reward, such as time off from work or opioid pain medications.

True or false: antidepressants play a role in the treatment of cyclothymic disorder.

True. They may be used judiciously as second- or third-line therapy.

Which teratogenic anticonvulsant has been linked to autism spectrum disorder?

Valproate

A 23-year-old woman gives birth to a child with spina bifida after being on an unknown medication during the first trimester of pregnancy. The medication was prescribed to treat a psychiatric condition marked by hypomanic and major depressive episodes. Which of the following medications is most likely to cause spina bifida in the fetus when used during pregnancy? A. Carbamazepine B. Lamotrigine C. Lithium D. Olanzapine

A. Carbamazepine Lamotrigine (B) is an antiseizure medication sometimes used as a mood stabilizer in the treatment of bipolar disorders. It has no known major teratogenic effects, although there is a possible risk of increases in oral clefts, such as cleft lip and cleft palate. Lithium (C) is classically associated with cardiovascular abnormalities following first-trimester exposure, such as the Ebstein anomaly (defect with the tricuspid valve). Olanzapine (D) is a second-generation antipsychotic medication used to treat a variety of psychiatric conditions. Available data have not demonstrated an increased risk of any major congenital abnormalities in the children of women taking olanzapine during pregnancy. Carbamazepine is more likely than lamotrigine, lithium, and olanzapine to cause neural tube defects, such as spina bifida. Bipolar II disorder is diagnosed in patients with at least one episode of major depression and at least one episode of hypomania. It is distinguished from bipolar I disorder by the absence of manic episodes. Hypomanic episodes present with features that are similar to manic episodes but less severe. Hypomanic episodes are characterized by an abnormal and distinct period of elevated or irritable mood with increased energy and activity. The episode must last at least 4 days, and the symptoms must be present most of the day nearly every day during this period. Patients with an elevated mood require three associated symptoms, while patients with an irritable mood require four associated symptoms. These associated symptoms include inflated self-esteem, decreased need for sleep, increased talkativeness, distractibility, racing thoughts (may manifest as flight of ideas), increased goal-directed activity, and risky behavior. The diagnosis of bipolar II disorder is made according to the criteria in the fifth edition of the Diagnostic and Statistical Manual of Mental Disorders. The symptoms must not be due to the physiologic effects of a substance. In addition, if the symptoms include psychotic features or require hospitalization, the episode is considered a manic episode. The management of bipolar II disorder includes treating hypomanic and depressive mood episodes and establishing maintenance treatment between episodes. The recommended treatment of hypomanic episodes is similar to mild or moderate manic episodes. Olanzapine and risperidone, which are atypical antipsychotics, are the first-line treatments. Second-line options include lithium, other antipsychotics, and antiseizure medications, such as valproate and carbamazepine. When possible, the treatment selection should be guided by the patient's previous response to therapies. Patients with bipolar II disorder who present with a depressive episode and who are not currently being treated with antimanic medications (lithium, valproate, carbamazepine, and lamotrigine) are treated initially with antipsychotic medications, specifically quetiapine or lurasidone. Patients who present with a depressive episode who are treated with antimanic medications should have an antipsychotic medication added to the regimen. The recommended maintenance treatment for patients with bipolar II disorder is the same regimen that led to improvement in the previous mood episode. However, if this regimen is not tolerated due to adverse effects, then lithium, valproate, quetiapine, or lamotrigine is preferred. However, valproate should ideally not be used in women of childbearing age due to its teratogenic effects. Teratogenic effects should be considered in women of childbearing age with bipolar disorder. Valproate is associated with an increased risk of neural tube defects, major malformations, and neurodevelopmental abnormalities. The major malformations can include craniofacial defects (oral clefts), cardiovascular malformations, and limb malformations (club foot and polydactyly). Children exposed to valproate in utero have lower IQ scores than the general population. Carbamazepine is associated with neural tube defects (spina bifida) and similar major malformations to those found with valproate use. However, the risk of neural tube defects is higher with exposure to valproate than carbamazepine.

A 32-year-old man presents to the clinic with recurrent, episodic intense fear with symptoms of choking and dyspnea. He cannot identify a trigger to these episodes but states they resolve spontaneously after 10 to 15 minutes. Which of the following comorbidities is the most likely to be found in this patient? A. Major depressive disorder B. Obsessive-compulsive disorder C. Social anxiety disorder D. Specific phobia

A. Major depressive disorder Obsessive-compulsive disorder (B) presents with obsessions that may be similar to panic attacks but are only associated with the underlying object of obsession and compulsion. Social anxiety disorder (C) manifests with fear of specific social situations, which can trigger a panic attack. Similarly, panic attacks associated with specific phobia (D) are caused by circumscribed phobic objects or situations. These conditions are important to rule out prior to diagnosing panic disorder, in which panic attacks are unexpected and have no known associated trigger. While these conditions may be comorbid, they are less likely to be present than major depressive disorder. Panic disorder is an anxiety disorder characterized by unexpected, sudden onset, overwhelming fear, which is termed panic attacks. These panic attacks are accompanied by somatic and cognitive symptoms, including racing thoughts, a choking sensation, racing heartbeat, sweating, chest pain, and numbness, among others. The median age of onset for panic disorder is 24 years old, with a prevalence twice as high in women versus men. Approximately 37% of patients with panic disorder also have a lifetime prevalence of major depressive disorder. The Diagnostic and Statistical Manual of Mental Disorders, fifth edition diagnostic criteria for panic disorder include recurrent, unexpected panic attacks followed by at least 1 month of anticipatory concern or worry about additional attacks, maladaptive behavioral changes related to the attacks, or both. Other causes of the panic attacks must be ruled out prior to the diagnosis of panic disorder. Common conditions in the differential diagnosis include social anxiety disorder, specific phobia, obsessive-compulsive disorder, and post-traumatic stress disorder. Initial treatment of panic disorder includes either cognitive behavioral therapy or antidepressants. Pharmacotherapy approaches initially involve selective serotonin reuptake inhibitors (e.g., fluoxetine, sertraline) for at least 4 weeks. If the patient is refractory to this treatment strategy, an alternative selective serotonin reuptake inhibitor or a serotonin and norepinephrine reuptake inhibitor should be implemented. Patients who do not respond to pharmacotherapy approaches should be started on psychotherapy, specifically cognitive behavioral therapy. Benzodiazepines (e.g., lorazepam, diazepam) can be used to augment antidepressant treatment in those with a partial response after 10 to 12 weeks of treatment. Treatment-resistant panic disorder in patients without substance or alcohol use disorders can be treated with long-acting benzodiazepines (e.g., clonazepam) either adjunctively or as a monotherapy. Alternatively, gabapentin, pregabalin, or mirtazapine is the preferred agent in patients with concomitant substance or alcohol use disorders Which serotonin and norepinephrine reuptake inhibitor is approved for the treatment of panic disorder? Answer: Venlafaxine.

A 24-year-old woman presents to her therapist to discuss her dietary habits. She reports that she intermittently consumes large amounts of food, such as two large pizzas or two dozen cookies, and subsequently feels guilty. Which of the following additional features is most suggestive of binge eating disorder? A. Abnormally low body weight B. Absence of compensatory behaviors C. Distortion of body weight D. Excessive exercise

B. Absence of compensatory behaviors Abnormally low body weight (A) is incorrect. Patients with binge eating disorder are typically overweight. Distortion of body weight (C) is an important feature of anorexia nervosa. It occurs when individuals have a view of their body image that is significantly different from reality. Excessive exercise (D) is an example of a compensatory behavior. The presence of compensatory behaviors is one of the key factors that distinguishes bulimia nervosa from binge eating disorder. Binge Eating Disorder Eating distinctly more than most people within a certain time period Lack of control, overeating, marked distress Tx: CBT is first line, weight management if also needed, medication options include lisdexamfetamine, topiramate, SSRIs Binge eating disorder is an eating disorder in which individuals have episodes of binge eating without compensatory behaviors. Binge eating is defined as consuming an amount of food in a discrete period of time that is larger than what most people would eat in a similar amount of time under similar circumstances. Binge eating episodes are also characterized by the following features: having a sense of loss of control, eating rapidly, eating until feeling uncomfortably full, eating when not physically hungry, eating alone due to embarrassment regarding the amount of food consumed, and feeling distress about overeating. The episodes must occur at least once per week for at least 3 months. Binge eating disorder is more common in women than men and typically begins during adolescence or young adulthood. Binge eating disorder is diagnosed according to the criteria in the fifth edition of the Diagnostic and Statistical Manual of Mental Disorders. Psychotherapy is the recommended first-line treatment for the disorder, and cognitive behavioral therapy is the recommended psychotherapy. Behavioral weight loss therapy is recommended in patients who either do not improve with psychotherapy or cannot try psychotherapy. The purpose of behavioral weight loss therapy is to improve behaviors by reducing caloric restriction, increasing physical activity, and improving nutrition. Pharmacologic treatment is another second-line treatment modality that can be used as monotherapy or to augment cognitive behavioral therapy. Selective serotonin reuptake inhibitors, such as sertraline, are the preferred pharmacologic class of medications.

A 26-year-old woman presents to the clinic with poor sleep. She states she has nightmares that cause her to scream and sweat several times per night. She also complains of daytime episodes of extreme anxiety, palpitations, sweating, and panic that occur after replaying mental images of a physical assault that happened to her last year. Which of the following is a common predisposing factor in the development of this patient's clinical disorder? A. Being married B. Female sex C. Higher socioeconomic status D. Older age at time of trauma

B. Female sex

A 26-year-old woman presents to the clinic with excessive worrying for the past 7 months. She reports poor sleep, fatigue, difficulty relaxing, and frequent tension-type headaches. You decide to start her on a medication from the same class as paroxetine. Which of the following is an adverse side effect of this class of medications? A. Dependence B. Sexual dysfunction C. Shortening of the QT interval D. Weight loss

B. Sexual dysfunction Generalized anxiety disorder is a psychiatric condition marked by excessive and persistent worry that is difficult to control and results in significant distress and impairment of everyday function. The etiology has both genetic and environmental components. Risk factors for generalized anxiety disorder include female sex, low income, recent adverse life events, chronic illness, parental loss, and a history of psychiatric disorder in parents. The excessive worry spans over many aspects of life. Furthermore, somatic symptoms often accompany the worrying. Patients often report poor sleep, fatigue, difficulty relaxing, headaches, and pain in the neck, shoulders, or back. The diagnosis is made according to the criteria in the fifth edition of the Diagnostic and Statistical Manual of Mental Disorders and is largely based on the classic symptoms. Symptoms must be present on most days for at least 6 months to meet the diagnostic criteria. The treatment options for generalized anxiety disorder include psychotherapy, pharmacotherapy, or both. Serotonergic reuptake inhibitors, which include selective serotonin reuptake inhibitors (sertraline and escitalopram) and serotonin and norepinephrine reuptake inhibitors (duloxetine and venlafaxine), are considered the first-line pharmacologic treatment of generalized anxiety disorder. These medications have similar efficacy, and the selection of a particular agent is based mainly on the slight variances in their side effect profiles. Common side effects of selective serotonin reuptake inhibitors include sexual dysfunction, gastrointestinal side effects (nausea and diarrhea), insomnia, somnolence, and weight gain. Common side effects of serotonin and norepinephrine reuptake inhibitors include nausea, dizziness, insomnia, sedation, and constipation. The clinical response should be evaluated after 4-6 weeks of treatment since these medications take several weeks to reach their full efficacy. Patients with no response should try another agent from either the selective serotonin reuptake inhibitor or serotonin and norepinephrine reuptake inhibitor class. Patients with a partial response can be treated with buspirone or pregabalin, in addition to the serotonergic agent, to augment the partial response. Common side effects of buspirone include insomnia, agitation, and nausea, while side effects of pregabalin can include sedation and dizziness. Patients who experience a good response should continue the serotonergic medication for at least 12 months to prevent relapse or recurrence. Benzodiazepines, such as lorazepam or clonazepam, are sometimes used for acute management of anxiety and worry during the first few weeks after a serotonergic agent is initiated. While benzodiazepines have efficacy in reducing symptoms of generalized anxiety disorder, there is potential for dependence and tolerance to develop. Dependence (A) is incorrect. However, stopping selective serotonin reuptake inhibitors abruptly or rapidly tapering can cause discontinuation or withdrawal symptoms, such as headache, dizziness, fatigue, and nausea. Selective serotonin reuptake inhibitors can prolong, not shorten, the QT interval (C). Weight gain, not weight loss (D), is a common side effect.

A 40-year-old woman presents with an ankle wound that has not healed in months. She has had an extensive workup by multiple physicians and has been treated at wound care centers by specialists on several occasions. She continues to have recurrent infections of the wound, and cultures have always grown atypical pathogens. A review of her medical history shows multiple drug allergies and the involvement of multiple medical professionals for issues that are pervasive and atypical. She has undergone multiple medical procedures over the last few years without any pathology being discovered for her symptoms. Which of the following should be included in the treatment plan as a general strategy in treating this patient? A. Discontinue general medical care B. Emphasize that retaliatory actions can be taken C. Ensure that the patient confesses D. Have one clinician oversee patient management

D. Have one clinician oversee patient management Factitious disorder imposed on self is characterized by an individual creating false medical or psychiatric symptoms and then seeking treatment for those factitious diseases or disorders. Individuals may report factitious symptoms or stimulate symptoms even in the absence of external rewards or gain. Individuals typically present with this disorder in their 20s to 30s. Risk factors for developing this disorder include female sex, single marital status, working in health care, past psychiatric history, and family conflict. The etiology of this disorder is unknown, but it has been associated with multiple factors, including early loss via sickness or death, positive experiences related to being sick, trauma, and attention-seeking behaviors. Patients will present with illness or injuries that commonly fall into the following categories: abdominal pain, joint pain, chest pain, coagulopathy, hypoglycemia, infections, nonhealing wounds, vomiting or diarrhea, weakness, or seizures. Psychiatric factitious symptoms most commonly include bereavement, depression, psychosis, and suicidal ideation. Individuals may tamper with medical instruments or specimens, forge medical records, discontinue medical treatment to worsen an existing condition, or fabricate symptoms. Clinicians should suspect factitious disorder in patients using multiple medical facilities, those with multiple drug allergies, and those who have inconsistencies in the history, examination, and laboratory results. Clinical intervention includes having one clinician overseeing the patient management, consulting psychiatry, and informing all clinicians caring for the patient about the diagnosis. Assessing the risk of suicide and monitoring the patient may be necessary to prevent self-injurious behaviors. Discussing the diagnosis with the patient in a nonjudgmental and nonthreatening way is important in taking steps to treat the patient. Psychotherapy is the standard treatment for factitious disorder.

A 27-year-old woman with a history of sexual assault 4 months ago presents to the clinic with concerns regarding frequent nightmares and anxious mood that began shortly after the assault. She states she is always feeling "on edge" due to frequent flashbacks and has since avoided going anywhere alone, as this triggers her anxiety. Which of the following is the most likely diagnosis? A. Acute stress disorder B. Adjustment disorder C. Panic disorder D. Post-traumatic stress disorder

D. Post-traumatic stress disorder Post-traumatic stress disorder (PTSD) is a disorder that is the result of experiencing or witnessing a traumatic or stressful event, including actual or threatened death, serious injury, or sexual violence. PTSD is more common in women than in men. Symptoms often include intrusive nightmares or flashbacks, avoidance of triggering stimuli, negative alterations in thought process or mood, and increased arousal or hypervigilance. Diagnosis is made using the following criteria: exposure to death, injury, sexual violence, presence of at least one intrusion symptom (distressing dreams, dissociative symptoms), avoidance of stimuli associated with the traumatic event, alterations in arousal and reactivity (exaggerated startle response, hypervigilence). Duration of symptoms must be more than 1 month and cause significant distress and impairment, and must not be better explained by a substance or other medical condition. Treatment includes trauma-focused psychotherapy that can include exposure therapy, cognitive behavioral therapy, or a combination of the two. Eye-movement desensitization and reprocessing may also be helpful. A selective serotonin reuptake inhibitor may be helpful in addition to therapy, or when therapy is not available. Prazosin may be a helpful adjunct medication to treat nightmares and hypervigilance. While benzodiazepines and atypical antipsychotics have been used as adjunct medications previously, they are no longer recommended, as studies have shown that adverse effects outweigh potential benefits. It is important to monitor patients with post-traumatic stress disorder for common comorbid disorders, including major depression and generalized anxiety disorder, and to watch for substance use disorder, as many of these patients self-medicate with substances. Acute stress disorder (A) has similar symptoms of less than 1-month duration. Adjustment disorder (B) is an emotional or behavioral response to a stressor (job loss, divorce, physical illness, etc.) rather than a trauma that causes a response that is disproportionate to the severity of the stressor and usually resolves within 6 months of the stressor. Panic disorder (C) is an anxiety disorder consisting of periods of recurrent and unexpected intense fear and panic that occur without an identifiable trigger and is unrelated to a traumatic event.

A 30-year-old woman with a history of anxiety presents to the clinic with postprandial regurgitation that occurs 5 minutes after most meals. She reports no associated retching or dysphagia. High-resolution postprandial impedance manometry shows reflux extending to the proximal esophagus with an increase in gastric pressure. Which of the following is the most likely diagnosis? A. Achalasia B. Esophageal stricture C. Gastroesophageal reflux disease D. Rumination syndrome

D. Rumination syndrome Rumination syndrome is defined as a functional gastrointestinal disorder in which ingested food regurgitates into the mouth after meals. The suspected pathogenesis involves a pressure gradient created by an increase in intra-abdominal pressure (due to abdominal wall activation) and a decrease in intrathoracic pressure. This pressure gradient causes ingested gastric contents to move backward into the mouth. Risk factors for rumination syndrome include anxiety, depression, and constipation. Rumination syndrome classically presents with regurgitation within 10 minutes of finishing most meals. The regurgitation occurs without any effort or retching, and the material can either be spat out or re-swallowed. The diagnosis of rumination syndrome is made clinically. The pediatric and adult diagnostic criteria vary slightly, but they include repetitive regurgitation soon after ingested meals that is not preceded by retching or explained by another medical condition. High-resolution impedance manometry is a diagnostic test that measures pressure and is often performed in patients with regurgitation to rule out achalasia. The classic postprandial impedance manometry finding in rumination syndrome is reflux extending to the proximal esophagus that is associated with an increase in gastric pressure. An upper esophagogastroduodenoscopy is also frequently performed in patients with regurgitation to rule out structural esophagogastric disorders. The management of rumination syndrome consists of treating underlying psychiatric disorders and educating patients to perform diaphragmatic breathing. Diaphragmatic breathing consists of intentionally engaging the abdominal muscles and the diaphragm to fill the lungs more efficiently. This practice helps reduce postprandial intragastric pressure and increases the pressure at the esophagogastric junction. Baclofen can be used in patients with refractory symptoms. Achalasia (A) is a condition marked by impaired relaxation of the lower esophageal sphincter and decreased peristalsis in the distal esophagus due to progressive degeneration of the ganglion cells in the myenteric plexus. Dysphagia is the classic finding, although regurgitation is possible. Furthermore, the classic manometry finding in achalasia is aperistalsis of the distal esophagus and increased lower esophageal sphincter pressure. Esophageal stricture (B) is defined by an abnormal narrowing of the esophageal lumen. It typically presents with dysphagia and can be identified on an upper gastrointestinal series or esophagogastroduodenoscopy. Gastroesophageal reflux disease (C) occurs due to inappropriate relaxation of the lower esophageal sphincter, which results in gastric contents moving backward into the esophagus. Most patients with gastroesophageal reflux disease have retrosternal burning chest pain. Regurgitation is uncommon and is typically a severe finding that may indicate complications, such as a stricture.

A 20-year-old man presents to the emergency department complaining of being given some "bad stuff" to smoke at a party. The patient exhibits paranoia, avoidance of eye contact, sedation, diaphoresis, vomiting, bradycardia, and hypotension. He complains of extreme muscle pain and states he knows people are thinking evil thoughts about him. A urine toxicology screen is negative. Which of the following substances is most likely causing his symptoms? A. Bath salts B. Cannabis C. Crack cocaine D. Spice

D. Spice Synthetic cannabinoids are produced in laboratories and come in seven different structures, all of which are dissimilar to naturally occurring marijuana. Their street names include K2, spice, K3, crazy clown, krypton, Aztec fire, and happy tiger incense. Synthetic cannabinoids lack cannabidiol, a flavonoid found in marijuana that has anxiolytic and antipsychotic properties. The synthetic cannabinoids have greater affinity for the CB1 receptors in the brain and can be up to 800 times more potent than natural marijuana. Synthetic cannabinoids are also mixed with varying degrees of herbs that may have their own psychogenic properties. Synthetic cannabinoid intoxication may present with a constellation of symptoms, including bradycardia, tachycardia, hypotension, hypertension, hyperthermia, acute kidney injury, rhabdomyolysis, angina, hypokalemia, hyperglycemia, paranoia, sedation, seizures, altered mental status, diaphoresis, anxiety, psychosis, avoidance of eye contact, nausea, and vomiting. Because synthetic cannabinoids are structurally dissimilar to marijuana (tetrahydrocannabinol), they are not detectable on urine toxicology screens and can only be detected by gas or liquid chromatography-mass spectrophotometry. Treatment of acute intoxication with synthetic cannabinoids is symptomatic and includes intravenous benzodiazepines, as no antidote exists. Bath salts (A) are synthetic cathinones that are also not detectable on urine toxicology screens. The synthetic cathinones are stimulants similar to methamphetamine. Many signs and symptoms of synthetic cathinone intoxication are similar to that of the synthetic cannabinoids, although bradycardia and hypotension are not among them. Synthetic cathinones are not smoked, but rather ingested, insufflated, or administered intramuscularly, rectally, or intravenously. Street names of synthetic cathinones include bath salts, blow, screen cleaner, vanilla sky, bubbles, MCAT, and research chemicals. Cannabis (B) is detectable on urine toxicology screening, as is crack cocaine (C). Crack cocaine is a potent stimulant and is unlikely to cause bradycardia or hypotension.

A 14-year-old boy presents to the emergency department with hallucinations after inhaling lighter fluid from a plastic bag. Which of the following is a possible acute toxicity associated with using this type of substance? A. Carbon monoxide poisoning B. Leukoencephalopathy C. Myeloneuropathy D. Ventricular tachydysrhythmias

D. Ventricular tachydysrhythmias Inhalants are volatile substances (gas or chemicals that evaporate at room temperature) that produce vapors that can be inhaled and absorbed through the pulmonary mucosa to produce a transient high. They are typically lipophilic, which leads to their rapid absorption into the brain and a quick high. Children 12-17 years of age and young adults are the most common demographic to use inhalants. Common examples of household products that can be used as inhalants include glues, gasoline, paint thinners, and spray paint. Inhalants can be used in several ways, which include sniffing (inhaled directly from the container), bagging (inhaled from a plastic bag), or huffing (inhaled from a soaked towel or rag). Intoxication with volatile inhalants typically only lasts for a few minutes unless there is continued use. Although most individuals who misuse inhalants do not require medical attention, the health consequences can be life-threatening in some cases. Central nervous system acute manifestations can include euphoria, hallucinations, lethargy, confusion, headache, seizures, and central respiratory depression. The cardiovascular acute manifestations can include ventricular tachydysrhythmias, myocarditis, and sudden cardiac death. Pulmonary manifestations may include pneumonitis and hypoxia. Furthermore, there are several clinical manifestations associated with chronic use. These include leukoencephalopathy, myeloneuropathy, sensorimotor peripheral neuropathy, and hepatotoxicity. The diagnosis of inhalant intoxication or misuse can be difficult to make. Clues that suggest inhalant misuse include chemical odors on the breath, skin, or clothes; empty solvent containers; and the presence of bags, rags, or gauze. The management of acute inhalant intoxication consists of supportive care. Carbon monoxide poisoning (A) can occur acutely after inhalation of methylene chloride, which is metabolized to carbon monoxide. Methylene chloride is an industrial solvent and can be used as a paint thinner. Leukoencephalopathy (B) is a manifestation of chronic inhalant toxicity that occurs after chronic exposure to toluene, which is an inhalant in glues, adhesives, and paint thinners. Leukoencephalopathy is marked by dementia, ataxia, anosmia, and eye movement disorders. Myeloneuropathy (C) is a manifestation of chronic inhalant toxicity that can occur after exposure to nitrous oxide. It is characterized by damage to the tracts of the spinal cord and the peripheral nerves. The manifestations include distal paresthesias, weakness of lower limbs, and an ataxic gait. Which vitamin deficiency causes the neurotoxic effects of nitrous oxide? -Vitamin B12

Which antimanic medication has proven to decrease the risk of suicide in patients with bipolar disorder in clinical studies?

Lithium

Which medication for treating bipolar disorder is associated with an increased risk of Ebstein anomaly following in utero exposure?

Lithium

What medication is contraindicated in treating cocaine-induced hypertension?

Propranolol due to unopposed alpha-constriction.

What rare but potentially fatal syndrome may occur during treatment for anorexia nervosa?

Refeeding syndrome

A 22-year-old woman with a history of congenital prolonged QT interval presents to the clinic with dysphoria for the past 2 months. She reports she has felt fatigued, been sleeping more than usual, and had decreased appetite. She has not been interested in doing activities she previously viewed as hobbies. Which of the following medications to treat her suspected condition would require an electrocardiogram prior to initiation? A. Amitriptyline B. Duloxetine C. Escitalopram D. Venlafaxine

A. Amitriptyline Duloxetine (B) and venlafaxine (D) are serotonin and norepinephrine reuptake inhibitors. They do not cause prolongation of the QT interval. The common side effects of serotonin and norepinephrine reuptake inhibitors include nausea, constipation, dizziness, dry mouth, diaphoresis, and sexual dysfunction. There is also a small increased risk of bleeding and bone resorption, particularly in patients on blood thinners or who have osteoporosis, respectively. Escitalopram (C) is a selective serotonin reuptake inhibitor. Selective serotonin reuptake inhibitors may infrequently prolong the QT interval but do not require an electrocardiogram prior to use. The common adverse effects of selective serotonin reuptake inhibitors are sexual dysfunction, drowsiness, insomnia, weight gain, headache, dizziness, anxiety, and dry mouth. Risk of abnormal heart rhythms, such as heart block and ventricular dysrhythmias, is one of the safety concerns associated with tricyclic antidepressants because these medications can prolong the QT interval on electrocardiogram. Therefore, patients should undergo a cardiac evaluation prior to the prescription of a tricyclic antidepressant. The cardiac evaluation should include a thorough history in all patients and a baseline ECG in patients with concerns identified on history and in all patients who are at least 40 years of age. The history should include screening the patient if they have any cardiac symptoms (such as chest pain, palpitations, shortness of breath, dyspnea on exertion, and syncope), known cardiac disease (e.g., congenital or acquired prolonged QT interval, coronary artery disease), or family history of heart disease. It is important to know which additional medications a patient is taking to see whether they are taking other medications that are known to prolong the QT interval. The patient in the vignette requires an ECG because she has a history of a congenital prolonged QT interval. Tricyclic antidepressants are frequently associated with other side effects, such as anticholinergic effects (blurred vision, constipation, dry mouth, and urinary retention), antihistaminic effects (increased appetite leading to weight gain, sedation, confusion, and delirium), decreased seizure threshold, sexual dysfunction, diaphoresis, and tremor. These adverse effects are caused by the interactions that tricyclic antidepressants have with a variety of neurotransmitter systems, such as muscarinic, histamine, and alpha-adrenergic receptors, in addition to the serotonin and norepinephrine receptors, which give tricyclic antidepressants their efficacy. Ask patients about physical symptoms prior to starting an antidepressant since depression itself can cause physical symptoms. An additional safety concern with tricyclic antidepressants is that they are more dangerous in overdose than other classes of antidepressants due to the prolongation of the QT interval. This is particularly concerning in patients with depression since they are at higher risk for suicide.

A 23-year old woman presents to your office for a new-patient appointment and expresses repeated concern over the appearance of her nose. She mentions she has had two prior nose plastic surgeries by different surgeons but is still unsatisfied. Throughout the visit, she continually uses a pocket mirror to check her nose and also notes she was recently fired from her job due to decreased work productivity. What is the most likely diagnosis? A. Body dysmorphic disorder B. Factitious disorder C. Illness anxiety disorder D. Obsessive-compulsive disorder

A. Body dysmorphic disorder Factitious disorder (B) is a mental disorder characterized by a patient or another person presenting a patient with a falsified medical condition in an attempt to deceive a medical clinician. The falsified condition may involve injuring themselves or causing body changes in an attempt to deceive the clinician to gain attention, sympathy, or reassurance without external incentives. Illness anxiety disorder (C), previously hypochondriasis, is a condition in which the patient is preoccupied with having or acquiring a serious illness but does not have a preoccupation with body appearance. Obsessive-compulsive disorder (D) is a disorder characterized by intrusive and recurrent thoughts (obsessions) occurring for hours each day, as well as repetitive behaviors (compulsions), including hand-washing or counting, that attempt to suppress the obsessive thoughts. The obsessions are not around body appearance, unlike body dysmorphic disorder. Body dysmorphic disorder can be suspected when someone has a preoccupation with at least one defect of a physical attribute that may be nonexistent or slight. This preoccupation leads to repetitive behaviors, including repetitive mirror checking, excessive grooming and skin-picking, and camouflaging, which leads to significant psychological distress. The psychosocial distress causes the patient to possibly avoid work or social situations due to concerns of other people making fun of them due to their perceived body defect. These patients have a history of repeated cosmetic procedures or have seen several medical clinicians that were unsatisfactory to them. In addition, the anxiety or depression is resistant to medication therapy. Patients who are found to meet the criteria of body dysmorphic disorder have varying degrees of insight from good or fair insight, in which the patient is able to recognize the beliefs are not true, to delusional beliefs, in which the patient believes the perceived body defect is true. The face or individual parts of the face, such as the nose, are the most common features for patients to fixate on.

A 24-year-old woman presents to the clinic complaining of palpitations and depressed mood. On physical exam, she is visibly cachectic with a body mass index of 13 kg/m2. A systolic murmur and systolic click are appreciated at the cardiac apex, and her skin is cool and dry with an abundance of fine, dark hair over her entire body. The patient reports eating only one small salad per day because she has an intense fear of becoming fat. The patient is admitted for evaluation and treatment. Which of the following medications should be avoided? A. Bupropion B. Buspirone C. Olanzapine D. Sertraline

A. Bupropion Buspirone (B) is an anxiolytic that may be used to treat patients with anorexia nervosa who also have generalized anxiety disorder. Olanzapine (C) is an antipsychotic medication with a side effect of weight gain that is sometimes used in patients with anorexia nervosa who do not gain sufficient weight while undergoing refeeding. Sertraline (D) is a selective serotonin reuptake inhibitor that may be used to treat generalized anxiety or major depression in patients with anorexia nervosa. This class of medications, however, has not been shown to be effective in the treatment of body dysmorphic disorder or anorexia nervosa itself. Anorexia nervosa is an eating disorder characterized by severe calorie restriction, low body mass index, and obsessive thoughts about food and weight gain. Patients with anorexia nervosa have an intense fear of gaining weight accompanied by a distorted view of their own bodies. Comorbid psychiatric disorders are common in patients with anorexia nervosa and include anxiety, depression, body dysmorphic disorder, obsessive-compulsive disorder, and post-traumatic stress disorder. Female patients are more likely to be affected with anorexia nervosa than male patients, and the average age of onset is 18 years. Medical complications of anorexia nervosa can include cardiac muscle atrophy, mitral valve prolapse, pericardial effusion, bradycardia, osteoporosis, amenorrhea, gastroparesis, constipation, brain atrophy, pancytopenia, hyperpigmentation, xerosis, lanugo, petechiae, and yellowing of the skin. Patients with anorexia nervosa require hospitalization if vital signs are not stable; body mass index is under 14 kg/m2; evidence is found of cardiac, kidney, or hepatic compromise; or there is marked dehydration. Treatment for anorexia nervosa is with careful refeeding and psychotherapy. Pharmacotherapy is aimed at specific medical complications or psychological comorbidities. Patients with anorexia nervosa who suffer concomitant generalized anxiety, obsessive-compulsive disorder, or major depression should not be given the atypical antidepressant bupropion, as an increased incidence of seizure activity is found when bupropion is administered to patients with an eating disorder.

A 32-year-old woman presents to her obstetrician at her 6-week postpartum visit. She reports having a depressed mood for the past 2 weeks with anhedonia and increased guilt. Which of the following is the most widely used screening test for the suspected diagnosis? A. Beck Depression Inventory for Primary Care B. Edinburgh Postnatal Depression Scale C. Patient Health Questionnaire-9 D. World Health Organization Well-Being Index

B. Edinburgh Postnatal Depression Scale Postpartum depression is defined as major depressive disorder occurring in the first 12 months after delivery. Individuals are at increased risk of depression during this time period. The risk factors for postpartum depression include prior depression, prior postpartum depression, stressful life events, and poor social or financial support. The clinical manifestations of postpartum depression are the same as depression outside of the postpartum period: depressed mood, anhedonia (loss of pleasure), increased or decreased appetite, weight gain or weight loss, insomnia or hypersomnia, fatigue, poor concentration, psychomotor agitation, excessive guilt, and suicidal ideation. However, the mood symptoms, which include depressed mood, anhedonia, excessive guilt, and poor concentration, tend to be more specific for postpartum depression than somatic symptoms, such as changes in weight or appetite, because somatic symptoms occur in most postpartum individuals. Postpartum depression is considered severe in patients who have one or more of the following features: at least seven symptoms, suicidal ideation, or psychotic features. The United States Preventive Services Task Force recommends that all postpartum individuals be screened for postpartum depression, and screening should typically occur 4-8 weeks after delivery. The Edinburgh Postnatal Depression Scale is the recommended screening tool. It is a 10-item self-report assessment that can be completed in < 5 minutes and can be repeated later to evaluate for improvement or worsening of symptoms. Individuals who score at least 11 on the Edinburgh Postnatal Depression Scale are considered to be positive for postpartum depression. Individuals with postpartum depression are often treated with psychotherapy, antidepressants, or both. Selective serotonin reuptake inhibitors are often the first-line antidepressant. The Beck Depression Inventory for Primary Care (A) is a highly sensitive and specific test for identifying major depressive episodes in primary care patients. It is not used to screen for postpartum depression. The Patient Health Questionnaire-9 (PHQ-9) (C) is a test used to screen for depression. It has a sensitivity of 88% and a specificity of 85%, which makes it more accurate than most screening tests for depression. It can also be used to monitor a patient's response to treatment. Patients who screen positive on the PHQ-9 should be carefully assessed for depression. However, it cannot be used alone to diagnose depression. The World Health Organization Well-Being Index (D) is a highly sensitive but less specific screening tool for depression. It is not used to screen for postpartum depression.

A 27-year-old man presents to the emergency department with acute agitated delirium. His girlfriend states he began an increased dose of duloxetine earlier today. Vital signs are notable for a heart rate of 135 bpm and blood pressure of 220/115 mm Hg. Physical examination reveals diaphoresis of the skin and mydriasis. Which of the following additional findings supports the suspected diagnosis? A. Bradyreflexia B. Inducible ankle clonus C. Moist mucous membranes D. Recent initiation of atropine

B. Inducible ankle clonus Serotonin syndrome is a life-threatening condition due to increased serotonergic activity in the central nervous system. It typically occurs within 24 hours of initiating or increasing the dose of a serotonergic agent. It most commonly occurs when two or more serotonergic drugs are combined. Selective serotonin reuptake inhibitors (e.g., sertraline and fluoxetine) or serotonin and norepinephrine inhibitors, such as duloxetine, are common pharmacologic causes. Illicit drugs, such as cocaine or amphetamines, are also common causes. The classic triad associated with serotonin syndrome includes mental status changes, neuromuscular excitation, and sympathetic hyperactivity. Patients are often agitated and anxious. Neuromuscular excitation may manifest as clonus (most often inducible ankle clonus), deep tendon hyperreflexia, tremors, akathisia, and hypertonia. Clonus is a classic and sensitive finding for serotonin syndrome. Sympathetic hyperactivity may manifest as tachycardia, hypertension, skin flushing, diaphoresis, dilated pupils, and dry mucous membranes. Hyperthermia is seen in severe cases. Complications may include cardiac dysrhythmias, seizures, rhabdomyolysis, and metabolic acidosis. Serotonin syndrome is diagnosed clinically. The differential diagnosis includes alcohol withdrawal, anticholinergic toxicity, sepsis, thyrotoxicosis, and neuroleptic malignant syndrome. Discontinuing all serotonergic agents and providing supportive care are the main principles of the management of serotonin syndrome. Benzodiazepines are often used to sedate patients. Serotonin antagonists, such as cyproheptadine, are recommended for patients whose vital signs and agitation are not improving with supportive care and benzodiazepines. Patients with severe hyperthermia require sedation, paralysis, and intubation to reduce muscle activity. Antipyretic agents, such as acetaminophen, are not effective in treating patients with hyperthermia due to serotonin syndrome Serotonin syndrome causes hyperreflexia. Neuroleptic malignant syndrome causes bradyreflexia (A). Serotonin syndrome causes dry mucous membranes (C). Atropine (D) is an anticholinergic medication, which could support a diagnosis of anticholinergic toxicity.

A 25-year-old man presents to the clinic with irritability 5 days after discontinuing a psychoactive substance containing delta-9-tetrahydrocannabinol. Which of the following is the most common manifestation of the withdrawal syndrome associated with this substance? A. Disturbed sleep B. Lacrimation C. Piloerection D. Seizures

A. Disturbed sleep Lacrimation (B) and piloerection (C) are clinical manifestations associated with opioid withdrawal. The other manifestations of opioid withdrawal include gastrointestinal distress (abdominal cramps, diarrhea, nausea, and vomiting), flu-like symptoms (rhinorrhea, diaphoresis, and shivering), sympathetic and central nervous system arousal (mydriasis, mild hypertension and tachycardia, anxiety and irritability, insomnia, agitation, restless legs syndrome, general restlessness, and tremor), yawning, sneezing, anorexia, dizziness, myalgias, and leg cramps. Seizures (D) are a severe manifestation of withdrawal from alcohol, benzodiazepines, or barbiturates. Other symptoms can include anxiety, tremors, perceptual disturbances, dysphoria, psychosis, and autonomic instability. Which other medications have efficacy in treating patients with only sleep-related disturbances after cessation of cannabis use? -Zolpidem and nitrazepam

A 62-year-old man is admitted in stable condition following an uncomplicated total hip arthroplasty. His known medical history includes hypertension, peripheral vascular disease, and chronic tobacco use. After 10 hours, he reports nausea and appears anxious. The patient is diaphoretic, and a tremor is noted during intentional movement. Which of the following medications is best to initiate at this time? A. Carbamazepine B. Diazepam C. Naltrexone D. Phenobarbital

B. Diazepam Alcohol withdrawal syndrome is characterized by autonomic hyperactivity, tremors, insomnia, nausea and vomiting, transient hallucinations, psychomotor agitation, anxiety, and tonic-clonic seizures. These symptoms typically appear within 1 to 2 days of alcohol abstinence but may begin as early as 2 to 6 hours after reducing alcohol consumption. Symptom severity can range from mild to a severe and life-threatening presentation known as delirium tremens. Tremulousness is the earliest symptom of alcohol withdrawal, beginning within 6 hours of abstinence. Associated sympathetic hyperactivity symptoms include insomnia, anxiety, gastrointestinal upset, diaphoresis, and palpitations. Withdrawal seizures occur in clusters of two to six episodes within 6 to 48 hours of abstinence and are most likely to occur in those with a history of chronic alcohol use disorder. Hallucinations can present as visual, tactile, or auditory stimuli, develop within 12 hours of abstinence, and typically resolve by 48 hours. Delirium tremens, which is characterized by fluctuating levels of consciousness, cognitive disturbances, profound confusion, and severe autonomic hyperactivity, begins within 48 to 72 hours of alcohol consumption reduction and can last several days to a week. Treatment for patients with signs and symptoms of alcohol withdrawal syndrome is initiated with benzodiazepines. Long-acting agents (e.g., diazepam, chlordiazepoxide) can be given in high doses to treat symptoms with slow clearance of the drug to prevent symptom recurrence. Short-acting benzodiazepines (e.g., lorazepam) can alternatively be used as needed for symptoms. Supportive measures include adequate hydration and replacement of deficient electrolytes. The Clinical Institute Withdrawal Assessment for Alcohol—Revised (CIWA-Ar) is a clinical scale used to guide treatment once the diagnosis of alcohol withdrawal syndrome has been made. This scale requires patients to rank 10 of the most common signs and symptoms of alcohol withdrawal from a baseline score of 0 to the most severe presentation score of 7 in all categories except orientation, which is rated from 0 to 4. Values from each category are then added for a cumulative score with a score < 8 representing mild withdrawal, a score of 9 to 15 indicating moderate withdrawal, and a score > 15 indicating severe withdrawal. If administering short-acting benzodiazepines, this scoring system provides timing recommendations for the next dose of medication with no medication necessary for mild withdrawal, doses every 2 hours for moderate withdrawal, and hourly medication administration for patients with severe withdrawal scores.

What is the optimal serum concentration of lithium?

0.8 to 1.2 mEq/L.

At what age should a child's lack of orientation to name be concerning for autism spectrum disorder?

12 months

What causes alcohol use disorder?

A combination of genetics, environmental factors, and certain personality traits.

What is the hallmark of obsessive-compulsive personality disorder?

A pervasive preoccupation with orderliness.

A 45-year-old woman presents to the clinic for her annual physical exam. You have seen the patient over the years and have met her husband multiple times. During the appointment, she discusses the sexual relationship she is having with Brad Pitt and tells you he recently proposed and they are engaged. On the patient's intake form, she wrote down her relationship status as married. When asked about her husband, she states "he is doing well and has no idea about Brad and me." Her physical exam and laboratory results are unremarkable, and the following week her husband comments he is concerned about his wife's claims to be engaged to Brad Pitt. What is the minimum length of time symptoms have to be present for the suspected diagnosis? A. 1 month B. 1 week C. 2 weeks D. 6 months

A. 1 month Either 1 week (B) or 2 weeks (C) of delusions that resolve may raise suspicion for substance- or drug-related delusions and do not meet the criteria for delusional disorder. Delusions that last more than 6 months (D) meet the criteria for delusional disorder, but delusions have to be present only for a minimum of 1 month to make the diagnosis. In schizophrenia, delusions or hallucinations have to be present for at least 6 months.

A 22-year-old man presents to the emergency department in a catatonic state. He was found at home with a few white tablets in his hand. Vital signs show tachypnea, tachycardia, hyperpyrexia, and hypertension. Physical exam shows vertical nystagmus and hyperreflexia. Laboratory results show elevated creatine kinase. Which of the following substances did this patient most likely ingest? A. 3-Methoxyphencyclidine B. Heroin C. Lysergic acid diethylamide D. Tetrahydrocannabinol

A. 3-Methoxyphencyclidine Phencyclidine is a dissociative anesthetic that has been used as a recreational drug since the 1970s. In recent years, synthetic derivatives of phencyclidine have been produced, such as 3-Methoxyphencyclidine (3-MeO-phencyclidine) and 4-Methoxyphencyclidine (4-MeO-phencyclidine), that offer more potent dissociative effects. Phencyclidine and its derivatives are antagonists of the N-methyl-D-aspartate receptors in the brain and also inhibit the reuptake of serotonin, norepinephrine, and dopamine. Phencyclidine and its derivatives also bind to the sigma receptor complex. Phencyclidine is sold in tablet, powder, crystal, or liquid form and can be smoked, insufflated, ingested, or injected. 3-MeO-phencyclidine is sold in tablet form. Signs and symptoms of intoxication with phencyclidine and its synthetic derivatives can include hallucinations, psychosis, catatonia, coma, agitation, hyperreflexia, hyperpyrexia, tachycardia, elevated blood pressure, disorientation, seizures, and nystagmus (horizontal, vertical, or rotatory). Common laboratory abnormalities associated with phencyclidine intoxication include elevated creatine kinase (indicative of rhabdomyolysis), hypoglycemia, elevated liver transaminases, and hyperuricemia. A urine drug test will detect PCP that has been used within about the last 14 days. Treatment for acute intoxication with phencyclidine and its derivatives is supportive care, with airway protection and benzodiazepines for agitation or seizures. Heroin (B) is an opioid that is either snorted, smoked, or injected. A urine toxicology screen will be positive for opioids in a patient who is acutely intoxicated with heroin. Lysergic acid diethylamide (C) is a hallucinogenic drug that may cause altered mental status but will not usually cause great alterations in vital signs. Lysergic acid diethylamide is not included in a general urine toxicology screen. Tetrahydrocannabinol (D) is the chemical name for marijuana. Marijuana is a cannabinoid that binds to CB1 receptors in the brain, causing euphoria, relaxation, increased hunger, and heightened sensory perception. Lactic acidosis, hyperreflexia, and hyperpyrexia are not common in tetrahydrocannabinol intoxications. Tetrahydrocannabinol will appear on a urine toxicology screen.

A 25-year-old woman presents with 1 week of persistently irritable mood, decreased need for sleep, racing thoughts, distractibility, and psychomotor agitation. Her husband has noticed the change in behavior and became concerned. She does not report any impairment in social or occupational functioning. She does not report any substance use or any pertinent medical history. Which of the following in the patient's history differentiates this as hypomania versus a manic episode? A. Absence of social impairment B. Decreased need for sleep C. Psychomotor agitation D. Racing thoughts

A. Absence of social impairment Decreased need for sleep (B) is associated with both bipolar I and bipolar II disorder. Psychomotor agitation (C) is also associated with both subtypes of bipolar disorder but is less severe in bipolar II disorder. Racing thoughts (D) are present in hypomanic episodes and manic episodes, but in manic episodes, these thoughts become disconnected and lead to ineffective hyperactive behaviors.

An 18-year-old woman presents with concerns of frequent crying, feeling hopeless, and feeling depressed ever since moving into a dormitory to attend college classes 1 month ago. She reports no significant incidents since moving in, but she is finding it difficult to attend social events or focus in class. She reports no alcohol or illicit drug use, and she does not use any medications. Which of the following is the most likely diagnosis? A. Adjustment disorder B. Panic disorder C. Social anxiety disorder D. Somatic symptom disorder

A. Adjustment disorder Panic disorder (B) is a psychiatric disorder characterized by recurrent, unexpected panic attacks followed by a month or more of either persistent worry about additional panic attacks or significant maladaptive behaviors of avoidance. Social anxiety disorder (C) is characterized by anxiety that occurs in social situations only and does not occur during other times or during other events. Somatic symptom disorder (D) is characterized by somatic symptoms that are pervasive and persistent.

A 4-year-old boy presents to the clinic for his annual physical. His vital signs are within normal limits, and the physical exam is normal. He avoids eye contact with the clinician and with his mother, speaks only in one- or two-word sentences, and is preoccupied with a toy car he has brought to the office. His mother states he has never liked to be hugged or touched, and he prefers to play by himself. Which of the following is a risk factor for the most likely diagnosis? A. Advanced parental age B. Infants born after 42 weeks gestation C. Measles, mumps, and rubella vaccination D. Slow rate of head growth during infancy

A. Advanced parental age Autism spectrum disorder refers to a neurodevelopmental disorder characterized by impaired social interaction and repetitive patterns of behavior. Risk factors for the development of autism spectrum disorder include male sex, advanced parental age (maternal or paternal), family members with autism, certain genetic syndromes, and brain abnormalities. Diagnosis of autism spectrum disorder can be made when a patient has deficits in social communication and interaction involving emotional reciprocity, nonverbal communication, and maintaining relationships. The patient must also show restricted or repetitive patterns of behavior in two or more areas, including repetitive movements (such as rocking or flapping), repetitive speech patterns, ritualized behavior, highly fixated interests, and indifference to or hyperresponsiveness to environmental aspects like sound or temperature. Treatment of autism spectrum disorder includes behavioral and educational interventions that aim to improve social function, develop independence, decrease negative behaviors, and improve academic functioning. Certain pharmacologic agents may be prescribed to limit self-injurious behaviors or diminish hyperactivity. These include stimulants, such as methylphenidate, alpha-2 adrenergic agents, such as guanfacine, and antipsychotics, such as risperidone. Patients with autism spectrum disorder may show a range of symptom severity, with some patients able to function independently and others needing lifelong assistance.

A 52-year-old woman with a known diagnosis of chronic glaucoma and schizophrenia presents with a resting tremor, cogwheel rigidity, and bradykinesia. Which of the following therapeutics would be most effective in treating these symptoms? A. Amantadine B. Diphenhydramine C. Lorazepam D. Propranolol

A. Amantadine Diphenhydramine (B) is used to treat acute dystonias and is administered intramuscularly or intravenously. Lorazepam (C) may reduce symptoms associated with akathisia but is associated with the potential for addiction, sedation, and withdrawal seizures. Propranolol (D) is used to treat akathisia. Blood pressure monitoring is required while taking this medication When patients present with extrapyramidal symptoms, the first-line intervention is to reduce the dosing of medication or choose an alternate antipsychotic medication that is less likely to cause these symptoms. If the medication regimen cannot be changed, then treating the extrapyramidal symptoms with an appropriate pharmacotherapeutic is the best next step. The choice of medication should be based on the type of extrapyramidal symptoms present. There are three categories of symptoms: akathisia, dystonia, and parkinsonian syndromes. Akathisia is characterized by a feeling of restlessness that is often accompanied by motor restlessness. Dystonia involves the involuntary contraction of muscles and is often disturbing to patients. Parkisonian syndrome is characterized by cogwheel rigidity, mask-like facies, shuffling gait, resting tremor, and bradykinesia. Amantadine is a nonanticholinergic antiparkinsonian medication that may be used to treat parkinsonian extrapyramidal side effects caused by antipsychotic medications. Anticholinergic medications, such as benztropine, are also used but present an increased risk in patients with glaucoma or cognitive impairment.

A 27-year-old man presents to the emergency department reporting chest pain. On physical exam, the patient is agitated and diaphoretic with dilated pupils. Current vital signs are temperature 101.2°F, BP 160/95 mm Hg, HR 124 beats per minute, RR 24 breaths per minute, and oxygen saturation 94% on room air. Which of the following is the most likely diagnosis? A. Amphetamine intoxication B. Anticholinergic intoxication C. Opioid intoxication D. Phencyclidine intoxication

A. Amphetamine intoxication Amphetamine intoxication is caused by the ingestion of phenethylamine, including traditional amphetamines and newer synthetic compounds. Amphetamines are lipophilic compounds that readily cross the blood-brain barrier and result in the stimulation of alpha- and beta-adrenergic receptors. Phenylethylamines cause the release of dopamine, serotonin, and norepinephrine and also inhibit reuptake. The excess dopamine and serotonin may cause psychotic symptoms. Ring substitutions and other designer modifications have been produced to cause various psychoactive properties. Intoxication with amphetamines or cathinone results in hyper-alertness, hypertension, tachycardia, mydriasis, diaphoresis, hyperthermia, and alterations in mental status, including anxiety, agitation, violent behaviors, and seizures. In high doses, hypervigilance, paranoid ideation, bruxism, tactile hallucinations of insect infestations, persecutory delusions, and aggressive responses may occur. In addition, overdose with amphetamines can produce persistent myoclonus and tremors that may result in rhabdomyolysis. Synthetic cathinones may result in acute kidney injury with acute tubular necrosis and kidney vasospasm resulting in kidney ischemia. Electrolyte abnormalities may also be seen, including hypokalemia, hyponatremia, hypermagnesemia, and elevated anion gap acidosis. Patients who inject amphetamines may have associated cellulitis, abscess formation, and necrotizing fasciitis. While specific testing is helpful to confirm the suspicion of intoxication with a specific substance, amphetamine and synthetic cathinone toxicity is often a clinical diagnosis, as serum drug concentrations are not always available in a timely manner. With an accurate history of amphetamine ingestion and present symptoms consistent with intoxication, a diagnosis can be made. Other recommended general testing includes fingerstick glucose, acetaminophen and salicylate levels, electrocardiogram, pregnancy test in women of childbearing age, serum electrolytes, creatine kinase and urine myoglobin, serum creatinine, serum aminotransferase concentrations, and coagulation studies. Treatment consists of protecting the patients airway and control of agitation and sympathetic excess. Hydration and assessment of nutritional status are also important. Gastrointestinal decontamination may be useful in cases of oral ingestion of large doses of amphetamines but otherwise provides minimal benefit. First-line therapy for agitation is IV benzodiazepines, which also treat the hypertension and seizures often seen in these patients. Benzodiazepines can also lessen the excessive muscle activity that generates heat contributing to hyperthermia. In some cases of hyperthermia, neuromuscular paralysis and evaporative cooling or ice packs may also be necessary. Anticholinergic intoxication (B) presents similarly to amphetamine intoxication with hyperthermia, tachycardia, dilated pupils and flushed skin. However, these patients do not have diaphoresis but rather have dry skin. Opioid intoxication (C) often presents with decreased consciousness, pinpoint pupils, and respiratory depression. Phencyclidine intoxication (D) may cause agitation and psychosis and is not generally accompanied by the adrenergic symptoms seen with amphetamine intoxication, such as hypertension, tachycardia, hyperthermia and diaphoresis.

A 24-year-old woman presents to the emergency department at the urging of her family. Her family reports they are concerned about her weight and self-induced vomiting following meals. The patient describes an intense fear of gaining weight and does not think her current weight is medically problematic. She is 5 feet 7 inches tall and weighs 98 pounds. On exam, you notice thin and soft unpigmented hair on her extremities and yellow discoloration on her palms. Which of the following is the suspected diagnosis? A. Anorexia nervosa B. Binge eating disorder C. Bulimia nervosa D. Restrictive food intake disorder

A. Anorexia nervosa Binge eating disorder (B) is an eating disorder marked by episodes of binge eating. Binge eating is defined as consuming an amount of food in a discrete period of time that is definitely larger than what most people would eat in a similar amount of time under similar circumstances. Anorexia nervosa is distinguished from binge eating disorder by the presence of a low body mass index and behaviors that prevent weight gain, such as self-induced vomiting, prolonged fasting, excessive exercise, use of laxatives, or use of diuretics. Bulimia nervosa (C) is an eating disorder marked by episodes of binge eating with compensatory behaviors, such as self-induced vomiting, prolonged fasting, intense exercise, and misuse of laxatives or diuretics. It can be distinguished from anorexia nervosa by the individual's body mass index. Individuals with bulimia nervosa typically have a normal or high body mass index. Restrictive food intake disorder (D) is similar to anorexia nervosa in that both conditions can lead to low body weight. However, the motivation is different. Restrictive food intake disorder is motivated by a lack of interest in food, aversion to characteristics of food, or concern about choking or vomiting. Individuals with restrictive food intake disorder do not have a distorted body image or an intense fear of gaining weight.

A 19-year-old man presents to the emergency department in police custody with dog bites on his left arm. The police were called because the patient was attempting to stab pets in the neighborhood. The police indicate that the patient has a history of vandalism and setting public buildings on fire. You examine the patient, and he demonstrates no remorse for his actions. Which of the following is the most likely diagnosis? A. Antisocial personality disorder B. Avoidant personality disorder C. Borderline personality disorder D. Narcissistic personality disorder

A. Antisocial personality disorder Avoidant personality disorder (B) is a cluster C personality disorder that is marked by social inhibition, feelings of inadequacy, and hypersensitivity to criticism. These individuals often have few social relationships despite a desire for social interaction. Borderline personality disorder (C) is marked by impulsive behavior, unstable mood and interpersonal relationships, fear of abandonment, and increased risk of suicidal behavior. Narcissistic personality disorder (D) is marked by grandiosity, excessive need for admiration, lack of empathy, and exploitative relationships. Antisocial personality disorder is a cluster B personality disorder along with borderline, histrionic, and narcissistic personality disorders. Cluster B personality disorders tend to be marked by dramatic, emotional, or erratic behavior. Antisocial personality disorder presents with a pervasive pattern of exploiting others, rule-breaking, violating the law, and deceitfulness. Individuals with antisocial personality disorder lack empathy or remorse for their actions. Patients usually develop symptoms initially during childhood, although antisocial personality disorder cannot be officially diagnosed until 18 years of age. However, symptoms often seen during childhood include fighting with peers, conflict with parents, stealing, vandalism, fire setting, cruelty to animals or other children, poor academic performance, and running away from home. Adults diagnosed with antisocial personality disorder frequently achieve poor job performance, pathologically lie, display sexually promiscuous behavior, and have unstable marriages. Antisocial personality disorder is diagnosed according to the criteria in the fifth edition of the Diagnostic and Statistical Manual of Mental Disorders (DSM-5). The treatment for antisocial personality disorder varies some based on the severity of symptoms and the patient's insight and motivation for improvement. Although cognitive behavioral therapy has not demonstrated efficacy in clinical studies, patients who have mild symptoms and a desire to be treated may benefit from therapy. Pharmacologic therapy is generally reserved for treating psychiatric comorbidities and patients with symptoms of severe aggression. Second-generation antipsychotics, such as risperidone and quetiapine, are the preferred agents for patients with severe aggression.

A 36-year-old woman presents to the clinic for an annual physical and reports new-onset intermittent headaches that have been ongoing for the past month. Despite normal laboratory studies, CT, MRI, and EEG, the patient is insistent that part of her brain is rotting and frustrated that no one believes her. She reports no constitutional symptoms, hallucinations, or changes in cognition or memory. Her physical exam results are unremarkable compared to her previous visits. Which of the following medications would be most beneficial for this patient? A. Aripiprazole B. Diazepam C. Duloxetine D. Fluoxetine

A. Aripiprazole Diazepam (B) is a benzodiazepine commonly used in patients with panic attacks or acute anxiety. This medication is not part of the treatment plan for patients with delusional disorder and should be ruled out as a cause of symptoms prior to the diagnosis of this condition. Duloxetine (C), a serotonin and norepinephrine reuptake inhibitor, and fluoxetine (D), a selective serotonin reuptake inhibitor, are antidepressants that are commonly used to treat patients with mood disorders. Additionally, they may be prescribed for patients with illness anxiety disorder or somatic symptom disorder, but they are not appropriate for patients with the fixed delusions of delusional disorder.

A 23-year-old woman with schizophrenia presents to the clinic for follow-up after beginning risperidone 5 months ago. The patient reports decreased symptoms after starting risperidone, but she has not menstruated for 3 months and has noticed an increased frequency of headaches. Which of the following medications is most appropriate for continued treatment? A. Aripiprazole B. Clozapine C. Paliperidone D. Pimozide

A. Aripiprazole Schizophrenia is a psychiatric disorder with associated social and occupational functioning impairments secondary to psychotic symptoms. Positive symptoms of schizophrenia include hallucinations and delusions, while negative symptoms present as flattened affect, alogia, apathy, and anhedonia. Patients with schizophrenia often display disorganized speech, including tangential speech, circumstantial speech, derailment, neologisms, and word salad. According to the fifth edition of the Diagnostic and Statistical Manual of Mental Disorders, patients must present with at least 1 month of delusions, hallucinations, or disorganized speech in addition to either negative symptoms or grossly disorganized or catatonic behavior. Functional capacity must be impaired by these symptoms for a period of at least 6 months (wherein at least 1 month consists of the active-phase symptoms described above). Clinicians should rule out mood disorders with psychotic features. Specifiers can be added to clarify how many episodes the patient has experienced (i.e., first episode or multiple episodes) and whether the condition is in the acute phase or in partial or full remission. First-line treatment for schizophrenia involves antipsychotic medications, with second-generation agents (e.g., aripiprazole, paliperidone, quetiapine, risperidone) generally preferred due to fewer extrapyramidal side effects than first-generation agents (e.g., chlorpromazine, haloperidol, pimozide). The most common extrapyramidal side effects include pseudoparkinsonism, acute dystonic reactions, akathisia, and tardive dyskinesia. Risperidone, a frequently administered second-generation antipsychotic, may also cause hyperprolactinemia leading to hypogonadism (e.g., oligomenorrhea, amenorrhea, galactorrhea, decreased bone mineral density). Patients with these manifestations should be switched to alternative antipsychotic medications with a lower probability of prolactin elevation, such as aripiprazole. Treatment-resistant schizophrenia is often treated with clozapine, but this agent can cause fatal agranulocytosis and should be reserved for those who cannot tolerate other agents or who are refractory to multiple medical approaches to therapy. Clozapine (B) is reserved for use in patients with residual symptoms despite medical therapy and is not appropriate for the patient in the vignette. Paliperidone (C) is a second-generation antipsychotic that has a high potential of causing hyperprolactinemia and thus would not benefit the patient in the vignette, who is experiencing these related side effects. Similarly, pimozide (D), a first-generation antipsychotic, has a higher probability of causing elevated prolactin levels.

A 5-year-old boy presents to the clinic with his father who reports persistent behavioral issues since age 4 that have been recently worsening. The patient's father is concerned about daily temper tantrums and disobedient behavior at home, including arguing with his parents and deliberately annoying his siblings. He also reports teacher concerns about irritable mood and disregard for classroom rules on an almost daily basis. Which of the following interventions is most appropriate for this patient given the most likely diagnosis? A. Assess psychosocial situation and parent training B. Begin cognitive behavioral therapy C. Begin pharmacological treatment with stimulant medication D. Refer for inpatient evaluation and treatment

A. Assess psychosocial situation and parent training While beginning cognitive behavioral therapy (B) is an important aspect of treatment of ODD, evaluating the patient's psychosocial situation and training parents is priority to identify potential causes and triggers of the behavior. Beginning pharmacological treatment with stimulant medication (C) is only appropriate for patients who also meet diagnostic criteria for ADHD. Referring for inpatient evaluation and treatment (D) is not indicated for any patient who is stable and not a threat to themselves or others. Oppositional defiant disorder (ODD) is a conduct disorder in childhood involving angry mood, argumentative or defiant behavior, and vindictiveness. Genetic predisposition is not certain, but the disorder is more common in boys than girls. It is thought that a child's upbringing and social environment play a role. For example, harsh, inconsistent, or neglectful child-rearing practices are common in families of children and adolescents with ODD. Temperamental factors related to problems in emotional regulation, such as high levels of emotional reactivity and poor frustration tolerance, have also been associated with ODD. ODD is among the most common reasons for referral to mental health professionals, affecting about 3% of all children. Over half of those who meet ODD criteria will no longer do so several years later. However, conduct disorder will develop in about one-third of these patients, especially in those with early-onset, severe symptoms, or comorbid attention-deficit/hyperactivity disorder (ADHD). Patients with ODD are typically angry and irritable, often disobeying authority figures, arguing with them, and refusing to comply with rules, often to annoy. For diagnosis, a patient must demonstrate at least four symptoms from the following categories: angry or irritable mood, defiant or argumentative behavior, or vindictiveness lasting at least 6 months. Symptoms must cause functional impairment to the patient, typically interfering with education and social functioning, and other differential diagnoses must be ruled out before a diagnosis of ODD is appropriate (ADHD, substance use, psychotic or mood disorders, ordinary childhood growth and development). Severity depends on the number of settings that symptoms occur in: symptoms of mild cases occur in only one location (i.e., home, school, or with friends), moderate cases occur in two or more locations, and severe cases occur in three or more locations. Treatment consists of parent training combined with outpatient psychological therapy. It is important to evaluate the child's psychosocial environment and also provide proper support to parents, as they may experience adverse mental health effects due to the difficult behavior and social disruption. Parent training is the most important aspect of treatment as it teaches parents to be more positive and less harsh in their discipline style. Collaborative problem-solving interventions facilitate joint problem solving rather than teaching children to comply with parental demands. Some studies have found that stimulant medications used to treat attention-deficit/hyperactivity disorder may be helpful in cases of oppositional defiant disorder with concomitant ADHD. However, studies have not shown that stimulants reduce the symptoms of ODD when ADHD is absent.

During a sleep study, electroencephalography indicates low voltage, sawtooth waves while electromyography indicates atonia. Electro-oculography indicates conjugate, irregular, sharply peaked eye movements. Which of the following is most accurate of this stage of sleep? A. Associated with vivid dreaming B. Comprises the largest percentage of total sleep time C. Frequently referred to as deep sleep D. Time in this stage is increased with benzodiazepine use

A. Associated with vivid dreaming Stage N2 of NREM sleep comprises the largest percentage of total sleep time (B), and time in this stage is increased with benzodiazepine use (D). Stage N3 of NREM sleep is frequently referred to as deep sleep (C).

A 4-year-old boy presents to the clinic with his mother, who has noticed he has more difficulty sitting still than her other children did at a similar age. His preschool teacher says he frequently interrupts other children and struggles to remain seated after being told to do so. Which of the following is the most likely suspected diagnosis? A. Attention-deficit/hyperactivity disorder hyperactive subtype B. Attention-deficit/hyperactivity disorder inattentive subtype C. Conduct disorder D. Oppositional defiant disorder

A. Attention-deficit/hyperactivity disorder hyperactive subtype Behavioral therapy is the first-line treatment for preschool-aged children, while stimulants, such as methylphenidate, are the first-line treatment for school-aged children or adolescents. ADHD inattentive subtype (B) is typically not diagnosed until children are closer to 8 years of age. The symptoms of the patient in the vignette are more consistent with the hyperactive subtype than the inattentive subtype. Conduct disorder (C) is a severe childhood behavioral condition that is marked by destruction of property, aggression toward people or animals, deceitfulness or theft, and serious violation of rules. Oppositional defiant disorder (D) is a maladaptive pattern of defiance or anger seen in children that results in distress for the patient or caregiver. The manifestations of oppositional defiant disorder are less severe than conduct disorder and do not involve the destruction of property, violation of the law, or physical violence against others.

A 30-year-old man presents to the clinic with a chief concern of worsening anxiety. During the appointment, his affect is flat and his speech is disorganized. He states his wife is having an affair with every man in the neighborhood. After the appointment, the patient's wife calls the office to express her concerns and states the patient talks to himself all the time as if he is having a conversation with someone else. Which of the following is the most common type of hallucination of the most likely diagnosis? A. Auditory B. Gustatory C. Somatic D. Visual

A. Auditory Gustatory (B) hallucinations alter a patient's taste and are not commonly present in schizophrenia but can occur. Patients experiencing gustatory hallucinations should raise concern for organic causes, such as infection or inflammation, nerve damage, drugs, vitamin deficiencies, and metabolic or endocrine abnormalities. Somatic (C) hallucinations include feelings of touch or pain in the body or perception of something inside the body. Visual (D) hallucinations are the second most common hallucination in patients with schizophrenia. Some examples are flashing lights, insects, or moving objects.

A 25-year-old woman presents to the clinic to discuss difficulty falling asleep for the past 6 months. She reports that it takes her 3 hours at night to fall asleep despite having ample opportunity to sleep. She is frustrated that the difficulty sleeping has decreased her productivity at work. Which of the following is appropriate sleep hygiene advice to give this patient? A. Avoid napping B. Eat a large meal prior to bed C. Exercise nightly prior to bed D. Put a clock next to your bed

A. Avoid napping Eat a large meal prior to bed (B) is incorrect. It is recommended to avoid large meals before bed but to also not go to sleep hungry. Exercise nightly prior to bed (C) is incorrect. Although frequent exercise improves sleep quality, it is best to not exercise within 2 hours of going to bed. Put a clock next to your bed (D) is incorrect. It is advised to avoid checking the time at night, which includes looking at phones, watches, and clocks in the bedroom. This is because checking the time increases arousal and prolongs wakefulness.

A 3-year old boy presents to the pediatrician with his mother. She reports the preschool he goes to is concerned about his behavior. She notes the patient has difficulty remaining seated during lessons, is always "on the go," blurts out the answers to questions without raising his hand, talks to his classmates during lesson times even after correction, and cannot wait his turn when required. Which of the following is the recommended first-line treatment for this patient's suspected diagnosis? A. Behavioral interventions B. Elimination diet C. Pharmacotherapy D. Psychotherapy

A. Behavioral interventions The diagnosis of attention-deficit/hyperactivity disorder (ADHD) can be determined when a patient < 17 years of age has more than six symptoms of hyperactivity and impulsivity. These symptoms must occur often for at least 6 months and be present in more than one setting. The patient must have had symptoms before 12 years old, and the symptoms should impair function in academic, occupational, or social activities. For preschool-aged children, behavioral therapy is the recommended initial intervention. This can include positive reinforcement and withdrawing privileges due to unwanted behavior. Parent-child behavioral therapy may also be beneficial. Many patients with ADHD do better with a clear daily schedule and consistent small target goals. Pharmacotherapy can be used as an adjunct therapy in preschool-aged children and is the first-line treatment for children ≥ 6 years of age. However, behavioral interventions are also used in school-aged children, with the medication to better reduce symptoms. Elimination diets (B) can be considered on a case-by-case basis, but the influence on the symptoms of ADHD is controversial. It is not the recommended first-line therapy. It can be considered if caregivers are concerned about medication side effects or use and are motivated to adhere to the diet. Pharmacotherapy (C) is the first-line treatment for school-aged children. Psychotherapy (D) is not generally a recommended treatment for patients with ADHD unless there are coexisting conditions, such as anxiety, depression, or social deficits. It has not been shown to be beneficial for the core symptoms of ADHD.

A woman presents to the emergency department via ambulance after her husband witnessed a brief episode of seizure-like activity. The patient has a medical history including insomnia, major depressive disorder, and schizophrenia. Under observation, the patient reports hyperacusis, photosensitivity, anxiety, and dizziness. Vital signs include HR 132 bpm, RR 18 breaths per minute, BP 148/92 mm Hg, T 98.9°F, and SpO2 99% on room air. Which of the following is the most likely diagnosis? A. Benzodiazepine withdrawal B. Cerebrovascular accident C. Neuroleptic malignant syndrome D. Serotonin syndrome

A. Benzodiazepine withdrawal Cerebrovascular accident (B) is less likely in this patient with episodic seizure-like activity and limited atherosclerotic risk factors (e.g., male sex, smoking, diabetes mellitus). Neuroleptic malignant syndrome (C) is clinically identified by fever, rigidity, mental status changes, and autonomic instability and most commonly results from the use of high-potency first-generation antipsychotic agents (e.g., haloperidol, fluphenazine). Serotonin syndrome (D) most commonly occurs with the combined use of serotonergic agents (e.g., citalopram, fluoxetine, tramadol, mirtazapine) and presents with shivering, hyperreflexia, myoclonus, ataxia, and a prodrome involving nausea, vomiting, and diarrhea.

A 19-year-old woman presents to the clinic with her family, who express concern over her recent behavior. She has been skipping her college classes and has not been to work for 2 weeks. When questioned about these behaviors, the patient states she cannot go to school or work because she has to focus on her fashion design career. She expresses profound pleasure in her new work and states she has been working closely with Giorgio Armani. The patient appears irritable and jumps from one unrelated topic to another without pause during the conversation. She has no significant medical history. Which of the following is the most likely diagnosis? A. Bipolar I disorder B. Bipolar II disorder C. Delusional disorder D. Schizophreniform disorder

A. Bipolar I disorder Bipolar II disorder (B) requires at least one episode each of hypomania and major depression. The patient in the vignette is displaying signs and symptoms consistent with a manic episode, making bipolar I disorder the better diagnosis. The patient in the vignette does not meet the criteria for delusional disorder (C), which is defined as the presence of one or more delusions for a month or longer in an individual who does not appear functionally impaired except for the delusions and resultant behavioral modifications. Schizophreniform disorder (D) has psychotic symptoms similar to those found during manic episodes of bipolar disorder, but these symptoms persist for longer durations of time. Additionally, patients with schizophreniform disorder do not have predominant mood symptoms.

A 19-year-old man presents to the office accompanied by his brother for a 7-month history of changes in his behavior. The brother reports the patient's college roommate notified the family that the patient has been acting strangely. The brother reports the patient has an infatuation with a local news reporter, who the patient believed was talking directly to him during her newscast. The patient started writing letters to her and calling the station frequently to talk to his "girlfriend." He has missed several lectures for one of his courses so he would not miss the newscast. At one point, he went to the station on Valentine's Day with flowers and was turned away by security. His grades have dropped. In speaking with the patient, his speech is often off topic, and he exhibits neologisms and poor insight. He has a flat affect. Based on his presentation and suspected diagnosis, you decide to start him on treatment. What is the mechanism of action for the recommended treatment for this patient's condition? A. Blocks dopamine D2 receptors at the postsynaptic membrane B. Blocks voltage-gated sodium channels C. Enhances the inhibitory effect of gamma-aminobutyric acid D. Inhibits acetylcholinesterase to increase the concentration of available acetylcholine at the synaptic cleft

A. Blocks dopamine D2 receptors at the postsynaptic membrane One of the mechanisms of action for the anticonvulsant topiramate is blocking voltage-gated sodium channels (B). Topiramate is not used for the treatment of schizophrenia but may be used off-label for antipsychotic-induced weight gain. The mechanism of action for benzodiazepines is enhancing the inhibitory effect of gamma-aminobutyric acid (C). Benzodiazepines may be used as adjunctive therapy for patients with schizophrenia who have agitation and are refractory to first-line antipsychotic drugs. Benzodiazepines are also useful in treating akathisia, which can develop as a side effect of antipsychotic drug use. Inhibiting acetylcholinesterase to increase the concentration of available acetylcholine at the synaptic cleft (D) is the mechanism of action for cholinesterase inhibitors, such as donepezil, which is used in the treatment of major neurocognitive disorders, such as Alzheimer disease.

A 7-year-old boy presents to the office accompanied by his mother, who reports that he has been struggling in school. She reports that she met with his teacher, who says he has difficulty maintaining attention and makes careless mistakes. His mother reports she has noticed he frequently daydreams and is easily distracted when she talks to him at home. Which of the following should be monitored if this patient is started on the first-line treatment for the suspected diagnosis? A. Blood pressure B. Oxygen saturation C. Peripheral blood counts D. Serum glucose

A. Blood pressure Peripheral blood counts (C) are not impacted by taking stimulant medications. Clozapine is an example of a psychiatric medication that is known to cause agranulocytosis, which is a decrease in neutrophils, basophils, and eosinophils. Oxygen saturation (B) and serum glucose (D) are also not impacted by stimulant medications.

A 20-year-old woman presents to the clinic due to feeling cold and having irregular menses, stating that her last menstrual cycle was over 6 months ago. Her body mass index is 17 kg/m2, and her urine pregnancy test is negative. She states she exercises every day for at least 2 hours because she is overweight and is terrified of gaining weight. Which of the following physical exam findings is most consistent with the diagnosis? A. Bradycardia B. Eroded teeth enamel C. Hypertension D. Oily skin

A. Bradycardia Eroded teeth enamel (B) is more likely to be present in bulimia nervosa than anorexia nervosa. Hypotension, not hypertension (C), would normally be seen in anorexia nervosa. Oily skin (D) is not seen in anorexia nervosa, rather dry skin or xerosis is seen due to dehydration.

A 64-year-old man presents to the clinic with severe depression that is refractory to treatment. Which of the following is considered a contraindication to the most efficacious treatment of severe unipolar depression? A. Brain tumor with elevated intracranial pressure B. Dementia C. Epilepsy D. Pregnancy

A. Brain tumor with elevated intracranial pressure Although there are associated risks, many patients undergoing electroconvulsive therapy have severe depression that has been refractory to other treatments and may be life-threatening. Therefore, there are few absolute contraindications to electroconvulsive therapy. One example of an absolute contraindication is a patient with a brain tumor with associated increased intracranial pressure. This condition is a contraindication because electroconvulsive therapy causes increased cerebral blood flow, which can result in neurologic deterioration. Patients with a brain tumor who do not have increased intracranial pressure can undergo electroconvulsive therapy. Patients with coronary heart disease are at increased risk of ischemic events during electroconvulsive therapy because the seizures increase cardiac workload and oxygen demand. However, most patients with coronary heart disease can tolerate electroconvulsive therapy, and it is appropriate to perform electroconvulsive therapy in patients with cardiac disease if the benefit is considered to outweigh the risks. Additional adverse effects of electroconvulsive therapy include aspiration pneumonia (particularly in patients who do not have an empty stomach), fractures in patients with osteoporosis, dental or tongue injuries, headache, nausea, and anterograde or retrograde amnesia. Dementia (B) is incorrect. Electroconvulsive therapy seems to be just as efficacious in patients with dementia as it is in patients without dementia. However, delirium is a common side effect in patients with dementia undergoing electroconvulsive therapy. Epilepsy (C) is not a contraindication to electroconvulsive therapy. However, anticonvulsants can make it more difficult to induce seizures in these patients. Nonetheless, patients with epilepsy generally continue their anticonvulsants during treatment with electroconvulsive therapy, although reducing the dose or skipping a dose can be considered if it is particularly difficult to induce seizures. Pregnancy (D) is incorrect. Electroconvulsive therapy is generally safe in pregnancy according to the American Psychiatric Association and the American College of Obstetrics and Gynecologists.

A 24-year-old woman presents to the emergency department with auditory hallucinations for the past 2 days. She reports that her husband passed away unexpectedly in a car collision last week. She states that she has not had symptoms like this previously. Her urine drug screen is negative. Which of the following is the most likely diagnosis? A. Brief psychotic disorder B. Schizophrenia C. Schizophreniform disorder D. Schizotypal personality disorder

A. Brief psychotic disorder Schizophrenia (B) requires at least two of the following psychotic symptoms: delusions, hallucinations, disorganized speech, disorganized behavior, and negative symptoms. The symptoms must persist for at least 6 months and not be due to another medical condition or substance use. Schizophreniform disorder (C) has similar diagnostic criteria to schizophrenia except the symptom duration is between 1 and 6 months. Schizotypal personality disorder (D) is marked by eccentric or peculiar behavior. However, patients with schizotypal personality disorder do not have psychosis as seen with brief psychotic disorder, schizophreniform disorder, and schizophrenia. Similar to other personality disorders, the symptoms begin in adolescence or young adulthood and occur in a chronic and long-standing pattern. Patients with brief psychotic disorder are treated with antipsychotic medications, such as risperidone, and often initially require inpatient psychiatric treatment. Patients who present with psychosis without any prior history of psychosis should continue antipsychotic medications for 1-3 months following remission of symptoms. Brief Psychotic Disorder Sudden onset of delusions, hallucinations, or disorganized speech Duration of at least 1 day and < 1 month with full return to premorbid level of functioning Tend to present with overwhelming confusion and no Hx of prodrome Severity often requires supervision to ensure safety or basic needs met Atypical antipsychotics recommended as first-line treatment

A 35-year-old woman presents to the clinic wanting to quit smoking. She has a 10 pack-year history of cigarette use. She desires to become pregnant in the next year, although she is currently using barrier methods of contraception. She takes no current medications, has no chronic diseases or drug allergies, and has a normal physical exam, including vital signs and weight. Which of the following treatment options is the best initial choice for this patient? A. Bupropion B. Clonidine C. Nicotine replacement via e-cigarette D. Nortriptyline

A. Bupropion Clonidine (B) is an antihypertensive medication that crosses the blood-brain barrier and can be used as a second-line agent in smoking cessation. Side effects of clonidine include low blood pressure, somnolence, and rebound hypertension if the medication is abruptly stopped. Nicotine replacement via e-cigarette (C) is a popular choice among patients, as e-cigarettes are often viewed as less harmful than "regular" cigarettes. However, clinicians should not recommend usage of e-cigarettes because of the lack of clinical evidence to suggest that they are an effective and safe nicotine-replacement tool. Nortriptyline (D) is a tricyclic antidepressant that can be used as a second-line agent in patients who do not achieve smoking cessation with first-line therapy. Nortriptyline causes somnolence and dry mouth.

An 18-year-old woman with a history of several episodes of major depression presents to the psychiatric clinic accompanied by her parents, who report she has only slept about 10 hours during the past week. Her parents are also concerned about her irresponsible spending spree the past 2 days and uncharacteristic promiscuous behavior on online dating sites. The patient is of Asian descent and reports skipping school to meet up with men she met online. During your assessment, the patient quickly jumps from one topic to another and has rapid speech. Her urine drug screen and urine pregnancy test are negative. Which of the following medications sometimes used to treat the suspected condition requires screening for the HLA-B*1502 allele prior to beginning treatment? A. Carbamazepine B. Lithium C. Olanzapine D. Valproate

A. Carbamazepine Lithium (B) is a mood-stabilizing medication often used in the treatment of bipolar I disorder. Studies have shown that lithium reduces the risk of suicide in patients with bipolar I disorder. Known adverse effects include nausea, tremor, polyuria, thirst, loose stools, weight gain, cognitive impairment, and impairment of kidney function. Contraindications include cardiovascular disease, kidney disease, thyroid disease, hyponatremia, and dehydration. Olanzapine (C) is an atypical antipsychotic that is frequently used to treat bipolar I disorder. Its adverse effects include weight gain, hyperglycemia, hyperlipidemia, extrapyramidal side effects, sedation, anticholinergic effects (dry mouth, constipation, and urinary retention), orthostatic hypotension, and QTc prolongation. Valproate (D) is an antiepileptic medication that is often used as a mood stabilizer in the treatment of bipolar I disorder. Valproate is often used in combination with an antipsychotic at the initial treatment of severe mania. Furthermore, valproate may be used as monotherapy to treat mild or moderate mania or hypomania. The known adverse effects include weight gain, nausea and vomiting, hair loss, and easy bruising. Rare, but more serious, adverse effects include hepatic failure, thrombocytopenia, and pancreatitis. Liver transaminases and platelets should thus be monitored every 6-12 months in patients who are on valproate.

A 5-year-old boy presents to the clinic for evaluation at the request of his teacher. He has been unable to sit still in class and often talks out of turn, walks around the room, and does not pay attention. His parents state he is very active at home, rarely sitting through a whole television program, and has trouble obeying three-step commands. Which of the following evaluation tools is considered to be a broadband assessment of this child's behavior? A. Child behavior checklist/teacher report form B. Conners third edition: short version C. Disruptive behavior rating scale D. Vanderbilt assessment scales

A. Child behavior checklist/teacher report form The Conners third edition: short version (B) is a narrowband assessment tool for attention-deficit/hyperactivity disorder that measures inattention, hyperactivity and impulsivity, difficulty in school, aggression, executive function, and social interactions. The disruptive behavior rating scale (C) is a narrowband assessment tool for attention-deficit/hyperactivity disorder, oppositional-defiant disorder, and conduct disorder that is completed by parents only. The Vanderbilt assessment scale (D) is a narrowband tool to assess attention-deficit/hyperactivity disorder that can also screen for anxiety, depression, oppositional-defiant disorder, and conduct disorder. Cardiac health must be assessed before medications for attention-deficit/hyperactivity disorder are chosen. Several evaluation tools exist to aid in the diagnosis of attention-deficit/hyperactivity disorder. These evaluation tools should be used on patients over 4 years old to determine the presence and severity of symptoms. The tools include both behavior ratings and educational evaluations. Some of these evaluation tools are considered broadband assessments because they measure behavioral symptoms that are not only specific to attention-deficit/hyperactivity disorder but to other behavioral and psychological disorders as well. The child behavior checklist/teacher report form is an example of a broadband assessment, as the tool assesses attention problems as well as anxiety, depression, aggression, withdrawal, and somatic complaints. Assessment tools that are more specific and sensitive for attention-deficit/hyperactivity disorder include narrowband assessments, such as Conners third edition: short version tool and the childhood attention problems scale. Treatment for attention-deficit/hyperactivity disorder involves behavioral therapy and medications, such as stimulants or alpha-2 adrenergic agonists. Pharmacologic treatment for attention-deficit/hyperactivity disorder should begin after the preschool years and may need to be life-long, although symptoms sometimes improve with age.

Which of the following antipsychotic medications is associated with significant agranulocytosis but not with extrapyramidal side effects? A. Clozapine (Clozaril) B. Haloperidol (Haldol) C. Chlorpromazine (Thorazine) D. Fluphenazine (Prolixin)

A. Clozapine (Clozaril) Clozapine is referred as an "atypical" antipsychotic agent. It can cause agranulocytosis and should be monitored with weekly CBC. This is not a first line treatment drug.

A 22-year-old man presents to the emergency department due to headache, myalgia, and chest pain. He is agitated, appears angry, and is difficult to restrain. Vital signs indicate blood pressure 180/100 mm Hg, pulse 130 beats per minute, and temperature 100.1°F. Physical exam reveals mydriasis, diaphoresis, and diffuse muscular tenderness to palpation. Which of the following is most likely found on this patient's urine toxicology screening? A. Cocaine B. Lysergic acid diethylamide C. Marijuana D. Opiates

A. Cocaine Cocaine is a stimulant that blocks the reuptake of biogenic amines, slows conduction through sodium channels, and stimulates excitatory amino acids. Cocaine is a potent vasoconstrictor. Signs and symptoms of acute cocaine intoxication include hypertension, tachycardia, agitation, headache, hyperthermia, rhabdomyolysis, intracerebral hemorrhage, gastric ulcers, splenic or renal infarct, mydriasis, and diaphoresis. Cardiac ischemia and acute left ventricular dysfunction may also occur. Hyperthermia is a sign of significant intoxication and increases the odds of mortality. Treatment of acute cocaine intoxication begins with stabilization of the airway and circulation. Lowering blood pressure can be accomplished with nitroprusside, nitroglycerin, or phentolamine. Beta-blocking agents should be avoided in acute cocaine intoxication to prevent unmitigated alpha-adrenergic stimulation. Diazepam is given in acute cocaine intoxication for treatment of psychomotor agitation. Immersion in ice may be necessary for severe hyperthermia Lysergic acid diethylamide (B) is a hallucinogen that can cause agitation, dysphoria, and panic during intoxication. Vital signs are not often greatly altered in lysergic acid diethylamide intoxication, although hyperthermia may occur in severe intoxication with psychomotor agitation. Marijuana (C) is a cannabinoid receptor agonist that can cause hypertension and tachycardia during acute intoxication. However, marijuana rarely causes agitation or anger, and mydriasis and diaphoresis would not be present. Dry mouth and conjunctival injection may be noted. Opiates (D) act upon mu, kappa, and delta receptors to reduce pain and anxiety. Opiate intoxication leads to decreased mental status, decreased respiratory rate, and pupillary constriction.

A patient under your care with a history of symptoms, including hypervigilance, insomnia, nightmares, flashbacks, and an exaggerated startle response, after being involved in a car collision 3 months ago presents requesting help managing their symptoms. Which of the following is the best initial treatment approach? A. Cognitive behavioral therapy B. Cognitive behavioral therapy with fluoxetine C. Dialectical behavior therapy D. Fluoxetine

A. Cognitive behavioral therapy Cognitive behavioral therapy with fluoxetine (B) or lone treatment with fluoxetine (D) are approaches used to treat patients who are refractory to psychotherapy alone or who express a desire for pharmacotherapeutic treatment. While these approaches can be efficacious, psychotherapy alone is the recommended first-line treatment. Dialectical behavior therapy (C) is a specific type of psychotherapy most commonly used to treat patients with borderline personality disorder and can be effective in treating patients with post-traumatic stress disorder who are comorbid with this condition.

A 12-year-old patient presents to the clinic for help with aggression toward parents and family pets, frequent violations of the law that include arson and theft, and bullying others at school. Which of the following is also most likely to be true of this patient? A. Comorbid attention-deficit/hyperactivity disorder B. Comorbid obsessive-compulsive disorder C. Female sex D. Grandiosity

A. Comorbid attention-deficit/hyperactivity disorder Conduct disorder is a behavioral disorder characterized by aggression toward people or animals, destruction of property, deceitfulness, theft, and serious violation of societal norms. Diagnosis requires a persistent and repetitive pattern of such behavior over a 12-month period. Risk factors for the development of conduct disorder include poverty, turbulent home life, child abuse, male sex, parental substance use, parental criminal behavior, changes in guardian figures during childhood, peers who participate in criminal activities, harsh parental discipline, and maternal smoking during pregnancy. Conduct disorder is often comorbid with oppositional defiant disorder, attention-deficit/hyperactivity disorder, and substance use disorders. Treatment of conduct disorder begins with treatment of any comorbid disorders, specifically attention-deficit/hyperactivity disorder. Psychosocial therapy is the most effective treatment modality for conduct disorder, including counseling for both the parents and the child. School-based interventional programs with a home component are also viable therapy options. Comorbid obsessive-compulsive disorder (B) is not associated with conduct disorder, although oppositional defiant disorder is. Female sex (C) is protective, not a risk factor. Grandiosity (D) is defined as an unrealistic sense of superiority over others and is not characteristic of conduct disorder. Pediatric patients with bipolar disorder may erroneously receive a diagnosis of conduct disorder during periods of mania, at which time they may indulge in risk-taking behaviors. However, patients with bipolar disorder often manifest grandiosity as well. Other characteristics of bipolar disorder that are not often present in patients with conduct disorder and can help differentiate the two include euphoria, decreased need for sleep, hallucinations, and delusions.

A 6-year-old girl presents to the primary care clinic with her mother for evaluation of generalized abdominal pain for the past 7 months. The patient has been evaluated several times by both her pediatrician and a gastrointestinal specialist for these symptoms and all have been within normal limits. Physical exam reveals a soft, nontender abdomen with normal bowel sounds throughout. You suspect the mother is being deceptive about the patient's symptoms. What is the best next step in the treatment of this patient? A. Consult medical child abuse specialist B. Give reassurance and limit medical intervention C. Order an abdominal ultrasound D. Prescribe acetaminophen

A. Consult medical child abuse specialist Giving reassurance and limiting medical intervention (B), is important but in moderate cases, such as in the scenario, it is important to involve a medical child abuse specialist. As unnecessary testing should be avoided, ordering an abdominal ultrasound (C), and prescribing acetaminophen (D), would be incorrect. Munchausen Syndrome by Proxy (Medical Child Abuse) Parent is causing or reporting the presenting nonorganic symptoms Morbidity and mortality can be high Symptoms disappear when the child is not with the caregiver creating the symptoms

A 24-year-old woman presents to the emergency department with right upper extremity weakness for the past 2 weeks. Physical examination reveals 2/5 strength in the right upper extremity and 5/5 strength in all other extremities. You review the patient's medical records and see that she has been to the emergency department four times in the past with similar symptoms, and diagnostic tests, including neuroimaging, have always been unremarkable. She has also been seen in a neurology outpatient clinic and had a normal nerve conduction study and a normal electromyography. You suspect a psychiatric condition marked by unintentional production of symptoms without clear secondary gain. Which of the following is the suspected diagnosis? A. Conversion disorder B. Factitious disorder C. Illness anxiety disorder D. Malingering

A. Conversion disorder Factitious disorder (B) is a condition marked by intentionally producing symptoms with the goal of playing the role of the patient. There is no clear external reward, which distinguishes factitious disorder from malingering. Factitious disorder can be distinguished from conversion disorder in a couple of ways: the symptoms are intentionally produced and the symptoms are not always neurologic. Illness anxiety disorder (C) is characterized by a preoccupation with having or developing a serious medical illness despite appropriate medical evaluation and reassurance. Somatic symptoms are either not present or minimally present. Malingering (D) is a condition in which a patient intentionally produces symptoms with the goal of a clear external reward, such as opioid pain medications, time off of work, or lawsuit settlements

A 24-year-old woman with a past medical history of untreated hepatitis C virus presents to the emergency department with suicidal ideation. She reports that, for the past 7 days, she has been on a spending spree and bought three cars, which she cannot afford. She also notes during this time she has felt well-rested with minimal sleep. During your assessment, she is easily distracted and talks rapidly. She has a negative urine drug screen. Which of the following is a concerning possible adverse effect of the recommended mood-stabilizing medication for this patient? A. Decline in kidney function B. Hepatotoxicity C. Metabolic syndrome D. Toxic epidermal necrolysis

A. Decline in kidney function Hepatotoxicity (B) is not associated with lithium use, but it is an uncommon yet possibly severe side effect of valproate. Metabolic syndrome (C), which includes increased blood pressure, serum glucose, serum cholesterol, and waist circumference, is associated with the use of antipsychotic medications. The first-line treatment for severe manic episodes is either lithium or valproate and an antipsychotic medication. Toxic epidermal necrolysis (D) is a severe and life-threatening rash that classically involves the mucosal surfaces. It is similar to Stevens-Johnson syndrome except it involves more than 30% of body surface area. Lamotrigine is the mood-stabilizing medication classically associated with toxic epidermal necrolysis.

A 6-year-old boy presents to the clinic for assessment after his teacher noticed that he has difficulty remaining seated during class, difficulty playing quietly, and frequently interrupts others. His mother has noticed a similar pattern at home. You discuss the suspected diagnosis with his mother and decide to start him on the first-line pharmacologic treatment. Which of the following is a common adverse effect of this medication? A. Decreased appetite B. Hypersomnia C. Increased suicidal ideation D. Priapism

A. Decreased appetite Hypersomnia (B) is incorrect because insomnia is the most common sleep-related side effect of stimulants. Stimulant medications do not cause hypersomnia. Increased suicidal ideation (C) is a boxed warning associated with atomoxetine, which is a nonstimulant medication used to treat attention-deficit/hyperactivity disorder. Stimulants are the first-line pharmacologic treatment for attention-deficit/hyperactivity disorder. Priapism (D) is a rare complication associated with stimulant medications.

A 27-year-old man is brought to the psychiatric emergency department by police. The police were called because he was screaming and punching the air on the side of a street. The man reports he was defending his castle from aliens. During your assessment, he has blunted facial expressions and answers your questions in a roundabout way. You review his electronic medical records and see he has been brought in six times over the past 7 months with similar presentations. He has not been able to keep a job during this time, and his urine drug screens have been negative each time. Which of the following is considered a negative symptom of the suspected diagnosis? A. Decreased goal-directed behavior B. Nonbizarre delusions C. Tangentiality D. Visual hallucinations

A. Decreased goal-directed behavior Nonbizarre delusions (B) are delusions that could theoretically be true. Examples of nonbizarre delusions include the belief that the IRS is after you for not paying taxes or that your spouse is cheating on you with your neighbor. Bizarre delusions do not have the possibility of being true, such as the belief that one's organs have been replaced with someone else's without any scars as evidence. Delusions are a positive symptom, and bizarre delusions are a particularly classic finding. Tangentiality (C) is a type of disorganized speech that shows an internal disorganized thought pattern. It occurs when a patient is asked a question and their answer gets further and further off topic. Disorganized speech is a positive symptom. Visual hallucinations (D) occur when an individual sees visual stimuli in the absence of an external source. They are typically unformed, such as flashing lights, but may be formed, such as another person's face.

An 18-month-old girl presents to the clinic with parental concerns for odd motor behaviors. They have noticed the patient flapping her hands frequently. They report she has frequent tantrums as well. Which of the following is an early sign of the most likely diagnosis? A. Delay in spoken language B. Frequent gesturing to indicate wants C. Interested in new sounds and sensations D. Wanting to be with others all the time

A. Delay in spoken language Frequent gesturing to indicate wants (B) might indicate a speech or hearing disorder. Being interested in new sounds and sensations (C) is not associated with autism spectrum disorder, as those with the disorder typically have difficulty with or unusual reactions to sounds, smells, or other sensations. Wanting to be with others all the time (D) is usually the opposite behavior of individuals with the social-emotional dysfunction associated with autism spectrum disorder.

A 25-year-old woman presents to a therapist due to persistent conflict in close relationships. She reports that one day it feels like her boyfriend is the love of her life, and the next day she wants to end their relationship. She also reports that she has had trouble maintaining a job because she will abruptly become upset with her boss and leave. She has attempted suicide twice in the past 6 months. Which of the following is the first-line treatment for the suspected personality disorder? A. Dialectical behavioral therapy B. Exposure and response prevention therapy C. Fluoxetine D. Lorazepam

A. Dialectical behavioral therapy Exposure and response prevention therapy (B) is a technique used in the treatment of obsessive-compulsive disorder. Exposure therapy involves exposing the patient to thoughts, images, objects, or situations but preventing the ritualistic compulsive behavior. This technique is not used to treat borderline personality disorder. Fluoxetine (C) is a selective serotonin reuptake inhibitor antidepressant. Mood stabilizers are recommended rather than antidepressants to treat patients with borderline personality and depressed mood, dysphoria, mood lability, anxiety, or anger. Lorazepam (D) is in the benzodiazepine class of medications, which may be used to treat anxiety. However, benzodiazepines have the potential for addiction or use in suicide attempts. Therefore, they should not be prescribed to patients with borderline personality disorder because they are at increased risk of misusing medications or attempting suicide. The treatment of borderline personality disorder involves patient education, psychotherapy, and symptom-focused pharmacologic therapy. Patient education consists of educating patients about the symptoms, course, and treatment options. Evidence-based psychotherapies are the primary treatment of borderline personality disorder. Dialectical behavior therapy and mentalization-based therapy are two therapies with evidence to support their use. Dialectical behavior therapy focuses on changing ineffective behaviors and improving coping skills, emotional regulation, and self-management. Mentalization-based therapy focuses on helping the patient understand how their mental status influences behavior. Symptom-focused pharmacologic therapy may be used in patients with borderline personality disorder as an adjunct to psychotherapy. Patients with cognitive-perceptual symptoms as the predominant manifestation, such as paranoid ideation or hallucination, should be treated with low-dose second-generation antipsychotics (e.g., risperidone, olanzapine), while patients with impulsivity and lack of self-control as the predominant symptoms should be treated with mood stabilizers (e.g., lithium, carbamazepine). Patients with borderline personality and a mood disorder, such as unipolar depression, should have treatment focused on borderline personality disorder rather than unipolar depression because this is most likely to alleviate both diagnoses.

A 45-year-old woman presents to the clinic due to insomnia. She has no trouble falling asleep but awakens in the middle of the night and cannot stay asleep. She has been practicing good sleep hygiene according to a prescribed cognitive behavioral approach for the past 6 months but continues to struggle maintaining sleep. Which of the following medications is most appropriate for this patient? A. Doxepin B. Ramelteon C. Triazolam D. Zaleplon

A. Doxepin Ramelteon (B) is a melatonin agonist that is primarily prescribed for patients who have sleep initiation insomnia because it is not effective for sleep maintenance. Common side effects of ramelteon include dizziness, somnolence, fatigue, depression, nausea, myalgias, and upper respiratory infection. Triazolam (C) is a benzodiazepine with a short half-life, making it a poor choice for sleep maintenance. The benzodiazepines as a class are not recommended for chronic treatment of insomnia due to their habit-forming nature and should only be used for 1 or 2 weeks at the most. Zaleplon (D) is a nonbenzodiazepine benzodiazepine receptor agonist with a short half-life that is used for sleep initiation, not sleep maintenance. Side effects of zaleplon can include chest pain, drowsiness, amnesia, anxiety, depression, migraine, nausea, pruritus, dysmenorrhea, tremor, and bronchitis.

A 25-year-old woman presents to the clinic with her husband due to concerns of difficulty sleeping and irritability. Her husband reports his main concern is that she has been calling out of work and has not been wanting to spend time with family because she is always tired. She reports that she recently stopped marijuana use due to being admitted into a master's program and reports occasional alcohol use. Which of the following is the appropriate treatment for this patient? A. Dronabinol B. Flumazenil C. Naloxone D. Relaxation techniques

A. Dronabinol Flumazenil (B) is the appropriate medical treatment for benzodiazepine intoxication. These patients present with drowsiness, confusion, agitation, bradycardia, and respiratory depression. Naloxone (C) is the appropriate treatment for opioid intoxication. Patients with opioid intoxication present with drowsiness, miotic pupils, bradycardia, and hypotension. Relaxation techniques (D), meditation, physical exercise, and improved sleep hygiene are appropriate for patients who have mild cannabis withdrawal symptoms, meaning those whose symptoms do not affect their day- to- day function. The patient in the above vignette is going through cannabis withdrawal. Cannabis use disorder is classified as a persistent use of cannabis that results in a significant functional impairment in two or more settings over the course of 12 months. Typical presentations include using cannabis during potentially hazardous situations, such as driving a car, a decrease in work/school function, or discontinuation of previously enjoyed activities. Typically when withdrawing from cannabis, a patient may be irritable, angry, anxious, or restless. They may also report a decrease in appetite, sleep difficulties, abdominal pain, vomiting, muscle aches, tremors, sweating, fevers, chills, or headaches. For patients who withdraw from cannabis and have mild symptoms that do not affect their daily life, no specific treatment is necessary. However, for patients, such as the one in the above vignette, who are withdrawing from cannabis and their symptoms begin to affect their work or school negatively, medication treatment should be considered. In the United States, the preferred treatment is either dronabinol or gabapentin. Patients who suffer from sleep disturbance may benefit from zolpidem.

A 66-year-old man with a history of hyperlipidemia, type 2 diabetes, and diabetic neuropathy presents for mood disturbance for the last 3 weeks. He reports feeling his life is worthless and has lost interest in activities that used to bring him joy. He has no interest in seeing friends or family and has recurrent thoughts of death, although he has no specific suicidal plan. He reports a 5 lb unintentional weight loss but states he has lost his appetite. He mentions he has not been controlling his glucose over the last 6 months and thinks his neuropathy has worsened. Which of the following medications would be best for this patient? A. Duloxetine B. Gabapentin C. Pregabalin D. Sertraline

A. Duloxetine Gabapentin (B) is an antiepileptic medication that would not be effective in treating major depressive disorder and may only be effective at relieving the neuropathic pain. Pregabalin (C) is similar to gabapentin and may be used to treat neuropathic pain but would be less effective in treating major depressive disorder. Sertraline (D) is used as a first-line treatment for major depressive disorder but is not used in treating diabetic neuropathy. Major depressive disorder is characterized by depressed mood, loss of interest in daily activities, sleep disturbance, weight changes, psychomotor symptoms, fatigue, thoughts of worthlessness, difficulty concentrating, and recurrent thoughts of death. At least five of these symptoms must be present for at least 2 weeks with at least one symptom being either depressed mood or anhedonia to diagnose major depressive disorder. These symptoms have a significant impact on the quality of life and interpersonal relationships. Selective serotonin reuptake inhibitors (SSRIs) are typically used as first-line treatment, but the selection of a medication should consider the side effect profile and the specific patient. Serotonin and norepinephrine reuptake inhibitors (SNRIs) are effective in treating unipolar major depression. These medications work by blocking presynaptic serotonin and norepinephrine transporter proteins, thereby increasing the available serotonin and norepinephrine in the synapse. Duloxetine is a first-line treatment for neuropathic pain. Duloxetine is absorbed by the gastrointestinal tract and metabolized by the liver. This drug is a moderately potent inhibitor of the cytochrome P450 2D6 enzyme. Patients with kidney disease or hepatic impairment should generally not use duloxetine. Common side effects include nausea, dry mouth, constipation, and dizziness.

A 4-year-old boy presents with his mother for his yearly checkup. The patient has a diagnosis of autism spectrum disorder, and his mother reports that, despite at-home treatment, he continues to have difficulty with verbal communication, outbursts of emotion, and socializing with peers. Which of the following has been shown to improve outcomes in children with autism? A. Early behavioral and educational interventions B. Pharmacotherapy C. Psychotherapy D. Supplementation with vitamin B6 and magnesium

A. Early behavioral and educational interventions Pharmacotherapy (B) can be used to treat a variety of symptoms as listed above, but they do not improve the overall outcome of patients and do not treat the underlying disorder. Psychotherapy (C) can be beneficial for some patients as part of a multidisciplinary team but should always be used in conjunction with early behavioral and educational interventions. Supplementation with vitamin B6 and magnesium (D) is not recommended for patients with autism. Very high doses (over 100 mg/day) of vitamin B6 can cause neuropathy.

A 35-year-old man presents to the clinic complaining of sexual dysfunction. He states he is able to obtain nocturnal erections during sleep but has difficulty obtaining and maintaining an erection during intercourse with his wife. He states his sexual desire is low since losing his job. Serum hormone levels and vital signs are within normal limits, and the patient is not on any medications. Which of the following is a serious side effect of the most commonly used medication for this patient's disorder? A. Erection lasting more than 4 hours B. Polycythemia C. Rash D. Severe hypertension

A. Erection lasting more than 4 hours Polycythemia (B) is a common side effect of testosterone replacement therapy. This patient does not suffer hypogonadism, as his serum hormone levels are within normal limits. Therefore, testosterone replacement is not recommended. Rash (C) is a side effect of patch formulations of testosterone replacement therapy and can also occur during use of a vacuum device. Vacuum devices are not considered first-line therapy for hypoactive sexual desire disorder. Severe hypertension (D) is not a common side effect of phosphodiesterase type-5 inhibitors. These medications cause vasodilation through stimulation of endogenous nitric oxide.

A 27-year-old woman is arrested after running onto a concert stage and hugging the main performer. She tells the police that she is the performer's special fan and that they will get married one day. The performer states he does not know the woman. Which of the following types of delusions is this? A. Erotomanic B. Ideas of reference C. Nihilistic D. Paranoid

A. Erotomanic Ideas of reference (B) is the delusion that random or neutral events are not random or neutral but rather have special meaning to the individual. For example, someone may believe that occurrences on the television or radio have special messages for them. Nihilistic (C) refers to the delusion that one is dead or their body is breaking down. Paranoid (D) delusions are seen with patients who have persistent false beliefs that other people are out to get them.

A 52-year-old man presents to the clinic for a wellness exam. Which of the following is the most appropriate frequency for screening this patient for unhealthy alcohol use? A. Every 12 months B. Every 24 months C. Every 6 months D. Every 60 months

A. Every 12 months Alcohol Use Disorder, DSM-5 Dx Problematic pattern of use leading to significant impairment or distress as manifested by at least 2 of 11 possible criteria, such as craving, unsuccessful efforts to control use, resultant physical or psychological problems, tolerance, and withdrawal Combines prior alcohol abuse and alcohol dependence Specifiers for severity, remission status, in a controlled environment The United States Preventive Services Task Force recommends all adults in primary care who have services available for follow-up should be screened annually to identify unhealthy alcohol use. Those with unhealthy use should be counseled on appropriate use. There are several methods of screening available. Some of the most commonly used methods include single-item screening, the Alcohol Use Disorder Identification Test-Consumption (AUDIT-C), and the AUDIT. Single-item screening has the benefit of being brief and easy to memorize and score. One example is asking, "How many times in the past year have you had five (for men, four for women) or more drinks in a day?" The screening test is positive when the response is one or more. The AUDIT-C is a screening test that consists of three questions aimed at assessing the frequency and quantity of alcohol use. The AUDIT is another screening test that has 10 items. Laboratory testing is generally not recommended for unhealthy alcohol use due to the additional cost and because it only detects recent alcohol use

A 26-year-old man is sexually aroused by exposing his genitals to nearby drivers while in traffic. Which of the following paraphilic disorders is the most likely diagnosis? A. Exhibitionistic disorder B. Frotteuristic disorder C. Sexual masochism D. Voyeuristic disorder

A. Exhibitionistic disorder Frotteuristic disorder (B) is a paraphilic disorder defined by recurrent and intense sexual arousal from touching or rubbing one's genitals against a nonconsenting person. Sexual masochism (C) is a paraphilic disorder characterized by sexual arousal from being humiliated. Voyeuristic disorder (D) is a paraphilic disorder characterized by recurrent and intense sexual arousal from observing an unsuspecting person who is naked, disrobing, or involved in sexual activity. Exhibitionistic disorder is a paraphilic disorder characterized by an individual becoming aroused from exposing their genitals to an unsuspecting person. Individuals with exhibitionistic disorder frequently masturbate during or after the exposure. The diagnosis requires either the person to have acted on these urges within the past 6 months with a nonconsenting person or to experience significant distress or impairment. Therefore, individuals who have exhibitionistic urges or desires but do not act on them are not diagnosed with exhibitionistic disorder unless there is significant distress. Exhibitionistic disorder most often begins prior to 18 years of age. It is more prevalent among males, which is consistent with other paraphilic disorders. Treatment options include pharmacotherapy and psychotherapy. Selective serotonin reuptake inhibitors are helpful in some cases, while long-acting agonists of luteinizing hormone and antiandrogens may be used in severe cases. Cognitive behavioral therapy is the recommended psychotherapy. Patients with exhibitionistic disorder who receive treatment frequently improve. However, the course is typically chronic when untreated.

A 20-year-old man with a prior history of recurrent episodes of major depression presents to the emergency department with his family, who has noticed a change in his mood in the past 4 days. They report he has been euphoric, overly confident, and engaging in reckless and uncharacteristic spending. He reports that he has slept 10 hours total the past 5 nights and does not feel tired. On exam, you notice he talks rapidly and bounces from one idea to another. Urine drug screen is unremarkable. Which of the following is the strongest risk factor for the suspected diagnosis? A. Family history of the suspected condition B. Lower socioeconomic status C. Male sex D. Stressful life events

A. Family history of the suspected condition Bipolar disorder occurs more often in patients of higher socioeconomic status, not lower socioeconomic status (B). Men and women are diagnosed with bipolar disorder at a similar frequency, thus male sex (C) is incorrect. Women are more likely to have severe bipolar I disorder or rapid cycling of mood episodes. Stressful life events (D) have not been proven to be a trigger for bipolar I disorder. However, patients often experience stressful life events secondary to manic bipolar disorder mood episodes.

A 22-year-old man with a new diagnosis of schizophrenia presents to the office for medication management. A decision is made to start quetiapine, and you discuss side effects with the patient, along with a need for regular appointments to monitor metabolic changes. Which of the following laboratory tests should be monitored regularly? A. Fasting blood glucose B. Kidney panel C. Liver panel D. Thyroid-stimulating hormone

A. Fasting blood glucose Paliperidone is the only antipsychotic of which 80% is excreted unchanged by the kidneys and 10% is processed in the liver. The dose of paliperidone should be adjusted in patients with kidney dysfunction, and this medication should be avoided in those with severe kidney impairment or injury. Despite this, regular monitoring of a kidney panel (B) is not recommended for patients taking antipsychotics. While most antipsychotic medications are metabolized through the cytochrome P450 system of the liver and caution is advised in prescribing these agents when there is existing hepatic impairment, routine monitoring of the liver panel (C) is not recommended. There are a number of medications that cause thyroid dysfunction, including lithium. Thyroid-stimulating hormone (D) should be monitored in patients taking lithium, but for those taking either first- or second-generation antipsychotics, it is not part of routine laboratory monitoring.

A patient with major depressive disorder is refractory to both first- and second-line therapies. You decide to prescribe selegiline. Which of the following is appropriate patient education regarding this agent? A. Foods that contain tyramine should be strictly eliminated while taking this agent B. Leafy green vegetable intake should be regulated to maintain level serum concentration levels C. Sexual dysfunction is a common side effect D. This agent should be taken concurrently while decreasing the most recent antidepressant dosage

A. Foods that contain tyramine should be strictly eliminated while taking this agent Leafy green vegetable intake should be regulated to maintain level serum concentration levels (B) in patients who are taking warfarin. This statement does not apply to patients taking an MAOI. Sexual dysfunction is a common side effect (C) with many antidepressants but has not been noted as frequently with selegiline and other MAOIs. This agent should be taken concurrently while decreasing the most recent antidepressant dosage (D) is an inaccurate statement. Patients beginning any MAOI agent should completely discontinue their prior antidepressants for at least 2 weeks before initiating MAOI treatment.

A 22-year-old man is admitted to the psychiatric hospital for a condition marked by recurrent psychosis. You examine him and note that he has a flat affect and is apathetic. In addition, he will maintain unnatural postures that he is put into. He was initially admitted after he attempted to break into the White House because he thought he was the president. Which of the following accurately describes the epidemiology of this condition? A. Individuals born in March are at increased risk B. Individuals in transitional countries are at increased risk C. Symptoms begin at a younger age in women D. Women are affected more than men

A. Individuals born in March are at increased risk Individuals in industrialized countries, not transitional countries (B), are at increased risk of schizophrenia. Symptoms begin at a younger age in women (C) is incorrect because symptoms occur earlier in men than in women, on average. Women are affected more than men (D) is incorrect, as men are affected 1.4 times as often as women.

A 26-year-old female arrives in the emergency department with friends who say she was standing in front of her church, dressed in a white bathrobe, claiming to be the Virgin Mary and handing out $100 bills to all passers-by. Her friends noted that she had been depressed lately but now seems completely euphoric. She had a similar episode two years ago. Which of the following is the most appropriate treatment? A. Inpatient olanzapine (Zyprexa) therapy B. Inpatient electroconvulsive therapy C. Outpatient fluoxetine (Prozac) therapy D. Outpatient psychotherapy

A. Inpatient olanzapine (Zyprexa) therapy Treatment of the manic phase is usually done in the hospital to protect patients from behaviors associated with grandiosity (spending inordinate amounts of money, making embarrassing speeches, etc.). Lithium, valproate, and olanzapine are considered effective in the manic stage; the depressive stage is treated monotherapy using quetiapine or lurasidone. If quetiapine monotherapy and lurasidone monotherapy are each ineffective or intolerable, next-step treatment options may include an SSRI as combination therapy (olanzapine plus fluoxetine)

A 27-year-old woman with bipolar I disorder is evaluated in the office upon urgent request. She was recently started on a new medication after having difficulty tolerating lithium. The patient reports a concerning skin rash that developed over the past 36 hours. She is concerned it may be an allergic reaction related to her new medication and indicates she did not slowly titrate the dose as indicated. On her physical exam, poorly defined erythematous macules and plaques with purpuric necrotic centers are noted on the trunk and the left cheek. With what medication is this reaction most commonly associated? A. Lamotrigine B. Lurasidone C. Quetiapine D. Valproate

A. Lamotrigine The goal of bipolar treatment is to reduce the risk of relapses and improve mood symptoms. Monotherapy with antidepressants is contraindicated due to the increased risk of manic episodes. Patients with acute mania should be hospitalized due to the risk of harm to themselves and potentially others. Treatment in acute mania is focused on the use of mood stabilizers, such as lithium or valproate. Antipsychotics, such as haloperidol, can also be used. In patients with acute depression, lithium, lamotrigine, and quetiapine may be considered. Antidepressants may be added if a therapeutic dosage of a mood stabilizer does not fully resolve depressive symptoms. Several medications are effective for maintenance therapy. First-line drug choice for maintenance therapy is whichever drug resolved the acute episode. Second-line drug choices include lithium, lamotrigine, valproate, and quetiapine. Maintenance therapy should be considered in all patients with bipolar disorder. Patients should be observed regularly to evaluate for improving mood and sleep symptoms, as well as potential side effects of drug therapy. Quetiapine has increased risks of parkinsonism and QT prolongation. Valproate has an increased risk of weight gain, tremor, hair loss, and liver failure and is contraindicated in pregnancy. Lithium can lead to diabetes insipidus and hypothyroidism and can also cause tremor. Lurasidone (B) is a second-generation antipsychotic approved for acute bipolar depression or used in combination for manic episodes. It is typically considered in patients who are refractory to first- and second-line drug options. It is considered off-label for maintenance therapy. It is not associated with Stevens-Johnson syndrome. Quetiapine (C) has rare postmarketing reports of Stevens-Johnson syndrome but is not associated with the rash to the same degree that lamotrigine is. Lamotrigine has a long titration schedule due to the risk of rash development. Quetiapine can cause QT prolongation and is included in the Beers criteria. Valproate (D) has risks of hepatotoxicity and congenital malformations. It should not be used in women of childbearing age. Similar to quetiapine, valproate has rare postmarketing reports of Stevens-Johnson syndrome occurring < 1% of the time.

A 6-year-old boy presents to the clinic for evaluation of a conduct disorder. His mother states he has been irritable and angry several days a week for the past year. When he is irritable, he defies her and his teachers, argues with everyone, and deliberately annoys his siblings. His grades are poor, and his friendships are few due to his anger and defiance. At least once per month, he acts vindictively toward his mother but has never harmed a person or animal. Which of the following environmental elements is more common in patients with this boy's most likely disorder? A. Maternal aggression B. Overprotective parenting C. Paternal absence D. Rigid structure in the home

A. Maternal aggression Overprotective parenting (B) is not a risk factor for the development of oppositional defiant disorder, although there seems to be a correlation between overprotective parenting and anxiety, depression, or withdrawal. Paternal absence (C) is not a risk factor for oppositional defiant disorder unless the mother or remaining guardian has a psychopathology or is abusive or the family is reduced to poverty due to the absence. Rigid structure in the home (D) tends to help patients with oppositional defiant disorder and attention-deficit/hyperactivity disorder, whereas lack of structure can contribute to these disorders or worsen their symptoms.

A patient with obsessive-compulsive disorder would most likely have which of the following findings? A. Raw, red hands B. Priapism C. Memory impairment D. Abdominal pain

A. Raw, red hands Common manifestations of obsessive-compulsive disorder include phobias of germ and contaminants, which results in frequent hand washing leading to chafe and reddened hands. The other answers are inconsistent with obsessive-compulsive disorder.

A 22-year-old man presents with "hearing voices." He has had progressively worsening symptoms over the last 7 months. He has heard voices saying negative statements that seem to come from inside of his head. He has also had a belief that his neighbors are trying to poison him and have planted cameras and listening devices inside his house. When speaking about these symptoms, he becomes distracted easily and gets off topic quickly. He is also speaking quickly and is very talkative. He notes he has not slept in 2 days but does not feel tired. He reports no substance use or use of other medications. What is the most likely diagnosis? A. Schizoaffective disorder B. Schizoid personality disorder C. Schizophrenia D. Schizophreniform disorder

A. Schizoaffective disorder Schizoaffective disorder is a psychotic disorder that is associated with significant impairment in functional capabilities. Characteristics include positive and negative symptoms, cognitive impairment, and mood disturbances. Positive symptoms include hallucinations and delusions. Disorganized thought content and processes are also a manifestation of a positive symptom and can include tangential or circumstantial speech and derailment. Negative symptoms are defined as the absence of normal behavior and include decreased expression, apathy, and a flat affect. Cognitive impairment usually affects memory, learning abilities, attention, and processing speed. Diagnosis is often one of exclusion of other differential diagnoses. The delusions and hallucinations specific to schizoaffective disorder are called first-rank symptoms. These symptoms include audible hallucinations, thought insertion or withdrawal, and audible self-thoughts. Diagnosis is made when two or more of the characteristic symptoms are present for at least 1 month and significant impairment is present for at least 6 months. Schizoaffective disorder is distinguished from schizophrenia by the presence of manic episodes or a significant depressive episode in addition to the symptoms listed above. Schizoid personality disorder (B) is characterized by the absence of normal behavior and social relationships in a long-standing and pervasive pattern. There is no psychosis with this disorder. Schizophrenia (C) is similar to schizoaffective disorder, but there are no manic or depressive episodes associated with schizophrenia. Schizophreniform disorder (D) is similar to schizoaffective disorder, but the total duration of the disorder is < 6 months and there is no mania associated with schizophreniform disorder.

Early clues to impending delirium tremens include A. agitation and decreased cognition. B. visual hallucinations and diaphoresis. C. autonomic hyperactivity and dehydration. D. mental confusion and sensory hyperacuity.

A. agitation and decreased cognition. Anxiety, decreased cognition, tremulousness, increasing irritability, and hyperactivity are common early clues to impending delirium tremens.

A 54 year-old patient has acute onset of palpitations, tremulousness, profuse sweating, shortness of breath, and numbness and tingling of the extremities. Physical examination reveals a pulse of 104 beats/min and regular, respiratory rate of 30/min, blood pressure of 160/95 mm Hg. Arterial blood gases reveal a low pCO2. ECG shows no acute changes. The most likely diagnosis is A. anxiety neurosis. B. atrial fibrillation. C. depressive neurosis. D. myocardial infarction

A. anxiety neurosis. Anxiety is characterized by short-lived, recurrent, unpredictable episodes of intense anxiety accompanied by marked physiological manifestations.

The most important initial component of evaluating a patient with depressive illness is A. assessment of suicidal risk. B. assessment of memory function. C. determination of the presence of hallucinations. D. determination of the degree of intellectual impairment

A. assessment of suicidal risk. Two-thirds of all depressed patients contemplate suicide, and 10 to 15% commit suicide.

Which of the following medications used in the management of anxiety has a delayed onset of action? A. buspirone (BuSpar) B. diphenhydramine (Benadryl) C. lorazepam (Ativan) D. butalbital (Fiorinal)

A. buspirone (BuSpar) Buspirone takes several days to weeks for it to have clinical activity.

A 20 year-old female presents with episodes of binge eating, overuse of laxatives, and periods of starvation. Which of the following is the best treatment option for this patient? A. fluoxetine (Prozac) B. gabapentin (Neurontin) C. amitriptyline (Elavil) D. phenelzine (Nardil)

A. fluoxetine (Prozac) Fluoxetine, a SSRI, is the drug of choice for the treatment of bulimia nervosa

A child has been under treatment for attention-deficit hyperactivity disorder (ADHD). No response has occurred with behavioral adaptations. Which of the following categories of medication should this patient be given? A. stimulants B. anxiolytics C. antipsychotics D. antidepressants

A. stimulants Stimulants, such as Ritalin, Dexedrine, and Cylert, are effective in 50 to 80% of children with ADHD.

A 65 year-old patient has a long history of schizophrenia that is treated with phenothiazines. On an unrelated clinic visit, the patient has difficulty sticking out her tongue, facial tics, increased blink frequency, and lip-smacking behavior. These involuntary movements are most suggestive of A. tardive dyskinesia. B. Parkinson's disease. C. Huntington's disease. D. Tourette's syndrome.

A. tardive dyskinesia. Tardive dyskinesia is characterized by abnormal involuntary movements of the face, mouth, tongue, trunk, and limbs and may develop after months or years of treatment with neuroleptic drugs.

How are Stevens-Johnson syndrome and toxic epidermal necrolysis distinguished?

According to the total percent of body epidermal involvement. Cases with < 10% of epidermal involvement are diagnosed as Stevens-Johnson syndrome, cases with > 30% are diagnosed as toxic epidermal necrolysis, and cases with 10-30% are considered overlap.

What is the most common type of extrapyramidal symptom?

Akathisia

Which antihypertensive medications are sometimes used in the treatment of post-traumatic stress disorder?

Alpha-blockers and clonidine Post-traumatic stress disorder is characterized by a prolonged psychologic and physiologic response to a traumatic experience. The response is somatic, behavioral, cognitive, and affective, involving reliving the inciting incident, flashbacks, nightmares, impaired sleep, hypervigilance, and avoiding triggers. Triggers may include people, places, or situations that bring the traumatic event to mind. The inciting traumatic experience is most often sexual relationship violence but may also include the death or life-threatening illness of a loved one, interpersonal violence, participation in or exposure to group violence, or other life-threatening events, such as a natural disaster, motor vehicle collision, or prolonged hospital stay. Predisposing factors in the development of post-traumatic stress disorder include young age at time of trauma; adverse childhood events; female sex; being widowed, separated, or divorced; lower socioeconomic status; lower education level; personal or family history of psychiatric disease; poor social support; and a severe stress reaction at the time of the inciting event. Treatment of post-traumatic stress disorder should start with trauma-focused psychotherapy with exposure and may also include selective serotonin reuptake inhibitors. Post-Traumatic Stress Disorder (PTSD) Sx duration > 1 month Persistently reexperiencing of event Persistently ↑ arousal Avoidance of stimuli ↑ risk for suicide, substance use

Which personality disorders are in cluster C?

Avoidant, dependent, and obsessive-compulsive.

A 20-month-old boy presents to the pediatrician with his mother, who is concerned that he does not seem interested in interacting with her. She has also noticed that he does not like to make eye contact and has intense interests in visual stimuli, such as flashing lights. He seems easily irritated by small changes in the daily routine. At which of the following months does the American Academy of Pediatricians recommend screening for the suspected diagnosis? A. 12 and 18 months B. 18 and 24 months C. 24 and 30 months D. 30 and 36 months

B. 18 and 24 months 12 and 18 months (A), 24 and 30 months (C), and 30 and 36 months (D) are incorrect. The American Academy of Pediatrics recommends screening for autism spectrum disorder at 18 and 24 months.

Which of the following patients would be most likely to present with generalized anxiety disorder? A. 16-year-old boy B. 21-year-old woman C. 35-year-old man D. 45-year-old woman

B. 21-year-old woman A 16-year-old boy (A) is not the most likely to present with generalized anxiety disorder. A 35-year-old man (C) is not the most likely to present with generalized anxiety disorder. Typical presentation is in a young adult woman. A 45-year-old woman (D) is not the most likely to present with generalized anxiety disorder. Generalized anxiety disorder is more common in women than men but is most likely to present in younger women. Generalized Anxiety Disorder (GAD) Excessive worry Persistent symptoms Difficult to control feelings Symptoms last over 6 months Treat with CBT, SSRI

A 38-year-old man presents to the emergency department with his wife. The patient is in an altered mental state and continues to report there are spiders crawling on him. His vitals are heart rate of 138 bpm, blood pressure of 158/96 mm Hg, and respiratory rate of 20 breaths per minute. He appears angry and diaphoretic on exam. The patient's wife reports heavy alcohol use at home. Which of the following timeframes is consistent with this patient's presenting symptoms? A. 12-48 hours after the last drink B. 48-96 hours after the last drink C. 6-24 hours after the last drink D. 6-48 hours after the last drink

B. 48-96 hours after the last drink The patient in the above vignette is going through alcohol withdrawal, specifically delirium tremens, which is an acute onset of confusion that occurs between 48-96 hours after the patient's last alcoholic drink. Risk factors include a history of drinking, a history of previous alcohol withdrawal seizures, a history of delirium tremens, and age > 30 years old. Delirium tremens presents as fluctuating disturbance of attention and cognition, sometimes with f hallucinations, disorientation, tachycardia, hypertension, agitation, fever, and diaphoresis after discontinuing alcohol. Hypomagnesemia is also common in patients with delirium tremens, which increases their risks for seizures and dysrhythmias. Delirium tremens is considered a medical emergency. Treatment is with immediate administration of IV benzodiazepines, such as diazepam or lorazepam, along with IV fluids and supplements, such as thiamine, folic acid, and multivitamins. 12-48 hours after the last drink (A) a patient may develop alcoholic hallucinations with normal and stable vitals. Although the patient in the above vignette is having hallucinations, this time period is not appropriate because the patient's vitals are unstable and he is experiencing delirium tremens. 6-24 hours after the last drink (C) symptoms of alcohol withdrawal begin. Patients may present with symptoms of increased CNS activity, such as tremors, anxiety, sweating, palpitations, and GI upset. These are all appropriate symptoms for an uncomplicated alcohol withdrawal presentation. Withdrawal seizures usually occur 6-48 hours after the last drink (D). The patient in the above vignette is not experiencing a seizure and thus this timeframe is not appropriate.

A 27-year-old man presents to the clinic for evaluation of a contused hand. He states he punched his mother in the face earlier in the day and now his hand hurts. The patient shows no remorse for the act of violence. He reports dropping out of school early, being fired from several jobs, having many girlfriends but for only 1 or 2 weeks at a time, and frequently breaking the law. He appears proud that he has been incarcerated several times. Which of the following is a risk factor for the development of this patient's most likely personality disorder? A. A brother with autism B. A father with antisocial personality disorder C. Early exposure to violent video games D. Overly indulgent parents

B. A father with antisocial personality disorder A brother with autism (A) is not a risk factor for the development of antisocial personality disorder. Schizoid personality disorder may be confused with autism, as patients with schizoid personality disorder do not enjoy physical or social relationships. Early exposure to violent video games (C) has been posited as a risk factor for the development of antisocial personality disorder, but no statistically significant relationship has been found. Overly indulgent parents (D) are generally not a risk factor for the development of antisocial personality disorder, but parents who do not adequately supervise their children may contribute to the development of the disorder. Abusive parents or parents who demonstrate antisocial behavior themselves are more likely to have children with antisocial personality disorder than overly indulgent parents.

A 24-year-old man with a previous history of alcohol use disorder presents to the clinic reporting symptoms of difficulty concentrating, forgetfulness, restlessness, carelessness, and fidgeting during class. He reports having similar symptoms when he was a child. He recently reenrolled in school and is concerned that he may not be able to pass his classes. Which of the following is the most appropriate treatment for this patient? A. Amphetamine B. Atomoxetine C. Methylphenidate D. Modafinil

B. Atomoxetine Amphetamine (A) and methylphenidate (C) are both appropriate treatment for patients with attention-deficit/hyperactivity disorder who do not have a previous history of substance use. Both of these medications are classified as schedule II drugs and therefore have a high potential for misuse or addiction. Modafinil (D) is a stimulant like the other attention-deficit/hyperactivity disorder medications. However, it is only approved to be used for narcolepsy and shift work sleep disorder.

A 50-year-old woman presents to the office due to diarrhea, abdominal pain, fever, and weight loss over the past year. She tells you she works as a nurse and is very health conscious. She says she saw a gastroenterologist 3 months ago, the workup was unremarkable, and she gives you photocopied records of the workup for review. Vital signs reveal a temperature of 98°F, blood pressure of 125/68 mm Hg, and pulse of 105 bpm. When she sees the temperature, she says the clinic thermometers are always wrong, and she has her own thermometer she wants you to use. Which of the following supports the suspected diagnosis? A. Age of the patient B. Attempt to falsify vital signs C. Previous visit with a specialist D. Type of symptoms described

B. Attempt to falsify vital signs While the age of onset for factitious disorder imposed on self is typically 30 to 40 years old, age of the patient (A) does not point to this diagnosis. Patients with factitious disorder imposed on self typically have a high rate of utilization of medical care and often see numerous specialists, but one previous visit with a specialist (C) is not evidence to suggest this diagnosis. There are a variety of symptoms, both medical and psychiatric, that individuals with factitious disorder imposed on self describe, so the type of symptom described (D) is not a part of clinical suspicion for the disorder.

A 24-year-old man presents to a therapist because he lacks confidence in social settings. Despite having a desire for close relationships, he states that, for years, he has avoided most social situations due to a fear of embarrassment. Which of the following personality disorders is the most likely diagnosis? A. Antisocial B. Avoidant C. Dependent D. Schizoid

B. Avoidant Antisocial (A) personality disorder is marked by a persistent pattern of exploitation of others and breaking rules. Individuals with antisocial personality disorder lack empathy for others and remorse for their actions. Dependent (C) personality disorder is characterized by an excessive need to be taken care of by others and difficulty with decision-making. It is also a cluster C personality disorder. Schizoid (D) personality disorder is marked by a persistent pattern of social withdrawal due to a lack of interest in close relationships. Individuals with schizoid personality disorder are often considered to be eccentric and reclusive. Avoidant personality disorder is marked by feelings of inadequacy, hypersensitivity to criticism or embarrassment, and avoidance of social situations. It is one of the cluster C personality disorders, which also includes dependent personality disorder and obsessive-compulsive personality disorder. Individuals with avoidant personality disorder avoid social interactions despite having a desire for companionship. This contrasts with schizoid personality disorder, in which individuals avoid social interactions because they do not desire close relationships. Individuals with avoidant personality disorder typically choose occupations with limited social interactions. Social anxiety disorder is a common comorbidity of individuals with avoidant personality disorder. It occurs with equal frequency in men and women. The fifth edition of the Diagnostic and Statistical Manual of Mental Disorders (DSM-5) diagnostic criteria for avoidant personality disorder include a pervasive pattern of social inhibition, feelings of inadequacy, and hypersensitivity to negative evaluation, which begin by early adulthood and present in a variety of contexts. Psychotherapy with social skills training is the most effective treatment of avoidant personality disorder.

Which of the following is the most appropriate intervention for a patient suffering from a specific phobia, such as fear of snakes? A. Lithium B. Behavioral therapy C. Insight-oriented therapy D. Electroconvulsive therapy

B. Behavioral therapy Specific phobias most commonly are treated with behavioral therapy including exposure therapy utilizing systemic desensitization. Hypnosis, supportive therapy and family therapy may also be useful adjunct treatment.

A 22-year-old woman with obesity presents to the clinic for episodes in which she eats abnormally large portions of food. She reports feeling a lack of control during these episodes and feels guilty afterward about her eating. The patient reports no self-induced vomiting and is not fasting, taking medications to lose weight, or participating in extreme exercise. These episodes have been occurring about five times per week for the past year. Which of the following is the most likely diagnosis? A. Anorexia nervosa B. Binge eating disorder C. Bulimia nervosa D. Prader-Willi syndrome

B. Binge eating disorder Anorexia nervosa (A) is incorrect. One of its key features is restricted caloric intake that leads to an abnormally low body mass index. The patient in this vignette has obesity with episodes of binge eating. Furthermore, patients with anorexia nervosa often have compensatory behaviors to prevent weight gain. Patients with either disorder can have an intense fear of gaining weight. Bulimia nervosa (C) is distinguished from binge eating disorder by the presence of compensatory mechanisms, such as purging, laxative use, diuretic use, starvation, and excessive exercise, to prevent weight gain following binge eating episodes. Prader-Willi syndrome (D) is a genetic disorder that results in obesity. However, it often presents during infancy with neonatal hypotonia. Infants often have difficulty feeding, which may cause failure to thrive. Toddlers typically have delays in major developmental milestones. Children with Prader-Willi syndrome typically have short stature and develop obesity if access to food is not restricted. Prader-Willi syndrome is unlikely to be the cause of this patient's presentation because the onset of the binge eating episodes began during adulthood. Treatment options include psychotherapy (preferably cognitive behavioral therapy), behavioral weight loss therapy, and pharmacotherapy. Psychotherapy is the preferred first-line treatment. Behavioral weight loss therapy is recommended for patients who do not have access to psychotherapy, decline psychotherapy, or do not improve with psychotherapy. It consists of improving lifestyle measures to promote weight loss, such as implementing moderate caloric restriction, increasing aerobic exercise, and improving nutrition. There are several pharmacologic agents that may be used for the treatment of binge eating disorder. Selective serotonin reuptake inhibitors, such as fluoxetine, are often first line. However, topiramate, zonisamide, and lisdexamfetamine may also be used.

A 25-year-old man with a history of recurrent major depressive episodes presents to his psychiatrist with racing thoughts for the past 5 days. He had a similar episode a month ago that lasted 6 days. He describes his mood as euphoric and states he has increased confidence in himself. He reports he has only needed to sleep around 3 hours and has spent hours on the phone catching up with old friends. He also reports plans to start an automotive repair business out of his garage and has created social media advertisements. He reports no drug use. Which of the following is the most likely diagnosis? A. Bipolar I disorder B. Bipolar II disorder C. Borderline personality disorder D. Narcissistic personality disorder

B. Bipolar II disorder Bipolar I disorder (A) is diagnosed in patients who have had a manic episode. Manic episodes and hypomanic episodes have similar clinical features but are distinguished by the severity of the episode. Any episode that has psychotic features (delusions or hallucinations) or is severe enough to require hospitalization is a manic episode. Manic episodes markedly impair an individual's ability to function, whereas hypomanic episodes either improve function or mildly impair function. The diagnostic criteria for a manic episode require a symptom duration of 7 days, whereas the criteria for a hypomanic episode only require 4 days of symptoms. Borderline personality disorder (C) and narcissistic personality disorder (D) are each marked by pervasive personality patterns rather than an acute episode. They are cluster B personality disorders. The core features of narcissistic personality disorder are grandiosity, hypersensitivity to negative feedback, exaggeration of accomplishments, lack of empathy, and lack of consideration of others. The core features of borderline personality disorder are unstable mood, behavior, and interpersonal relationships; fear of abandonment; intense attachment; suicidal behavior; and poorly formed identity.

A patient presents with an episode of an expansive, elevated mood during which she cleaned excessively without sleeping. Which of the following is the most likely diagnosis? A. Major depressive disorder B. Bipolar disorder C. Schizoaffective disorder D. Dysthymic disorder

B. Bipolar disorder Bipolar disorder is characterized by episodic mood shifts from depression to manic type moods which is often rapid with depression lasting longer than manic episodes. Bipolar disorder may initially present with a manic episode.

A 22-year-old woman presents to clinic for evaluation. At her annual cleaning, the dentist noted she had significant erosion over the lingual aspect of her teeth, and the patient reported episodes of self-induced vomiting when she "eats too much at one time." Which of the following personality disorders is most likely to be present, given the suspected diagnosis? A. Antisocial B. Borderline C. Dependent D. Obsessive-compulsive

B. Borderline Antisocial (A) personality disorders are not associated with the development of bulimia nervosa. Dependent (C) personality disorders are not the most common personality disorder to be associated with bulimia nervosa. About 10 percent of patients diagnosed with bulimia nervosa also have a diagnosis of a dependent personality disorder. Obsessive-compulsive (D) personality disorders are present in about 9 percent of patients diagnosed with bulimia nervosa compared to 21 percent of patients with a borderline personality disorder. Bulimia nervosa is characterized by binge eating with associated purging activities. Diagnostic criteria of bulimia are the following: episodes of binge eating an unusually large amount of food in a short period of time; binge eating is uncontrollable; inappropriate compensatory behavior to prevent weight gain, such as laxative misuse or self-induced vomiting, at least once a week for 3 months; unduly influenced by body shape and weight; and disturbance not occurring only during episodes of anorexia nervosa. Patients typically have a history of lifetime psychiatric disorders, where anxiety disorders typically precede a diagnosis of bulimia and depression or substance use disorders occur as a consequence of the bulimia. It is more common in women and patients with borderline personality disorder. Patients with electrolyte disturbances or suicidal ideation are candidates for inpatient hospitalization, but otherwise, most patients can be treated as an outpatient. Outpatient treatment should include a variety of approaches. Cognitive behavioral therapy or interpersonal psychotherapy are recommended, and group therapy has been shown to have a high degree of success. If cognitive therapy alone produces a suboptimal response, pharmacotherapy can be indicated. Fluoxetine is the only FDA-approved medication for the treatment of bulimia, but antidepressants of all classes can reduce the symptoms of bulimia in patients without depression. Bupropion has a black box warning indicating it can increase seizures in patients with bulimia and should be avoided.

A 35-year-old woman presents to the clinic with suicidal thoughts. She states she has been an impulsive person throughout her entire life and has always had difficulties with interpersonal relationships. Which of the following personality disorders carries an increased risk of suicide? A. Avoidant personality disorder B. Borderline personality disorder C. Obsessive-compulsive personality disorder D. Schizoid personality disorder

B. Borderline personality disorder Avoidant personality disorder (A) is characterized by avoidance of social situations due to hypersensitivity to criticism, restraint in interpersonal relationships due to fear of ridicule, feelings of inadequacy, and preoccupation with embarrassment. Impulsivity is not characteristic of avoidant personality disorder, and suicidality is not associated with this personality disorder. Obsessive-compulsive personality disorder (C) does not increase a patient's risk for suicide. Characteristics of obsessive-compulsive personality disorder include rigidity, perfectionism, and preoccupation with orderliness. Schizoid personality disorder (D) is not a risk factor for suicide but does cause difficulty in interpersonal relationships. Patients with schizoid personality disorder do not desire close relationships or intimacy, demonstrate emotional coldness, and do not seem to be affected by the criticism of others.

A 26-year-old woman presents to the women's health clinic complaining of decreased sexual desire and pain during intercourse. She reports that she does not become lubricated during sexual activity and has never had an orgasm. Which of the following is part of the initial treatment for the suspected diagnosis? A. Abstinence B. Bringing novelty to the sexual repertoire C. Flibanserin D. Topical estrogen

B. Bringing novelty to the sexual repertoire Abstinence (A) is not recommended as part of the treatment of female sexual interest and arousal disorder. Flibanserin (C) is a pharmacologic agent used to treat premenopausal women with sexual interest and arousal disorder. It works by altering serotonin and dopamine concentrations in certain areas of the brain. Flibanserin results in a small increase in sexual desire and sexually satisfying events. One of the drawbacks of flibanserin is that it is a daily medication that cannot be taken within hours of drinking alcohol. Pharmacologic therapy for women with sexual interest and arousal disorder should not be attempted until women have tried more conservative measures. Topical estrogen (D) is indicated in postmenopausal women with isolated symptoms of vaginal dryness and dyspareunia after menopause. Female sexual interest and arousal disorder has various manifestations, including lack of sexual desire, impaired arousal, pain with sexual activity, and inability to achieve orgasm. The etiology of these complaints are often complex and may include relational strife; psychiatric comorbidities (e.g., depression, anxiety, and substance misuse); prior physical, emotional, or sexual abuse; pelvic floor issues; genitourinary syndrome of menopause; and medication side effects. The diagnosis of female sexual interest or arousal disorder requires that these symptoms cause significant distress. The initial treatments focus on improving physical and psychological health, the quality of the relationship between the patient and her sexual partner, and other nonpharmacologic interventions. Reducing fatigue and stress may improve sexual function. Furthermore, weight loss may improve sexual function in women whose struggle with body image contributes to their sexual dysfunction. It is important for women who experience sexual dysfunction to spend quality time with their sexual partner. Consistent date nights may lead to improved sexual function. In addition, bringing novelty to the sexual relationship can improve sexual satisfaction. This can be accomplished by trying new sexual positions, having sexual intercourse in new locations, or by using sex items, such as vibrators. Additional nonpharmacologic interventions include the application of vaginal lubricants and moisturizers prior to sexual intercourse. Women with sexual dysfunction often benefit from seeing a couples or sex therapist with their partner. Sex therapists are counselors who are highly trained in human sexuality and can offer practical tips and exercises to improve sexual function. Women with pelvic floor hypertonus (vaginismus) often benefit from the use of vaginal dilators. Hormonal treatment options include estrogen and androgens (testosterone). Topical estrogens are often used in postmenopausal women with complaints of vaginal dryness and dyspareunia due to genitourinary syndrome of menopause. Androgens have shown some benefit in postmenopausal women with sexual dysfunction, but the contraindications include having a history of cardiovascular disease, liver disease, endometrial hyperplasia, endometrial cancer, or breast cancer. There are several serotonergic or dopaminergic pharmacologic treatments that may help females with sexual dysfunction. These include flibanserin, bupropion, and buspirone. Bupropion is often used off-label (without Food and Drug Administration approval) to treat sexual dysfunction that is induced by selective serotonin reuptake inhibitors. There is some data showing that buspirone, which is typically used as an augment to selective serotonin reuptake inhibitors in the treatment of generalized anxiety disorder, is helpful in the treatment of decreased libido.

A 27-year-old man presents to the psychiatric emergency department with auditory and visual hallucinations. He believes he is the president and is under attack from a foreign country. During your examination, you notice the patient has a disheveled appearance and gets further off-topic when asked questions. You review the patient's prior medical records and notice he has had several similar episodes in the past decade and has always had negative urine drug screens. Which of the following is a risk factor for the suspected diagnosis? A. Birth during the summer months B. Cannabis use C. Female sex D. Paternal age < 20 years at birth

B. Cannabis use Birth during the summer months (A) is incorrect because schizophrenia occurs more often in individuals born during the late winter or early spring, which is thought to be due to increased viral exposures during the second trimester. Female sex (C) is incorrect because schizophrenia occurs more often in men. Men have more severe symptoms on average because women tend to have better premorbid function, fewer negative symptoms, more complete recovery from symptoms, better social function, lower risk of suicide, and better response to antipsychotics. Paternal age < 20 years at birth (D) is incorrect. Paternal age > 50 years increases the risk of schizophrenia, which is thought to be due to an increased risk of de novo mutations.

A 52-year-old woman presents with concerns for mood changes. She states she has had feelings of hopelessness and has been having difficulty sleeping for the last month. She has lost weight due to a poor appetite and has lost interest in daily activities. She has a medical history significant for a seizure disorder and controlled hypertension. Which of the following medications would be best for this patient in addition to psychotherapy? A. Amitriptyline B. Citalopram C. Phenelzine D. Venlafaxine

B. Citalopram Unipolar major depression is characterized by depressed mood, loss of interest in daily activities, sleep disturbance, weight changes, psychomotor symptoms, fatigue, thoughts of worthlessness, difficulty concentrating, and recurrent thoughts of death. There must be at least five of these symptoms present for at least 2 weeks with at least one symptom being either depressed mood or anhedonia to diagnose unipolar major depression. These symptoms cause a significant impact on the quality of life and interpersonal relationships. Assessment should include laboratory testing to rule out physiologic and pathologic causes of depression. Treatment goals should focus on restoring baseline function and the ability to participate in activities of daily living without impairment. Initial therapy should include psychotherapy and pharmacological therapy. Many antidepressants have demonstrated efficacy in treating unipolar major depression. Selective serotonin reuptake inhibitors (SSRIs) are usually used as first-line treatment, but the selection of a medication should consider the side effect profile and the specific patient. Citalopram is an SSRI that has a side effect profile that includes headache, nausea, nervousness, sexual dysfunction, and weight gain. Despite these side effects, SSRIs are generally well tolerated, although at high doses, they may cause QT prolongation. Amitriptyline (A) is a tricyclic antidepressant that may lower the seizure threshold, making this an unsuitable choice for an individual with a history of seizure disorder. Phenelzine (C) is a monoamine oxidase inhibitor and is not used first line due to the extensive side effect profile and dietary restrictions associated with this drug. Venlafaxine (D) is a serotonin and norepinephrine reuptake inhibitor that can cause dose-related hypertension and requires monitoring of blood pressure while taking this medication.

A 32-year-old man with a history of heavy alcohol use presents to the emergency department with anxiety, tremulousness, diaphoresis, vomiting, and headache. He reports that he has heavily used alcohol for 6 years but abruptly stopped yesterday afternoon. Which of the following can be used to assess this patient's symptom severity and determine the most appropriate site for treatment? A. Alcohol Use Disorders Identification Test B. Clinical Institute Withdrawal Assessment for Alcohol C. Cut down, annoyed, guilty, eye opener questions D. Overuse, worry, losing interest, and sedation screening tool

B. Clinical Institute Withdrawal Assessment for Alcohol The Alcohol Use Disorders Identification Test (A), also known as AUDIT, is a 10-item screening tool that can be used in primary care settings to screen adults for alcohol use disorder. The scores on the AUDIT can range from 0 to 40. Scores of at least 8 are considered to suggest unhealthy alcohol use, and scores of at least 20 suggest the presence of alcohol dependence. Cut down, annoyed, guilty, eye opener (CAGE) questions (C) are a series of four questions that can be used to further assess patients who screen positive for possible unhealthy alcohol use on a single-item screening question. The questions include asking patients whether they feel they should cut down on drinking, if they have been annoyed by others criticizing their drinking, if they have ever felt bad or guilty about drinking, and if they have ever taken a drink first thing in the morning (eye opener). Answering yes to two or more of the CAGE questions suggests alcohol misuse or dependence. The overuse, worry, losing interest, and sedation (OWLS) screening tool (D) is a four-item self-administered screening tool used to detect prescription opioid use disorder in individuals who are prescribed long-term opioid therapy. Alcohol Withdrawal Syndrome Early symptoms: anxiety, nausea, restlessness Risk of seizure Delirium tremens may start after 48 hours Treat with benzodiazepines Correct nutritional deficiencies Offer alcohol cessation support and resources

A 54-year-old man presents to the emergency department after being found unconscious in the street. He has a history of intravenous heroin use and was given naloxone en route to the emergency department. The patient is alert, irritable, and restless on presentation. Vital signs include a temperature of 98.6°F, heart rate of 115 bpm, and blood pressure of 165/94 mm Hg. You notice pupil dilation, yawning, and piloerection during physical examination. Which of the following is the recommended treatment? A. Buprenorphine B. Clonidine C. Methadone D. Morphine

B. Clonidine Buprenorphine (A) is a partial agonist of opioid receptors. Partial agonists bind to the receptors but only have a partial effect on the receptor compared to a full agonist. Buprenorphine is one of the opioid replacements that can be administered to patients who have an unwanted interruption in opioid use. Methadone (C) is a long-acting opioid receptor agonist used for opioid replacement therapy. It has a long duration of action, which helps it to prevent opioid withdrawal symptoms. Morphine (D) is a naturally occurring opioid used to provide analgesia. It is not used to treat opioid withdrawal due to its short duration of action.

A 28-year-old man presents to the emergency department with fatigue and chest pain. Electrocardiogram reveals nonspecific ST changes. Serum erythrocyte sedimentation rate and cardiac troponin are elevated. Physical exam reveals pitting lower extremity edema. The patient states he was recently started on a medication to control hallucinations, self-injurious behavior, and delusions of persecution. Which of the following medications is most likely causing his current symptoms? A. Amitriptyline B. Clozapine C. Olanzapine D. Risperidone

B. Clozapine Amitriptyline (A) is a tricyclic antidepressant that is not indicated in the treatment of psychosis or schizophrenia. Amitriptyline may cause dose-dependent ECG changes, cardiac conduction abnormalities, and dysrhythmias. Olanzapine (C) is an antipsychotic medication that has a relatively mild side effect profile. The main concern with olanzapine is weight gain and metabolic disturbance. Risperidone (D) is a second-generation antipsychotic that is slightly less efficacious than clozapine but has a safer side effect profile. Cardiomyopathy and myocarditis are not common side effects of risperidone treatment. Risperidone has a propensity to cause hyperprolactinemia and extrapyramidal symptoms

A 34-year-old man who works as a neurosurgeon presents to a therapist with his wife for marital counseling. His wife discusses that he has a high sense of self-importance and is hypersensitive to negative evaluation by others. She also reports that he continuously brags about his professional accomplishments and lacks empathy for others who are not as successful. Which cluster of personal disorders does this patient's diagnosis most likely belong to? A. Cluster A B. Cluster B C. Cluster C D. Cluster D

B. Cluster B Narcissistic personality disorder is one of the cluster B personality disorders, along with borderline, antisocial, and histrionic personality disorders. It is more common in men, and it typically begins in early adulthood. The core features include grandiosity (sense of high self-importance), hypersensitivity to evaluation, exaggeration of accomplishments, exploitative relationships, a lack of empathy, and a lack of consideration for others. Most individuals with narcissistic personality disorder do not have insight into their narcissism. The diagnosis of narcissistic personality disorder is made based on clinical criteria found in the fifth edition of the Diagnostic and Statistical Manual of Mental Disorders. The core clinical features must be present by early adulthood and in a variety of contexts. Psychotherapy is the recommended treatment for narcissistic personality disorder. Cluster A (A) includes schizoid, paranoid, and schizotypal personality disorders. Cluster C (C) includes dependent, avoidance, and obsessive-compulsive personality disorders. Cluster D (D) is not one of the clusters of personality disorders.

A 20 year-old male presents to the ED with complaints of palpitations and agitation, which developed suddenly while attending a party. On examination, the patient is moderately agitated and tremulous. Vital signs include a pulse of 110/minute and regular; respiratory rate 22/minute and blood pressure 160/92 mmHg. Skin is diaphoretic and pupils are dilated. Which of the following is the most likely diagnosis? A. Heroin use B. Cocaine use C. Scombroid ingestion D. Alcohol intoxication

B. Cocaine use Cocaine, as well as amphetamines, leads to a clinical picture of increased sympathetic stimulation and dilated pupils.

A 45-year-old man presents to the clinic concerned that he may have brain cancer. He reports that since a coworker was diagnosed with brain cancer 5 years ago, he has spent more than 3 hours per day reading about brain cancer treatment in anticipation of being diagnosed himself. He has had 11 magnetic resonance images performed of his brain during the past 5 years. He reports no headaches or other neurologic symptoms. Which of the following is the first-line treatment for the suspected diagnosis? A. Acceptance and commitment therapy B. Cognitive behavioral therapy C. Duloxetine D. Fluoxetine

B. Cognitive behavioral therapy Acceptance and commitment therapy (A) is appropriate as a second-line psychotherapy for patients with illness anxiety disorder. Duloxetine (C) is a serotonin and norepinephrine reuptake inhibitor that is a second-line pharmacologic agent in the treatment of illness anxiety disorder. Fluoxetine (D) is a selective serotonin reuptake inhibitor antidepressant, which is the preferred pharmacologic therapy for illness anxiety disorder. However, psychotherapy is recommended as the first-line treatment for patients with illness anxiety disorder.

A 25-year-old man presents, stating his boss told him to be evaluated because he is obsessed with perfectionism. Which of the following historical elements would be present in a patient with obsessive-compulsive personality disorder versus obsessive-compulsive disorder? A. Avoiding situations because of a fear a dog is going to bite him B. Conflicts with family and friends about his excessive perfectionism C. Inability to drive across bridges because he is afraid the bridge will fail D. Symptoms get better or worse depending on the amount of anxiety a situation causes

B. Conflicts with family and friends about his excessive perfectionism Avoiding situations because of a fear a dog is going to bite him (A) would be characteristic of obsessive-compulsive disorder. Inability to drive across bridges because he is afraid the bridge will fail (C) is characteristic of an obsessive-compulsive disorder, where actions are governed by fear of an imaginary catastrophe. Patients with obsessive personality disorder are motivated by the need to be perfect. Symptoms that get better or worse depending on the amount of anxiety a situation causes (D) is a symptom of obsessive-compulsive disorder. Patients with an obsessive-compulsive personality disorder have symptoms that are consistent over time and are not associated with anxiety.

A 24-year-old man presents to the emergency department with agitation after ingesting a substance in brownies. He reports he has inhaled this substance before and it usually gives him a relaxed and euphoric feeling. He also reports dry mouth, nausea, and vomiting. Vital signs include a heart rate of 108 bpm and blood pressure of 155/92 mm Hg. Which of the following additional findings supports the suspected diagnosis? A. Auditory hallucinations B. Conjunctival injection C. Diaphoresis D. Pinpoint pupils

B. Conjunctival injection Auditory hallucinations (A) are not a typical finding of marijuana intoxication. However, they may occur with cocaine, methamphetamine, phencyclidine, or hallucinogen intoxication; alcohol or benzodiazepine withdrawal; anticholinergic or salicylate toxicity; serotonin syndrome; neuroleptic malignant syndrome; or primary psychiatric pathology (bipolar I disorder or schizophrenia). Diaphoresis (C) is not associated classically with marijuana intoxication. It is seen in patients with methamphetamine or cocaine intoxication, serotonin syndrome, neuroleptic malignant syndrome, and alcohol or benzodiazepine withdrawal. Pinpoint pupils (D) are classically associated with opioid intoxication.

According to the National Institute on Alcohol Abuse and Alcoholism, which of the following would classify a male patient to have an increased health risk due to his alcohol consumption? A. Drinking 2 standard drinks in a single day B. Drinking more than 14 standard drinks per week C. Drinking more than 3 standard drinks in a single day D. Drinking more than 7 standard drinks per week

B. Drinking more than 14 standard drinks per week Drinking 2 drinks in a single day (A) would not lead to increased risk of health issues in both men and women under the age of 65 years old. Men would be at increased risk with 4 standard drinks in a given day, while women are at increased risk after consuming 3 standard drinks. Drinking more than 3 drinks in a single day (C) and drinking more than 7 standard drinks per week (D) are both correct classifications for risky alcohol use for women under the age of 65.

A 60-year-old man presents to the clinic due to panic attacks. He previously canceled three of his office visits and is very nervous about this encounter. He states that, for the past 2 months, he mostly stays home because he is afraid he will have a panic attack in public and be embarrassed. Which of the following situations is this patient also likely to avoid? A. Being at home with a spouse B. Driving a car C. Sharing a meal with a close friend D. Speaking to a group via a video conference

B. Driving a car Agoraphobia is fear of being in a situation where help may not be readily available or escape may not be easily attained in the event of developing panic-like, embarrassing, or incapacitating symptoms. Agoraphobia is often accompanied by panic disorder, and the patient is often fearful of having a panic attack in certain situations. This fear leads to avoidant behavior and can severely limit a patient's daily activities. Patients with agoraphobia commonly avoid driving a car, flying in a plane, waiting in line, being at home alone, being away from home, going to a doctor's or dentist's office, or going to a salon or barber. Patients also often avoid shopping malls, public transportation, restaurants, movie theaters, wide open spaces, grocery stores, and crowded spaces. The mobility inventory is a useful tool that can assess the severity of a patient's agoraphobia. A score of 1.61 or greater on the avoidance alone scale indicates the presence of agoraphobia. Treatment of agoraphobia includes cognitive behavioral therapy and pharmacotherapy for panic disorder if present. Selective serotonin reuptake inhibitors are first-line pharmacotherapy for panic disorder. Being at home with a spouse (A) is often where a patient with agoraphobia feels most comfortable. In this situation, they feel assured of help and lack of embarrassment should they have a panic attack. Sharing a meal with a close friend (C) is less likely to be a situation avoided by a patient with agoraphobia than driving, especially if the meal is at home. Speaking to a group via a video conference (D) may not bother a patient with agoraphobia because they can easily escape the situation by turning off the video feed. Public speaking may not necessarily produce fear in a patient with agoraphobia, but fear of having a panic attack during the event may be present.

A 54-year-old woman presents to the office for follow-up for her major depressive disorder. She was started on low-dose imipramine a couple of months ago, and the dose was slowly titrated at her last visit after a partial response in mood symptoms was noted. Today, she reports her mood and anhedonia are improved, but she is concerned about the development of possible side effects related to the increased dosage of her medication. What side effect would she most likely report? A. Diarrhea B. Dry mouth C. Urinary urgency D. Weight loss

B. Dry mouth Diarrhea (A) is more frequently seen as a side effect of selective serotonin reuptake inhibitors and atypical antidepressants. Constipation is more frequently reported with TCAs due to the anticholinergic effects. Urinary urgency (C) is incorrect. Patients on a TCA would more commonly report urinary retention due to the anticholinergic effects of the drug. It is because of this side effect that the drug must be cautioned in individuals with prostatic hyperplasia. Weight loss (D) is incorrect. As with many antidepressants, weight gain is more commonly reported. Weight gain is associated with the increase in appetite that occurs secondary to the antihistamine effects.

A 20-year-old man presents to the clinic with his mother. She is concerned because the patient continues to report that a famous celebrity is in love with him. Per the patient, he was at the celebrity's concert about 4 weeks ago, and although he was sitting in the upper level, the celebrity was directly singing her love song to him. The patient also notes that the celebrity was on a TV interview the other night during which she stated she was in love with him. The patient's mother is concerned because her son has not even met this celebrity in the past. Which of the following is the correct classification for this patient's delusion disorder? A. Delusion of reference B. Erotomanic delusion C. Grandiose delusion D. Jealousy delusion

B. Erotomanic delusion Delusion of reference (A) is when the patient believes a random event in life is specifically directed to them, such as the news broadcaster is speaking directly to them. Grandiose delusion (C) is when the patient has unrealistic beliefs of their abilities or powers, such as believing they are the best athlete in the world or a billionaire. Jealousy delusion (D) is when the patient suspects someone to be unfaithful when they have no reason to.

A 25-year-old woman presents to the clinic with neck and shoulder pain. She also complains of excessive worry about her children's well-being and her husband's fidelity. She states she has frequent headaches, trouble sleeping, feelings of being overwhelmed, difficulty concentrating, and increased irritability. Vital signs are within normal limits, and complete blood count, comprehensive metabolic panel, and thyroid panel are within normal limits. Which of the following would help confirm the most likely diagnosis? A. Abnormal cervical spine X-ray B. Excessive worry has caused the patient to quit her job C. Patient Health Questionnaire-9 score of 21 D. Symptoms more days than not, lasting 2 to 3 weeks

B. Excessive worry has caused the patient to quit her job Generalized anxiety disorder is the most common psychiatric disorder encountered in primary care and describes a state of excessive worry that is difficult to control and causes significant impairment in daily functioning. Predisposing factors to the development of generalized anxiety disorder include female sex, poverty, adverse life events, chronic physical or mental illness, parental loss, insufficient emotional support as a child, or a family history of psychiatric illness. Symptoms of generalized anxiety disorder include excessive worry about several areas of life, restlessness, easy fatigability, difficulty concentrating, irritability, muscle tension, sleep disturbances, headaches, or palpitations. For a diagnosis of generalized anxiety disorder to be made, symptoms must be present more days than not for 6 months or more, cause significant impairment in daily functioning, be unrelated to substance use or a different physiologic disorder, and not be better attributed to another psychiatric disorder. Excessive worry that leads to the loss of employment represents a significant impairment in occupational functioning. Treatment of generalized anxiety disorder includes cognitive behavioral therapy and medications, such as selective serotonin reuptake inhibitors, serotonin and norepinephrine reuptake inhibitors, benzodiazepines, buspirone, and pregabalin. An abnormal cervical spine X-ray (A) would point to a physical, not psychiatric, etiology for the headaches, neck pain, shoulder pain, and poor sleep. The possibility exists to have both generalized anxiety disorder and a physical disorder, but the concurrent presence of both may make the diagnosis of either one more difficult. A Patient Health Questionnaire-9 score of 21 (C) would indicate major depressive disorder may be the cause of the patient's symptoms, not generalized anxiety disorder. On the other hand, both generalized anxiety disorder and major depressive disorder can coexist and make the diagnosis of generalized anxiety disorder a bit more difficult. Symptoms more days than not, lasting 2 to 3 weeks (D) are not present long enough to diagnose a patient with generalized anxiety disorder, where symptoms must be present for 6 months or more. A patient with anxiety symptoms that occur within 3 months of an identifiable stressor can be diagnosed with adjustment disorder.

A 20-year-old woman currently taking fluoxetine 20 mg daily for her anxiety disorder reports that she is still unable to work because she has intrusive thoughts about death and germs that lead to over-frequent handwashing and checking behaviors. The addition of which of the following clinical interventions or therapeutics would be most helpful to this patient? A. Buspirone 10 mg daily B. Exposure and response prevention C. Hypnotherapy D. Risperidone 2 mg daily

B. Exposure and response prevention Buspirone 10 mg daily (A) is an anxiolytic medication that shows little more efficacy than placebo and is not indicated for the treatment of obsessive-compulsive disorder. Hypnotherapy (C) is not indicated for the treatment of obsessive-compulsive disorder and has fallen out of favor as a psychotherapeutic intervention in general. Risperidone 2 mg daily (D) is an antipsychotic medication that would not be indicated for the patient in the above vignette, as she does not suffer from psychotic symptoms.

A 72-year-old woman presents to the clinic due to sadness and crying every day since her husband's death 8 months ago. She reports loss of appetite, insomnia, and preoccupation with items that belonged to her husband. Which of the following is indicative of a complicated grief reaction? A. Confusion about one's identity apart from the deceased B. Extreme yearning for the deceased more than 6 months after their death C. Impaired attention and memory D. Increased use of tobacco and alcohol

B. Extreme yearning for the deceased more than 6 months after their death Confusion about one's identity apart from the deceased (A) is a common reaction to the loss of a loved one and constitutes an acute grief reaction. Impaired attention and memory (C) is also common in an acute grief reaction. Increased use of tobacco and alcohol (D) is an adverse consequence of a complicated grief and not indicative of a complicated grief reaction. Grief is a natural response to the loss of a loved one. Symptoms of acute grief include loneliness, crying, yearning for the deceased, social withdrawal, identity confusion, distressing physical sensations, insomnia, persistent thoughts and memories of the deceased, disbelief, shock, numbness, and impaired attention and memory. Grief does not proceed in a logical stepwise fashion but is expressed differently in each patient, depending on the closeness of the individual lost, the cultural norms surrounding the grieving process, and the baseline psychological state of the aggrieved. Although the course is varied, grief is generally a self-limited process, and patients may be expected to return to baseline functioning at 6 months after the death of the loved one. Patients who continue to have symptoms longer than 6 months after the death of a loved one may be diagnosed with a complicated grief reaction. The symptoms of complicated grief include extreme persistent yearning for the deceased, preoccupying thoughts about the deceased, intense feelings of loneliness, recurrent thoughts that it is unbearable to live without the deceased, or recurrent urge to join the deceased. Symptoms of complicated grief cause significant distress and impair psychosocial functioning. The treatment for a complicated grief reaction is cognitive behavioral therapy.

A 25-year-old man presents to the clinic complaining of having trouble concentrating at work and holding down a job. He states he has never been able to sit still, even when he was a small child. He has trouble focusing, difficulty finishing projects, and often interrupts others when they are talking. He has no known chronic health conditions and takes no medications. He reports no illicit drug or alcohol use. Which of the following is required to make the most likely diagnosis? A. A total of 10 or more symptoms of hyperactivity and inattention B. Five or more symptoms of hyperactivity C. Four or more symptoms of inattention D. Symptoms that are present for at least 1 month

B. Five or more symptoms of hyperactivity A total of 10 or more symptoms of hyperactivity and inattention (A) is not necessary for a diagnosis of attention-deficit/hyperactivity disorder in an adult. A patient may have symptoms of inattention or symptoms of hyperactivity but does not necessarily need to demonstrate both for the diagnosis. Four or more symptoms of inattention (C) are too few for a diagnosis of attention-deficit/hyperactivity disorder. Five or more are needed for diagnosis in an adult, and six or more are needed for diagnosis in a child. Symptoms that are present for at least 1 month (D) do not allow sufficient time to diagnose a chronic disorder, such as attention-deficit/hyperactivity disorder. The symptoms must be present for 6 months or more. To diagnose attention-deficit/hyperactivity disorder, a patient under 17 years old must demonstrate six or more symptoms of hyperactivity or inattentiveness in the absence of physical, psychological, or situational conditions that could better account for the symptoms. A patient 17 years or older must demonstrate five or more symptoms of hyperactivity or five or more symptoms of inattentiveness for a diagnosis of attention-deficit/hyperactivity disorder. In addition to the number of symptoms, the patient must also demonstrate symptoms that are present in more than one setting, that are persistent for at least 6 months, that occur often, that presented before the age of 12 years, that impair function, and that are excessive for the developmental level of the child (if the patient is under 17 years). The World Health Organization has a self-assessment form for adults called the adult ADHD self-report scale that can be used to screen for attention-deficit/hyperactivity in adults. The self-report form includes questions regarding organizational difficulties, procrastination, fidgeting, and overactivity. Treatment for attention-deficit/hyperactivity disorder involves behavioral therapy and medications such as stimulants or alpha-2 adrenergic agonists. Pharmacologic treatment for attention-deficit/hyperactivity disorder should begin after the preschool years and may need to be lifelong, although symptoms sometimes improve with age. Pharmacologic treatment for adults involves the same type of medications as those used for children. What percentage of affected children have symptoms of attention-deficit/hyperactivity disorder that persist into adulthood? -60%

A 24-year-old woman presents to the clinic at the request of her family. Her mother reports that she has caught her daughter purging after several meals recently. The patient reports she sometimes eats two large pizzas in one sitting, purges, and then later feels guilty. She says she is highly concerned with maintaining her body weight because she is a gymnast and cannot gain weight. Her body mass index is 25 kg/m2. Which of the following is the first-line pharmacologic treatment for the suspected diagnosis? A. Bupropion B. Fluoxetine C. Olanzapine D. Sertraline

B. Fluoxetine Bupropion (A) is contraindicated in patients with current or prior eating disorders because it increases the risk of seizures in these patients. Olanzapine (C) is a second-generation antipsychotic and is the first-line medication used in patients with anorexia nervosa who are treated with pharmacologic therapy. It is not used in the treatment of bulimia nervosa. Sertraline (D) is a selective serotonin reuptake inhibitor and is considered a second-line agent for bulimia nervosa. Fluoxetine is the only agent approved by the Food and Drug Administration for bulimia nervosa and has the most studies supporting its efficacy. Common laboratory findings associated with bulimia nervosa include hypochloremia, hypokalemia, and metabolic alkalosis. The diagnosis of bulimia nervosa is made according to the criteria in the fifth edition of the Diagnostic and Statistical Manual of Mental Disorders. The recommended treatment for bulimia nervosa includes a combination of pharmacotherapy, psychotherapy, and nutritional rehabilitation. However, psychotherapy alone is more effective than pharmacotherapy alone, and cognitive behavioral therapy is the preferred psychotherapy. Pharmacotherapy alone has proven efficacy in patients who do not have access to or wish to avoid psychotherapy and nutritional rehabilitation. The preferred pharmacotherapy is fluoxetine, which is a selective serotonin reuptake inhibitor. This agent has been studied the most in clinical trials. Second-line agents include other selective serotonin reuptake inhibitors, such as sertraline. Patients who improve with pharmacotherapy should be kept on maintenance therapy for at least 6 to 12 months to prevent relapse.

A 20-year-old woman presents to the clinic with anhedonia for 1 month. She reports the activities she usually enjoys are not fun anymore. She also reports hypersomnia, fatigue, weight gain, and difficulty concentrating. You decide to start her on a medication that is in the same class as sertraline. Which of the following medications from this class has the longest half-life? A. Escitalopram B. Fluoxetine C. Fluvoxamine D. Paroxetine

B. Fluoxetine Major depressive disorder (unipolar depression) is a psychiatric disorder characterized by depressed mood and anhedonia. The etiology has genetic and environmental components. The risk factors include younger age, prior depressive episode, family history, female sex, childbirth, childhood trauma, stressful life events, lower income, substance use, serious medical illness, and poor social support. The diagnosis of an episode of major depression requires an episode of depressed mood or loss of interest for at least 2 consecutive weeks and includes at least five of the following depressive symptoms: depressed mood, anhedonia, unintentional weight changes, hypersomnia or insomnia, psychomotor agitation or retardation, fatigue, inappropriate guilt, difficulty concentrating, and suicidal ideation. The treatment options for depression include pharmacologic therapy and psychotherapy. The combination of pharmacologic therapy and psychotherapy is better than either alone. Selective serotonin reuptake inhibitors are considered the first-line class of antidepressants due to tolerability and safety in overdose. Examples of serotonin selective reuptake inhibitors include fluoxetine, sertraline, paroxetine, citalopram, and escitalopram. Serotonin and norepinephrine reuptake inhibitors, atypical antidepressants, and monoamine oxidase inhibitors are reasonable alternatives. Common side effects of selective serotonin reuptake inhibitors include sexual dysfunction, gastrointestinal side effects (nausea and diarrhea), insomnia, somnolence, and weight gain. Fluoxetine has the longest half-life, between 1 and 3 days. The half-life of the other selective serotonin reuptake inhibitors ranges from 20-30 hours. Patients often begin to notice improvement within 2 weeks of starting an antidepressant, but the maximal effect takes 6-12 weeks. Patients who do not respond to pharmacologic monotherapy can be treated with a combination of pharmacologic agents. The most common second pharmacologic agents used include second-generation antipsychotics (quetiapine and risperidone), lithium, an antidepressant from another class, and thyroid hormone. Electroconvulsive therapy can be used in patients with severe suicidal ideation or malnutrition. Escitalopram (A), fluvoxamine (C), and paroxetine (D) do not have as long of a half-life as fluoxetine.

A 62-year-old man presents with concerns over having "too much stress." He describes feeling worried most days about family, friends, finances, and health concerns. He notes feeling restless and unable to relax. He also reports he is having difficulty sleeping and thinks that is leading to difficulty concentrating at work. These symptoms have been present for the last 8 months. What is the most likely diagnosis? A. Adjustment disorder B. Generalized anxiety disorder C. Panic disorder D. Separation anxiety disorder

B. Generalized anxiety disorder Adjustment disorder (A) occurs within 3 months of an event or identifiable stressor and includes anxiety and depression over the stressor. Panic disorder (C) includes panic attacks that occur suddenly and unexpectedly. Separation anxiety disorder (D) is centered on the fear of losing close family or friends. What is the instrument used to measure the severity of generalized anxiety disorder? -The hospital anxiety and depression scale. Generalized Anxiety Disorder (GAD) Excessive worry Persistent symptoms Difficult to control feelings Symptoms last over 6 months Treat with CBT, SSRI

An 8-year-old boy presents to the clinic with his parents, who have behavioral concerns. At a recent parent-teacher conference, the teacher brought up concerns of inattention, emotional outbursts, and general restlessness in class. The parents confirm these behaviors have been disruptive at home as well. They report the patient gets frustrated easily and has difficulty handling his emotions. Which of the following predisposes the patient to the most likely diagnosis? A. Excessive dietary iron and zinc B. Genetic factors C. Higher birth weight D. Prenatal exposure to salicylates

B. Genetic factors Excessive dietary iron and zinc (A) is not thought to be a factor in developing ADHD. Dietary iron and zinc deficiency is thought to contribute to the development of ADHD. Lower birth weight and prematurity, not higher birth weight (C), are associated with ADHD. Prenatal exposure to salicylates (D) is not associated with ADHD, but prenatal exposure to acetaminophen and antidepressants is being investigated as a possible factor in developing this condition.

A 48-year-old man presents to the clinic for a rash. He states he has had poor personal hygiene for as long as he can remember. He likes to wear his shirts until they fall apart then keep a piece of them in his pocket when he picks out a new shirt. He believes his Magic 8-ball gives him good advice every morning. He has been a factory worker for the past 10 years and is often faulted for his disorganization and lack of attention to detail. He reports no auditory or visual hallucinations. He states he has always been too nervous, distrustful, and shy to get married. Which of the following pharmacologic interventions may be beneficial for this patient's personality disorder? A. Fluoxetine B. Guanfacine C. Haloperidol D. Mirtazapine

B. Guanfacine Pharmacological treatment of schizotypal personality disorder is primarily aimed at disordered thinking and inattention. Guanfacine (alpha-2A adrenergic agonist) can help patients with schizotypal personality disorder gain cognitive organization while also decreasing anxiety. Patients may also be treated with stimulant medications, like those used to treat attention-deficit disorder, although these may increase anxiety levels and should be monitored closely. Fluoxetine (A) is a selective serotonin reuptake inhibitor. This class of medications has not shown efficacy in the treatment of anxiety associated with schizotypal personality disorder. Haloperidol (C) is an antipsychotic medication that is used for patients who are suffering psychosis or who have schizophrenia. Haloperidol is not recommended for the treatment of schizotypal personality disorder, as antipsychotics do not appear to ameliorate cognitive symptoms in these patients. Mirtazapine (D) is an atypical antidepressant that is not recommended therapy for a patient with schizotypal personality disorder.

A 32-year-old man with a medical history of uncontrolled hypertension and chronic kidney disease is brought to the emergency department in police custody after he was found digging for buried treasure in his neighbor's yard in the middle of the night. He reports he has not slept for days, and he heard voices from God telling him there was treasure in his neighbor's yard. You notice during your assessment he abruptly changes topics and his speech is pressured. He has a negative urine drug screen. The patient reports he has had prior episodes of depression but is currently not taking any psychiatric medications. Your preceptor tells you the best regimen for him includes olanzapine and one other medication. Which of the following is the black box warning for the additional recommended medication for this patient? A. Agranulocytosis B. Hepatotoxicity C. Stevens-Johnson syndrome D. Weight

B. Hepatotoxicity Agranulocytosis (A) is classically associated with the atypical antipsychotic clozapine. Agranulocytosis refers to a reduction in specific types of white blood cells, including neutrophils, eosinophils, and basophils. Stevens-Johnson syndrome (C) is a life-threatening rash that classically involves mucosal surfaces and is associated with the mood-stabilizing (antimanic) medication lamotrigine. Weight gain (D) is one of the adverse effects of valproate, but it is not a black box warning. The recommended initial treatment for patients with severe mania includes lithium and an antipsychotic or valproate and an antipsychotic if lithium is contraindicated. Valproate is the recommended antimanic (mood-stabilizing) medication in this patient because he has a history of chronic kidney disease, which is a contraindication for lithium. The adverse effects of valproate include weight gain, nausea and vomiting, hair loss, and easy bruising. Hepatotoxicity, thrombocytopenia, and pancreatitis are rare adverse effects that may occur. Valproate has a black box warning regarding the potential for hepatotoxicity and pancreatitis. It is contraindicated in women who could possibly become pregnant due to the possibility of teratogenicity. It is important to monitor liver enzymes (aspartate aminotransferase and alanine aminotransferase) and platelets at least every 6-12 months in patients who are taking valproate.

A 26-year-old woman presents for therapy after being arrested for public nudity. Her family reports she has a persistent pattern of attention-seeking behavior and excessive emotionality. Which of the following personality disorders does this patient most likely have? A. Borderline B. Histrionic C. Narcissistic D. Schizotypal

B. Histrionic Borderline (A) personality disorder is characterized by instability of self-image and interpersonal relationships and is marked by impulsivity, increased risk of self-injury or suicide, and fear of abandonment. While both histrionic and borderline personality disorders manifest with difficulty with interpersonal relationships, sexually seductive behavior and attention-seeking are more defining features of histrionic personality disorder. Narcissistic (C) personality disorder is characterized by a need for admiration and a pattern of grandiosity. Individuals with narcissistic personality disorder often require excessive admiration, have a sense of entitlement, and are envious of others. Schizotypal (D) personality disorder is characterized by eccentric behavior, cognitive or perceptual distortions, and discomfort with close relationships. This condition has similar but less severe features than schizophrenia.

A 28-year-old man presents to the psychiatrist for treatment of his anxiety. He reports that he is suspicious his wife is having an affair because she goes to the gym three times per week. He states that he has had trouble developing friendships because he cannot trust people due to suspicion that they are trying to exploit him. Which of the following features is consistent with the suspected diagnosis? A. Cognitive-perceptual disturbances B. Holding grudges C. Hypersensitivity to negative evaluation by others D. Lack of desire for interpersonal relationships

B. Holding grudges Cognitive-perceptual disturbances (A) are a core feature of schizotypal personality disorder. The other core features include oddness and few close interpersonal relationships due to social anxiety. The cognitive-perceptual disturbances do not reach the level of delusions or hallucinations seen with schizophrenia. Hypersensitivity to negative evaluation by others (C) is an important feature of dependent personality disorder and narcissistic personality disorder. Lack of desire for interpersonal relationships (D) is a core feature of schizoid personality disorder.

A 28-year-old woman presents with a pervasive behavior pattern that includes cognitive-perceptual problems and interpersonal dysfunction. Among her symptoms is the belief she can see auras around others and she can read minds. Which of the following additional symptoms is most likely associated with the diagnosis? A. Auditory hallucinations B. Ideas of reference C. Impulsivity D. Tangential speech

B. Ideas of reference Schizotypal disorder is characterized by a long-standing and pervasive behavior pattern that includes cognitive-perceptual problems and interpersonal dysfunction associated with significant disability. It is a rare psychiatric disturbance that often co-occurs with borderline personality disorder, bipolar disorder, or panic disorder with agoraphobia. Clinical manifestations include three subgroups: cognitive-perceptual, oddness or disorganized, and interpersonal. The cognitive-perceptual symptoms are chronic distortions that include odd beliefs, unusual perceptual experiences, ideas of reference, and paranoia. Odd beliefs or magical thinking may include mind reading or thought transfer. Unusual perceptual experiences include feelings of body distortion or the thought of seeing a halo or aura around others. Ideas of reference are defined as a belief that events are occurring in direct relationship to a certain individual. For example, the belief a person on a television show is talking directly about the patient who is watching. Oddness is usually observed in these patients, who exhibit eccentric or unconventional behaviors. Interpersonal manifestations include social anxiety and an inability to develop close relationships. Auditory hallucinations (A) are often associated with schizophrenia but are not part of the diagnostic criteria for schizotypal personality disorder. Impulsivity (C) is associated with attention-deficit/hyperactivity disorder. Tangential speech (D), or derailment, is associated with schizophrenia.

A 24-year-old man presents to the clinic worried he has lung cancer. He reports his father died of lung cancer and one of his colleagues was just diagnosed with lung cancer. He has no reported symptoms other than an occasional nonproductive cough, he has no significant medical history and reports no use of any medications, supplements, or illicit substances. He does not use tobacco products. He has normal vital signs, a normal physical exam, and no known risk factors. Chart review reveals similar presentations over the past 8 months with a negative chest X-ray. Which of the following is the most likely diagnosis? A. Factitious disorder B. Illness anxiety disorder C. Somatic symptom disorder D. Specific phobia

B. Illness anxiety disorder Patients with factitious disorder (A) have external motivation behind their health concerns, which is absent in the vignette. Somatic symptom disorder (C) is similar to illness anxiety disorder, but somatic symptoms predominate and drive health fears in this condition, whereas patients with illness anxiety disorder have minimal or no somatic symptoms. Specific phobia (D) involves fear of a specified object or situation (e.g., injections, flying, animals), while the fears related to illness anxiety disorder center around having or acquiring a medical disease. Illness Anxiety Disorder Hypochondriasis Preoccupied with serious illness, despite negative exam or testing Evaluate for other medical diagnosis Management is CBT, psychotherapy, antidepressants

A 21-year-old man presents to the clinic due to recurrent nausea, vomiting, and abdominal pain. He says that, in the last 6 months, he has unintentionally lost 20 pounds. Physical exam findings are unremarkable other than mild abdominal tenderness with palpation. Urine drug screen is positive for tetrahydrocannabinol. Which of the following is most likely to confirm the diagnosis? A. Abdominal pain in the right lower quadrant B. Improvement in symptoms with hot showers C. Improvement in symptoms with ondansetron D. No improvement in symptoms with intravenous fluids

B. Improvement in symptoms with hot showers Cannabis is a drug that produces both psychoactive and antiemetic effects. Medical marijuana is now legal in many states and is sometimes used to treat nausea and vomiting due to chemotherapy. When used chronically, the overstimulation of cannabis receptors causes a paradoxical effect, resulting in recurrent nausea, vomiting, and abdominal pain. Cannabinoid hyperemesis syndrome is a type of cyclical vomiting syndrome. It occurs most commonly in young men and patients who have a history of chronic or daily marijuana use. Symptoms start in the morning and may be accompanied by bloating, diaphoresis, or weight loss. Compulsive hot showers help relieve the symptoms, and a positive history of this helps to confirm the diagnosis. Diagnosis is clinical and is made after ruling out more serious medical conditions, which can mimic the symptoms of cannabinoid hyperemesis syndrome, including pancreatitis, intestinal obstruction, acute gastroenteritis, and other disorders of the bowel. Complications of the syndrome include Mallory-Weiss tears, acute kidney injury, hypovolemia, and rhabdomyolysis. Initial treatment involves rehydration and bowel rest. Haloperidol may help to alleviate symptoms, although it should not be used in patients who are pregnant. Cannabinoid hyperemesis syndrome is ultimately cured with cessation of marijuana use. Patients may need a referral to a substance use treatment program or 12-step program to help with cessation efforts and recovery.

A 42-year-old woman with a history of bipolar disorder presents with multiple concerns. She reports that she has not been taking her medication since she moved a few months ago and feels that her symptoms may be worse. While she is talking, you notice that her speech is pressured, fast, and loud. She speaks with descriptive sounds and dramatic gestures. Which of the following would you expect when taking a history? A. Feelings of hopelessness B. Increased risk-taking behaviors C. Rumination D. Somatic symptoms

B. Increased risk-taking behaviors Bipolar I disorder is a psychiatric disorder that is characterized by manic episodes, hypomanic episodes, and major depressive episodes. Patients can experience a range of severity of these symptoms. Manic episodes are characterized by abnormally elevated and labile moods. There can be disinhibition present, which includes a disregard for social boundaries and a need for hyperstimulation. Increased energy, talkativeness, planning behaviors, impulsivity, and risk-taking are core characteristics of mania. Manic episodes are often associated with poor judgment and reckless behaviors, such as high-risk sexual encounters, infidelity, or spending sprees. Increased mental activity, flight of ideas, irrelevant thoughts, and increased mental activity are associated with mania. Manic speech is pressured, loud, and associated with sounds and dramatic gestures. Mania typically has a sudden onset and gets worse over the course of a few days. Episodes of mania can last from weeks to months and often resolve completely. First-line treatment of bipolar disorder often includes quetiapine or lurasidone, lithium may also be used in severe mania. Medications are chosen based on the patient and the side effect profiles. Feelings of hopelessness (A) accompany episodes of major depression that are associated with bipolar disorder. These are often not present during manic episodes. Rumination (C) and somatic symptoms (D), such as unexplained and poorly differentiated pain, are also associated with major depression and are not present during manic episodes.

A 22-year-old woman presents to the clinic due to episodes of irritable mood, increased energy, and increased talkativeness that last for 2 days and are then followed by 2 days of low energy with hypersomnolence and lack of interest in activities. The episodes cause the patient distress and have been occurring for more than 2 years on most days of the year. Which of the following, if present, would indicate a diagnosis of cyclothymic disorder? A. Auditory hallucinations B. Inflated self-esteem C. No impairment in social functioning D. Suicidal ideation

B. Inflated self-esteem Cyclothymic disorder is a mood disorder characterized by mood alterations that swing between increased energy and mood and decreased energy and mood. The episodes of increased energy and irritable or expansive mood do not qualify as hypomanic or manic episodes, do not last longer than 4 days, and do not require hospitalization, but they do cause the patient distress. The episodes of low energy and depressed mood also do not meet the criteria for major depression, do not last longer than 2 weeks, and do not necessitate hospitalization. These mood disturbances should be present for at least 2 consecutive years and occur more days than not for a diagnosis of cyclothymic disorder. Symptoms of irritable or expansive mood in cyclothymia can include increased goal-directed behavior, decreased need for sleep, inflated self-esteem, talkativeness, and distractibility. Symptoms of depressed mood can include lack of interest in normal activities, increased or decreased sleep, increased or decreased appetite, poor self-esteem, guilt, and sadness. Treatment of cyclothymic disorder includes cognitive behavioral therapy and the use of low-dose mood stabilizers with the goal of achieving periods of 6 months of relative mood stability at a time. Auditory hallucinations (A) are considered a psychotic feature. Patients with expansive mood, increased energy, and increased talkativeness who also complain of auditory hallucinations meet the criteria for mania, not for cyclothymic disorder. No impairment in social functioning (C) is incorrect, one of the criteria includes symptoms cause distress or significant impairment in social or occupational functioning. Suicidal ideation (D) is a criterion for major depressive disorder and may also require hospitalization. Patients with cyclothymic disorder do not require hospitalization and do not meet the criteria for major depression.

A 28-year-old woman presents to the clinic with 6 or 7 nights of feeling a decreased need to sleep, racing thoughts, irritability, and an elevated mood lasting all day for the past week. She states her roommates are aware of the change in her behavior, but she is able to go to work each day and perform her duties normally. She reports no auditory or visual hallucinations, thoughts of persecution, or paranoia. Her medical history includes one episode of major depression 12 months ago that lasted 3 months and went unmedicated. Which of the following treatment options should be avoided in a patient who is hospitalized for mania but would be an appropriate choice for this patient? A. Carbamazepine B. Lamotrigine C. Lithium D. Olanzapine

B. Lamotrigine Carbamazepine (A) is an antiepileptic drug that inactivates voltage-gated sodium ion channels and has proven useful in stabilizing patients with acute mania. Carbamazepine is not a first-line maintenance therapy for bipolar I or bipolar II disorder. Other antiepileptic drugs, such as divalproex and oxcarbazepine, have shown greater efficacy in maintenance therapy. Lithium (C) is a mood stabilizer that works by down-regulating the N-methyl-D-aspartate receptor, thus modulating the release of the excitatory neurotransmitter glutamate. Lithium is useful in both acute mania and in maintenance therapy for bipolar I and bipolar II disorders. Olanzapine (D) is an antipsychotic medication that antagonizes dopamine and is useful in the treatment of acute manic episodes. Olanzapine is used as a second- or third-line agent in the management of chronic bipolar I and bipolar II disorders.

An 18-year-old woman presents to the clinic due to right foot pain. She states she is on the cross-country team and has been running an additional 5 miles per day. Her body mass index is 16 kg/m2. She reports she has been limiting herself to only vegetables because she "feels fat." The X-ray of her right foot reveals multiple stress fractures. Which of the following additional abnormalities is most likely to be observed in this patient? A. Hyperkalemia B. Leukopenia C. Short QT interval D. Thrombocytosis

B. Leukopenia Hyperkalemia (A) is not seen in anorexia nervosa. Hypokalemia can be seen in anorexia nervosa, especially in the binge eating and purging subtype. Short QT interval (C) is not seen in anorexia nervosa, rather patients with anorexia nervosa may experience a prolonged QT interval. The QT interval is defined as the point of ventricular depolarization to ventricular repolarization. The slower the heart rate, the longer the QT interval. Hypomagnesemia, hypocalcemia, and hypokalemia can also prolong the QT interval, which can occur in anorexia nervosa patients. Anorexia nervosa patients are also at an increased risk for developing reduced cardiac chamber volumes, mitral valve prolapse, bradycardia, hypotension, and decreased cardiac mass. Because of bone marrow atrophy due to malnutrition, thrombocytopenia, not thrombocytosis (D), can be seen in severe anorexia nervosa and can cause bleeding complications.

A 28-year-old man presents to the clinic reporting episodes of both elevated and depressed moods over the past few years. He reports that some days he goes without sleeping and continues to have excessive thoughts and ideas that race in his mind, while other days, he is stuck in bed with little to no desire to do anything. Which of the following is the most appropriate treatment for this patient? A. Fluoxetine B. Lithium C. Sertraline D. Venlafaxine

B. Lithium Fluoxetine (A), sertraline (C), and venlafaxine (D) are all antidepressants that are commonly used to treat patients with major depressive disorder. These patients will have anhedonia in addition to five or more other associated depressive symptoms, such as fatigue, insomnia, feelings of guilt, thoughts of death or suicide, psychomotor agitation, weight changes, changes in appetite, or changes in concentration. The presentation in the patient in the above vignette does not align with major depressive disorder and thus antidepressant medication is not warranted.

A 21-year-old woman presents to the emergency department saying she feels as if she will die soon. She is sweating profusely, is tachypneic, and has tachycardia. All other vital signs are within normal limits. Radiologic and laboratory studies reveal no cardiopulmonary disease. The patient states that she tends to have feelings of impending doom when she is in crowded areas like she was earlier today. Which of the following clinical interventions would be most appropriate treatment of this patient's acute symptoms? A. Buspirone 15 mg B. Lorazepam 1 mg C. Mirtazapine 15 mg D. Sertraline 20 mg

B. Lorazepam 1 mg Buspirone 15 mg (A) is a 5-hydroxytryptamine receptor 1A antagonist that can be used to treat generalized anxiety disorder but generally shows no greater efficacy than placebo in panic disorder and panic attack. Mirtazapine 15 mg (C) is an atypical antidepressant with an indication for use in major depressive disorder. Mirtazapine can be used off-label as second-line therapy in panic disorder among patients who do not respond to or cannot take a selective serotonin reuptake inhibitor. Mirtazapine, like other antidepressants, does not work to immediately relieve symptoms of a panic attack but rather works to decrease the frequency and severity of panic attacks. Sertraline 20 mg (D) is a selective serotonin reuptake inhibitor and is used as first-line treatment for panic disorder but is not effective in immediately suppressing symptoms of an acute panic attack. True or false: exercise is effective at decreasing the frequency and severity of panic attacks. -FALSE

A 42-year-old man with a medical history for type 2 diabetes controlled with metformin presents with erectile dysfunction. He reports that he is sometimes unable to acquire an erection and there are times when he cannot sustain an erection during intercourse. He is otherwise feeling well and reports no decrease in energy or libido. Which of the following is the most likely cause of his diagnosis? A. Decreased blood flow through the hypogastric artery B. Low intracavernosal nitric oxide levels C. Metformin use D. Testosterone deficiency

B. Low intracavernosal nitric oxide levels Normal penile erections require vascular flow into the corpora cavernosa and the corpus spongiosum while venous outflow is reduced. Intrapenile nitric oxide levels rise and act as a local neurotransmitter that relaxes the intracavernosal trabeculae, promotes generation of cyclic guanosine monophosphate, and leads to vasodilation. Intracavernosal nitric oxide levels are lower in smokers, patients with diabetes, and those with testosterone deficiency. These lower levels of nitric oxide are associated with loss of libido and cause a higher frequency of erectile dysfunction in these patient populations. In addition to diabetes, chronic diseases, such as chronic kidney disease and cardiovascular disease, are also associated with the development of sexual dysfunction in males. Risk factors for developing sexual dysfunction include obesity, depression, alcohol use disorder, relationship problems, and certain medications. Medications that cause low libido and sexual dysfunction include selective serotonin reuptake inhibitors (SSRIs), antiandrogens, 5-alpha reductase inhibitors, thiazide diuretics, spironolactone, and opioid analgesics. Cardiovascular disease, particularly atherosclerosis, can cause decreased blood flow through the hypogastric artery (A). Patients without a clear cause for erectile dysfunction should be screened for cardiovascular disease. Metformin use (C) is not associated with male sexual dysfunction. Antihypertensives and antidepressant medications are the most common drugs associated with the development of erectile dysfunction in men. Testosterone deficiency (D) can cause male sexual dysfunction, but men with testosterone deficiency typically report decreased energy and loss of libido. What exercise has been shown to increase the risk of erectile dysfunction? -Biking

A 19-year-old woman presents to the clinic complaining of headache and acid reflux. She states she has no chronic illnesses and takes no medications, but she reports eating large quantities of food in discrete time periods five times weekly. These episodes are accompanied by feelings of loss of control, and she subsequently forces herself to vomit. This behavior has been present for the last 3 months. Which of the following diagnostic abnormalities might be expected in this patient? A. Hypernatremia B. Metabolic alkalosis C. Narrowing of the QRS complex on ECG D. Peaked T waves on ECG

B. Metabolic alkalosis Hypernatremia (A) is not commonly seen in bulimia nervosa. Hyponatremia, hypophosphatemia, and hypomagnesemia may be noted. Narrowing of the QRS complex on ECG (C) is not a common finding in bulimia nervosa, but a prolonged Q-T interval, QRS widening, and supraventricular ectopic rhythms may be noted. Peaked T waves on ECG (D) can be seen in hyperkalemia, which is not common in bulimia nervosa. Patients with bulimia nervosa may have hypokalemia and subsequent flattening or inversion of T waves, or in more severe cases, Q-T prolongation and Torsades de pointes. Bulimia Nervosa DSM-5: recurrent episodes of binge eating followed by inappropriate compensatory behavior via self-induced vomiting, laxative misuse, excessive exercise, or caloric restriction (occurring at least once per week for 3 months) Sense of lack of control during eating episodes Self-evaluation is unduly influenced by body shape or weight PE: body weight usually within or above normal range, dental erosions, parotid gland swelling, callused knuckles Tx options: cognitive behavioral therapy, fluoxetine or other SSRIs, or combined CBT/pharmacotherapy

A 13-year-old boy presents with behavior issues that have been worsening over the past year. According to his school and his parents, he exhibits a persistent pattern of disruptive and aggressive behavior. He has been violent toward the family pet and has destroyed household belongings. His parents recently found out he has been stealing objects from other students' lockers at school. He has also been using both marijuana and opiates. Which of the following would be the best clinical intervention when considering the most likely diagnosis? A. Family therapy B. Multisystemic treatment C. Parent training D. Social skills training

B. Multisystemic treatment Family therapy (A), parent training (C), and social skills training (D) are all interventions used within a multisystemic approach, but these are more effective alone when treating oppositional defiant disorder, while a multisystemic approach has been shown to be more effective in treating conduct disorder. Oppositional defiant disorder is characterized by less aggressiveness, and symptoms are typically reactive and less severe in nature.

A 21-year-old woman presents to the office due to anxiety. She says that, to relax enough to go to sleep at night, she spends 2 hours going through her entire house to make sure all the appliances are unplugged and there are no fire hazards in each room. She has to spend exactly 20 minutes in each room before she can move on to the next one and repeats the words "no fire" to herself throughout this process. These behaviors have been going on for 2 years and are causing the patient a great deal of distress. Which of the following is the most likely explanation for these findings? A. Generalized anxiety disorder B. Obsessive-compulsive disorder C. Obsessive-compulsive personality disorder D. Schizophrenia

B. Obsessive-compulsive disorder While individuals with obsessive-compulsive disorder experience a great deal of anxiety related to their symptoms, the recurrent thoughts that occur in generalized anxiety disorder (A) are typically related to real-life concerns, while the obsessive thoughts in obsessive-compulsive disorder are not. Concerns found in obsessive-compulsive disorder are generally irrational or odd and are linked to compulsive behaviors. Obsessive-compulsive personality disorder (C) is characterized by a pattern of excessive perfectionism and strict control that sometimes leads to ritualized-type behavior. Obsessive thoughts do not occur with this disorder, and the repetitive behaviors are not done relative to the perfectionistic thoughts. Schizophrenia (D) is a psychotic disorder with symptoms of hallucinations, delusions, and disorganized thoughts. Patients with obsessive-compulsive disorder may have obsessive thoughts that appear delusional, but they do not experience hallucinations and disorganized thinking, which are the hallmarks of schizophrenia.

A 53-year-old man with a history of schizophrenia was recently transitioned to a new antipsychotic drug after becoming refractory to several other antipsychotics. At his follow-up visit, he has gained 10 pounds. Which antipsychotic most commonly causes weight gain? A. Haloperidol B. Olanzapine C. Prochlorperazine D. Ziprasidone

B. Olanzapine Haloperidol (A) is a first-generation antipsychotic that can cause EPS, such as acute dystonic reactions, tardive dyskinesia, and akathisia. Many antipsychotics can also cause cardiac toxicity. Prochlorperazine (C), also a first-generation antipsychotic, causes agranulocytosis in 1-2% of patients treated. This side effect is most commonly seen between 6 and 18 weeks of treatment and is reversible with discontinuation of the drug. Ziprasidone (D), a second-generation antipsychotic, carries a high risk of QT prolongation, which can lead to life-threatening dysrhythmias. Careful consideration should be taken to not combine it with other QT-prolonging medications. Antipsychotic medications are the first-line medication in the treatment of schizophrenia. Antipsychotic medications can be broken down into two categories: first-generation, or typical, antipsychotics and second-generation, or atypical, antipsychotics. First-generation antipsychotic medications include haloperidol and chlorpromazine. Second-generation antipsychotic medications include olanzapine and risperidone. First-generation medications are used less often than second-generation medications due to the high rate of extrapyramidal symptoms (EPS) they can cause. While second-generation antipsychotics have fewer EPS, they may lead to weight gain, causing metabolic syndrome, cardiac side effects (including QTc prolongation), hyperprolactinemia, or drowsiness. Typical antipsychotics cause more side effects in general, which often cause patients to stop taking them. Side effects differ between medications and may depend on the dose. Weight gain can be a common side effect seen with the use of olanzapine. When a patient is on olanzapine, food intake, hyperglycemia, and hyperlipidemia should be monitored by obtaining fasting blood glucose and lipids every 3-6 months and obtaining a weight at each visit. Metformin has been shown to be effective in treating patients struggling with weight who are on antipsychotic medications.

A 25-year-old man presents to marital counseling with his wife, who reports that he emotionally abuses her. His wife also reports that he is unable to keep a job and has been fired three times in the past year. The man reports that, as an adolescent, he was sent to juvenile detention once for setting the school on fire and dropped out of high school in ninth grade. Which of the following additional findings would most support the suspected diagnosis? A. Excessive need for admiration B. Pathological lying C. Relationships beginning with intense attachment and ending with slight conflict D. Use of physical appearance to draw attention

B. Pathological lying Excessive need for admiration (A) is a core feature of narcissistic personality disorder, which is a cluster B personality disorder. Relationships beginning with intense attachment and ending with slight conflict (C) is a core feature of borderline personality disorder, which is a cluster B personality disorder. Use of physical appearance to draw attention (D) is a core feature of histrionic personality disorder, which is a cluster B personality disorder.

A 25-year-old woman with a history of substance use disorder presents to the office for follow-up evaluation. She was started on sertraline and titrated to a dose of 200 mg daily after struggling with symptoms of excessive worry, fatigue, and insomnia. She has tolerated the medication at this dosage for the past 8 weeks but has only noted partial improvement in her anxiety symptoms. What would be an appropriate intervention at this time? A. Continue sertraline for 6 months and then reassess B. Start adjunctive treatment with buspirone C. Supplement sertraline with lorazepam up to three times daily as needed D. Taper off sertraline and change to fluoxetine

B. Start adjunctive treatment with buspirone Continue sertraline for 6 months and then reassess (A) is incorrect. A timeline of 4-6 weeks is appropriate to determine the therapeutic response to a serotonergic reuptake inhibitor dosage. The patient in the vignette was started on sertraline earlier in the year and had been on the maximum dosage of 200 mg for 8 weeks at the time of the visit. Evaluating for an additional 6 months is unlikely to yield a different result. Supplement sertraline with lorazepam up to three times daily as needed (C) is incorrect. Benzodiazepines can be used as adjunctive therapy in patients with a partial response to a first- and second-line medication. This patient has not tried a second-line medication (buspirone, pregabalin) in conjunction with sertraline. Taper off sertraline and change to fluoxetine (D) is incorrect. Since the patient had a partial response to sertraline, it is recommended to adjust the response with the addition of a second-line drug, such as buspirone or pregabalin. If the patient did not have any response to sertraline, then tapering off the medication and transitioning to an alternative serotonin reuptake inhibitor would be an appropriate option.

Phenothiazines exert their antipsychotic effects by blocking A. alpha-adrenergic receptors. B. dopamine receptors. C. histamine receptors. D. serotonin receptors.

B. dopamine receptors. Blockage of the dopamine receptors in the mesolimbic areas of the brain is responsible for the antipsychotic effects of the phenothiazines; blockage of the dopamine receptors in the nigrostriatal areas lead to the motor side effects, such as dystonia and akathisia.

Which of the following laboratory abnormalities is most commonly noted in bulimia nervosa? A. glycosuria B. hypokalemia C. metabolic acidosis D. hyperalbuminemia

B. hypokalemia Episodes of binge eating are followed by purging in the bulimic patient. Vomiting and laxative abuse are the most common methods of purging, leading to hypokalemia.

A 36 year-old woman admits that her husband has abused her for over ten years. You should inform the woman that she is at most risk for injury or death A. just before a holiday. B. just after leaving an abusive spouse. C. when an abusive spouse arrives home after work. D. when an abusive spouse has been drinking heavily.

B. just after leaving an abusive spouse. Women are more likely to be assaulted or murdered when attempting to report the abuse or leave the abusive relationship; up to 75% of domestic assaults occur after separation

What drug is commonly used in the treatment of dystonic reactions other than diphenhydramine?

Benztropine.

What is the mechanism of action of methylphenidate?

Blocks reuptake of dopamine and norepinephrine.

Which antidepressant is contraindicated in patients with eating disorders because it increases the risk of seizures?

Bupropion

A 24-year-old man presents to the emergency department with intractable vomiting. He reports he has had similar episodes previously, and his symptoms seem to be relieved by hot showers. How long is the substance suspected to be causing this patient's symptoms typically detected in the urine? A. 14 days B. 2 days C. 30 days D. 60 days

C. 30 days Cannabis (marijuana) is a plant that is often used as a recreational or medicinal drug. Its active ingredient is tetrahydrocannabinol. Acute cannabis intoxication manifests with conjunctival injection, dry mouth, increased appetite, and infrequently, neurologic symptoms, such as slurred speech, ataxia, and nystagmus. It also may cause mild tachycardia, hypertension, and tachypnea. Individuals using cannabis experience a high marked by a pleasurable feeling, increased sociability, and a decrease in anxiety and tension. Cannabis hyperemesis syndrome is a rare complication of cannabis use in which individuals experience abdominal pain, nausea, and vomiting that is typically relieved by hot showers. It is most common in chronic cannabis users. Cannabis use can be detected by self-report or drug testing. Testing is often not necessary clinically, due to the typically mild symptoms of cannabis intoxication. However, testing can be performed clinically in individuals with less common presentations, such as cannabis hyperemesis syndrome, and is frequently performed on individuals who work in professions in which there is a premium on safety, such as transportation. The most common method for cannabis drug testing is by urine test to detect delta-9-tetrahydrocannabinol. However, saliva, blood, and hair also can be used for testing. Urine testing has a higher sensitivity than specificity, which means there are more false positives than false negatives. Therefore, urine testing that is positive for cannabis should be confirmed with more specific testing methods, such as liquid or gas chromatography. Due to its pharmacokinetics, cannabis can be detected in urine testing for as long as 30 days, particularly in chronic users (who it can be found in for many months). Most patients who present in clinical settings with cannabis intoxication have mild symptoms, such as dysphoria. These patients can be treated with reassurance, a dimly lit room, decreased stimulation, and benzodiazepines, such as lorazepam, in some cases. The average detection period of phencyclidine (PCP) in the urine for chronic users is 14 days (A). The typical detection period for cocaine in urine drug testing is 2 days (B). However, with chronic heavy use, cocaine can sometimes be detected for up to 2 weeks in the urine. Cannabis can be detected for as long as 30 days, not 60 days (D), however it can be found for months in chronic users.

A 32-year-old woman presents to the clinic complaining of irritability, difficulty concentrating, and fatigue for the past 8 months. She reports that she feels stressed at home and work and easily becomes anxious in response to trivial stressors. She reports no anhedonia or a depressed mood. You decide to start her on venlafaxine to treat the suspected diagnosis. How long should treatment be continued before increasing the dose or switching to another medication if she is not experiencing improvement on a therapeutic dose? A. 12 weeks B. 3 weeks C. 6 weeks D. 9 weeks

C. 6 weeks 12 weeks (A), 3 weeks (B), and 9 weeks (D) are each incorrect. Patients who are taking either selective serotonin reuptake inhibitors or serotonin and norepinephrine inhibitors for the treatment of generalized anxiety disorder should have their dose increased or be switched to a new medication if they are not experiencing significant improvement in symptoms after 4-6 weeks.

A 23-year-old woman with an unknown medical history presents to the emergency department via ambulance with altered mental status. The patient's partner reports finding her unconscious at home with an empty bottle of alprazolam nearby. Exam reveals a somnolent patient with slurred speech. Current vital signs are temperature 98.2°F, BP 98/65 mm Hg, HR 52 beats per minute, RR 10 breaths per minute, and oxygen saturation 92% on room air. You and your team suspect benzodiazepine overdose and are considering administering a medication to reverse the effects of the benzodiazepine. Which of the following most accurately describes the method of action of this medication? A. Agonist of GABA receptor B. Agonist of nicotinic acetylcholine receptor C. Antagonist of GABA receptor D. Antagonist of opiate receptor

C. Antagonist of GABA receptor Agonist of GABA receptor (A) describes the mechanism of action of benzodiazepines themselves. Agonist of nicotinic acetylcholine receptor (B) describes the mechanism of action of varenicline, which is given for smoking cessation. Antagonist of opiate receptor (D) describes the method of action of naloxone, which is given in opioid overdose. Naloxone can be useful in the incidence of benzodiazepine overdose if coingestion of alcohol or an opioid medication is suspected.

A 35-year-old man presents to the clinic for an annual wellness exam. You ask him about tobacco use, and he reports that he smokes cigarettes daily. According to the five As approach to helping patients with tobacco cessation, which of the following is the third step? A. Advise quitting B. Arrange follow-up C. Assess readiness to quit D. Assist smokers who are ready to quit

C. Assess readiness to quit The U.S. Preventive Services Task Force recommends that clinicians use the five As approach to help patients stop using tobacco. In sequential order, the five As are ask, advise, assess, assist, and arrange. The first step is for the clinician to ask the patient about tobacco use. Patients who report using tobacco should be advised to quit using tobacco, which is the second step. The third step is to assess the patient's readiness to quit, as smokers vary in their readiness to change tobacco use. The Stages of Change is a model that discusses five stages patients may go through on the continuum between smoking and abstinence: precontemplation (not ready to quit), contemplation (considering a quit attempt), preparation (actively planning a quit attempt), action (actively involved in a quit attempt), and maintenance (achieved smoking cessation). The fourth step is to assist smokers who are ready to quit, which is defined as anyone who is at least at the contemplation stage. It is important for the clinician to understand what methods the patient has previously used to quit smoking and whether the patient has had any success with these methods. Patients who want to quit using tobacco should set a quit date. The clinician should address barriers to quitting (e.g., triggers to smoke) and discuss the possibility of nicotine withdrawal symptoms. The symptoms of nicotine withdrawal peak in the first 3 days of smoking cessation and then gradually subside over the next 3-4 weeks. The symptoms of nicotine withdrawal include increased appetite, weight gain, changes in mood (dysphoria or depression), insomnia, irritability, anxiety, and restlessness. These symptoms can be relieved by pharmacologic treatment, including nicotine replacement, varenicline, or bupropion. The fifth step of the five As approach is to arrange follow-up. The recommended time for follow-up is 1-2 weeks after the patient's quit date. This timing of follow-up allows the clinician to monitor smoking cessation therapy, including adverse effects of pharmacotherapy, and address any questions the patient may have. Advise quitting (A) is the second step, assist smokers who are ready to quit (D) is the fourth step, and arrange follow-up (B) is the fifth step of the five As approach to helping patients with tobacco cessation.

Which of the following is the most appropriate intervention in suspected child abuse? A. Arrange for the arrest of the parents. B. Confront the suspected assailant in front of the child. C. Assure the safety of the child, with hospitalization if necessary. D. Contact social service department after discharge of the child.

C. Assure the safety of the child, with hospitalization if necessary. The primary goal should be the safety and well-being of the child. Hospitalization may be the only way the clinician has to remove the child from the care of a possible abusive home if no other recourse is available due to a lack of social services and investigators.

A mother brings in her five year-old boy for his school physical. She voices some concerns about his readiness for school, saying he seems to be socially immature. She has noticed he does not interact with other children well, and that when he plays with them, he has a tendency to "place them" and then run around them as if they were statues. He rarely cries when he is hurt, and he shrugs off any attempt to hug him. He has good attention to details, and will sit and draw the same geometric shapes over and over again, but does not seem interested in learning the alphabet. He avoids eye contact with anyone. Which of the following is the most likely diagnosis? A. Normal 5 year-old B. Social phobia C. Autism D. Avoidant personality

C. Autism Children with autism do not tend to make eye contact, and even avoid it. They do not accept comfort when hurt and stiffen up when hugged. They do not tend to play with others, and do not tend to imitate grown-ups in play. They approach play in a more mechanical way, using others as props rather than interacting with them.

A 25-year-old woman presents for evaluation after making a suicidal threat. While interviewing her, she reports difficulties with interpersonal relationships. She feels that people in her life do not support her as much as they should and are never there for her when she needs them. She reports fluctuations in her mood that can be intense throughout a single day, changing from feeling good to being depressed and angry if something happens that causes her stress. She states that she always feels empty and has taken extreme measures at times to avoid being abandoned by her friends. She reports that she has gone on spending sprees to cheer herself up when she has been stressed. Which of the following is the most likely diagnosis? A. Attention-deficit/hyperactivity disorder B. Bipolar disorder C. Borderline personality disorder D. Major depressive disorder

C. Borderline personality disorder Attention-deficit/hyperactivity disorder (A) can include impulsivity, but it is not characterized by the fear of abandonment or recurrent self-injury like borderline personality disorder. Bipolar disorder (B) includes mood disturbances that last longer and are not as labile as the affective states of borderline disorder. Mood changes in bipolar disorder are less triggered by stressful environmental factors than those changes associated with borderline personality disorder. Major depressive disorder (D) is characterized by depression that is present for at least 2 weeks, and borderline personality disorder includes depressive episodes that fluctuate with events daily. Which of the disorder's diagnostic criteria is most commonly found in individuals with borderline personality disorder? -Affective instability.

A 22-year-old man presents to his therapist to discuss his fear of spiders. He reports that every night before he goes to sleep he takes off every pillowcase and sheet to ensure there are no spiders in his bed. He also avoids outdoor activities in which he may encounter a spider, including walking around the neighborhood and going in his backyard. Which of the following is the recommended first-line treatment for the suspected condition? A. Alprazolam B. Cognitive behavioral therapy that includes imaginal exposure C. Cognitive behavioral therapy that includes real-world exposure D. Sertraline

C. Cognitive behavioral therapy that includes real-world exposure Alprazolam (A) is a benzodiazepine. Benzodiazepines can be used as needed as second-line therapy, short term, for patients who do not have access to cognitive behavioral therapy with exposure, cannot tolerate cognitive behavioral therapy with exposure, or who prefer medications. Cognitive behavioral therapy that includes imaginal exposure (B) is acceptable, but real-world exposure is preferred unless it is too infrequent or too expensive to recreate. Sertraline (D) is a selective serotonin reuptake inhibitor. Selective serotonin reuptake inhibitors are not typically used to treat patients with specific phobias because there is minimal evidence to support their use and a potential for adverse effects, such as sexual dysfunction. Specific phobias are a type of anxiety disorder in which patients experience clinically significant anxiety or fear related to anticipation of or exposure to situations or objects. The anxiety frequently leads to avoidant behaviors in which individuals attempt to reduce the likelihood of being exposed to the feared situation or object. Specific phobias begin most often during childhood or adolescence. The pathogenesis has both genetic and environmental components. The main categories of specific phobias are animal, natural environment (heights, storms, or water), blood-injection-injury, situational (airplanes, elevators, or enclosed spaces), and other. The severity of functional impairment varies according to the degree of the fear and the situation or object of fear. For example, fear of snakes may have minimal impact on an individual's life, whereas fear of injections may lead to an individual avoiding necessary medical procedures. The recommended treatment of specific phobias is cognitive behavioral therapy that includes real-world exposure. Benzodiazepines can be used as a second-line treatment when cognitive behavioral therapy with exposure is unavailable or cannot be tolerated, or in patients who prefer medications on a short-term basis.

A 39-year-old woman presents to the clinic with a persistent depressed mood. She reports anhedonia, increased sleep, and feelings of worthlessness for most days of the week, most weeks of the year, without a significant symptom-free period over the last 6 years. She reports no suicidal ideation, mania, hypomania, or psychotic features. She is otherwise healthy. Which of the following treatment options is the best choice for this patient? A. 6 months of pharmacotherapy followed by life-long psychotherapy B. 6 months of psychotherapy followed by life-long pharmacotherapy C. Combined pharmacotherapy and psychotherapy for an indefinite period D. Pharmacotherapy alone for an indefinite period

C. Combined pharmacotherapy and psychotherapy for an indefinite period 6 months of pharmacotherapy followed by life-long psychotherapy (A) may not be adequate for patients with persistent depressive disorder. Patients experiencing their first episode of major depressive disorder may prefer a trial of 6 months of antidepressant therapy followed by a reprieve to see if normal mood is restored. However, patients with more than two episodes of major depression should be treated with pharmacotherapy, psychotherapy, or both indefinitely. 6 months of psychotherapy followed by life-long pharmacotherapy (B) would also not be adequate treatment for patients with persistent depressive disorder. While lifelong pharmacotherapy is advised, concomitant treatment with both psychotherapy and pharmacotherapy for an indefinite period (life-long) is most efficacious. Pharmacotherapy alone for an indefinite period of time (D) may be what some patients choose based on preference and financial concerns, but in general, better outcomes are demonstrated with dual pharmacotherapy and psychotherapy indefinitely.

A 21-year-old man presents to the clinic with his father for a routine follow-up. His father notes he has improved significantly over the past few months. He states the patient is no longer worried that the FBI has hacked his computer and cell phone. The patient reports he is no longer hearing voices, and he has been making efforts to attend social gatherings on the weekend. The patient has been stable and treated with clozapine. Which of the following lab panels should be monitored for this patient? A. Arterial blood gas B. Basic metabolic panel C. Complete blood count D. Hepatic function panel

C. Complete blood count Arterial blood gas (A) is used to determine respiratory or metabolic acidosis or alkalosis. Basic metabolic panel (B) is important for checking kidney function along with electrolytes. Hepatic function panel (D) is used to check liver function. Use of antipsychotics, especially clozapine, is not known to affect any of these laboratory tests.

A 16-year-old boy presents to the clinic with his parents after being arrested for killing the neighbor's dog. The parents report that he has vandalized his school and other property and is consistently getting into fights with other children at school. Which of the following is the most likely diagnosis? A. Antisocial personality disorder B. Borderline personality disorder C. Conduct disorder D. Oppositional defiant disorder

C. Conduct disorder Antisocial personality disorder (A) cannot be diagnosed until 18 years of age. However, symptoms typically begin during childhood and adolescence and are initially diagnosed as conduct disorder. Both conditions have similar clinical features, including vandalism, violence against others, and lack of empathy. Borderline personality disorder (B) is marked by unstable behavior, mood, and interpersonal relationships; fear of abandonment; impulsivity; and increased risk of suicidal behaviors. Personality disorders typically become apparent in young adulthood. Oppositional defiant disorder (D) is another childhood disorder that is marked by irritability, anger, and defiance against authority figures. However, the symptoms are much less severe than in conduct disorder. The typical manifestations of oppositional defiant disorder do not go to the extent of being dangerous toward others.

A 59-year-old man with a past medical history of obesity, hypertension, generalized anxiety disorder, and type 2 diabetes mellitus presents to the primary care clinic for his annual wellness exam. Medications include lisinopril 40 mg once daily, sertraline 50 mg once daily, and metformin 1,000 mg twice daily. His blood pressure today is 150/90 mm Hg, and his A1C is 9.2%. During his visit, the patient mentions concerns about worsening difficulty achieving and maintaining an erection for the past 3 months. He is upset about these symptoms and is requesting medication. What is the most appropriate next step in management of the patient's suspected diagnosis? A. Begin cognitive behavioral therapy B. Begin sildenafil as needed C. Decrease dose of sertraline D. Use of a vacuum-assisted device

C. Decrease dose of sertraline Beginning cognitive behavioral therapy (A) may be helpful, especially in the instance of comorbid psychiatric conditions, but it is not considered a first-line therapy for ED. Beginning sildenafil as needed (B) would be most appropriately done after lifestyle changes and medication adjustments have been made to better control comorbid conditions and sertraline is decreased in dosage or changed to another medication. The patient will likely see improvement or resolution in symptoms after these changes are made. Use of a vacuum-assisted device (D) may be necessary if first-line management, including watchful waiting and medication adjustments, fails.

A 19-year-old man presents to the office with concerns and wants to be evaluated. He reports his friend gave him a brownie approximately 1 hour ago that contained an unknown substance. You note slurred speech. He appears mildly paranoid. His vital signs indicate tachycardia and a slightly elevated blood pressure. Eye examination reveals conjunctival injection with normal pupils and horizontal gaze nystagmus. What component of this unknown substance is primarily responsible for its psychoactive response, given the most likely cause for his presentation? A. 3,4-methylenedioxymethamphetamine B. 3-methylfentanyl C. Delta-9-tetrahydrocannabinol D. Lysergic acid diethylamide

C. Delta-9-tetrahydrocannabinol 3,4-methylenedioxymethamphetamine (A) is the active ingredient in the stimulant ecstasy. Acute intoxication with ecstasy can cause increased alertness and euphoria. Vital signs will often show elevations in blood pressure and heart rate, as observed in the patient above, but hyperthermia can also occur. Ecstasy is associated with serotonin syndrome. 3-methylfentanyl (B) is the active ingredient in fentanyl. Acute intoxication with fentanyl would result in depression of vital signs, particularly respiratory rate, and pupillary constriction. Acute intoxication with lysergic acid diethylamide (D) (LSD) results in hallucinations, anxiety, and delusions, often referred to as a trip. Patients often have observable pupillary dilation.

A 42-year-old man presents to marital counseling with his wife who he states is being unfaithful. He believes she is unfaithful because she typically arrives home late from work about twice a month and has male coworkers. His wife has attempted to reassure him by sharing her location with him and having several other coworkers vouch that she is working the nights she comes home late. Nonetheless, he remains unconvinced. This has led to conflict over the past 6 months. Which of the following is the most likely diagnosis? A. Bipolar disorder type 1 B. Brief psychotic disorder C. Delusional disorder D. Schizophrenia

C. Delusional disorder Bipolar disorder type 1 (A) is a condition defined by the presence of at least one manic episode. Manic episodes are defined by a persistently elevated or irritable mood that lasts for at least a week and causes an increase in goal-directed behavior. Patients who are experiencing a manic episode may have delusions, but patients with delusional disorder must experience delusions in the absence of a manic episode. The patient in the vignette does not meet the criteria for a manic episode. Brief psychotic disorder (B) is marked by hallucinations or delusions that last more than a day but less than a month. The patient described in the vignette has a symptom duration of at least 6 months. Schizophrenia (D) is another psychiatric condition that may cause delusions. However, patients with delusional disorder do not exhibit other important features of schizophrenia, such as hallucinations (unless they are associated with the delusion), disorganized thoughts or behavior, and negative symptoms. The patient in the vignette does not have these other symptoms to suggest schizophrenia. Delusional Disorder One or more delusions for at least 1 month Criterion A for schizophrenia never met Apart from the impact or ramifications of the delusion(s), functioning not markedly impaired and behavior not obviously odd or bizarre Erotomanic: belief that another person is in love with the individual Grandiose: conviction of having some great talent or insight Jealous: delusion that partner is unfaithful Persecutory: belief that being conspired against, spied on, poisoned, etc. Somatic: delusion about bodily functions or sensations With bizarre content: delusions are clearly implausible, not understandable, and not derived from ordinary life experiences (e.g., belief that a stranger removed one's internal organs and replaced with another's organs without leaving a scar)

A 26-year-old woman presents to the clinic for group therapy. She reports that she is prone to interpersonal submissiveness, has a fear of abandonment, and relies heavily on others in making everyday decisions. Which of the following is the most likely personality disorder? A.Avoidant B. Borderline C. Dependent D. Schizoid

C. Dependent Avoidant (A) personality disorder is a cluster C personality disorder marked by social inhibition, feelings of inadequacy, and hypersensitivity to negative evaluation. Individuals with avoidant personality disorder desire social relationships but fear criticism. Although dependent and avoidant personality disorders each involve feelings of inadequacy and hypersensitivity, dependent personality disorder is marked by reliance on others to make decisions, whereas avoidant personality disorder is more focused on a fear of criticism. Borderline (B) personality disorder is a cluster B personality disorder that is marked by unstable moods, behaviors, and interpersonal relationships; fear of abandonment; intense attachment; impulsive behavior; poorly formed identity; and increased risk of suicide. Although both dependent and borderline personality disorders share fear of abandonment as a core feature, this fear results in appeasement toward others in dependent personality disorder and manipulation toward others in borderline personality disorder. Schizoid (D) personality disorder is a cluster A personality disorder marked by social isolation due to a lack of desire for close interpersonal relationships. These individuals choose solitary activities, have little interest in sexual experiences, and are indifferent to the praise or criticism of others.

A 63 year-old retired engineer presents with one month of difficulty sleeping. He has a hard time staying asleep and says he is just restless. He also states that he has been more forgetful and can't pay attention very well. He lives with his wife. He denies fever, chills, recent trauma, or difficulty walking. When asked about the specifics of his symptoms, he repeatedly replies, "I don't know," without really trying. He states his wife thinks he just sits around the house all day since he retired. His neurological exam is unremarkable, except for some mild psychomotor retardation. Which of the following is the most likely diagnosis? A. Dementia B. Delirium C. Depression D. Dissociative disorder

C. Depression Depression often presents with difficulty thinking and concentrating, lessened sleep, and withdrawal from activities.

A 42-year-old woman with a history of schizophrenia presents to the emergency department, brought by EMS, after being found downtown. She is very agitated. The patient is unable to be verbally calmed and is becoming a safety concern to herself and staff. You order haloperidol 5 mg IM to be administered to calm the patient. After administration, the patient begins having involuntary contractions of her neck, causing torticollis. What medication can be used to treat the patient's suspected diagnosis? A. Atropine B. Cyclobenzaprine C. Diphenhydramine D. Prochlorperazine

C. Diphenhydramine Atropine (A) is an anticholinergic and antispasmodic medication most commonly used in the acute treatment of bradycardia and organophosphate poisoning by blocking the action of acetylcholine at parasympathetic sites in secretory glands, smooth muscle, and the central nervous system. It helps to increase cardiac output and dries secretions. It is not indicated for the treatment of dystonic reactions. Cyclobenzaprine (B) is a skeletal muscle relaxant that reduces tonic motor activity that causes acute muscle spasms. Cyclobenzaprine is indicated for the acute relief of muscle spasms associated with musculoskeletal conditions, such as cervical strains or muscle sprains. It should only be used for the short term, 1-3 weeks. Prochlorperazine (D) is an antiemetic and was one of the first typical (first-generation) antipsychotics. Prochlorperazine blocks dopamine receptors in the brain and stomach, so it is often used in the treatment of nausea or vomiting. It may cause a dystonic reaction due to the dopamine blockade. Diphenhydramine is often administered at the same time as prochlorperazine to prevent a dystonic reaction.

A 35-year-old patient reports an alcohol intake of two cases of beer per week. This alcohol consumption causes him to miss work frequently, and he reports risky sexual encounters while acutely intoxicated. He would like to discuss options to help him reduce his reliance on alcohol. In addition to behavioral and social interventions, which of the following medications is appropriate? A. Acarbose B. Chlordiazepoxide C. Disulfiram D. Naloxone

C. Disulfiram Alcohol use disorder is a behavioral disorder in which patients repetitively use alcohol to manage emotional problems, such as anxiety. Patients with moderate to severe alcohol use disorder continue this at-risk drinking despite the negative impact on their daily lives, including social role disruption, alcohol-related legal problems, and dangerous and risky behaviors. At-risk drinking is defined by the National Institute on Alcohol Abuse and Alcoholism as more than four drinks per day or 14 drinks per week for men and more than three drinks per day or seven drinks per week for women. The Centers for Disease Control and Prevention defines one drink as 12 oz of beer, 8 oz of malt liquor, 5 oz of wine, or 1.3 oz of 80-proof distilled spirits. Patients with alcohol use disorder are more likely to have a psychiatric disorder in their lifetime, with depression being a common comorbidity. Men are four times more likely to be diagnosed with alcohol use disorder, but rates are beginning to converge according to recent data. As alcohol is a central nervous system depressant, patients who present with acute intoxication often exhibit drowsiness, errors of commission, psychomotor dysfunction, disinhibition, dysarthria, ataxia, and nystagmus. Alcohol withdrawal, alternatively, presents with anxiety, decreased cognition, tremulousness, and irritability. Severe cases of alcohol withdrawal can result in life-threatening delirium tremens. Patients with chronic alcoholism can develop encephalopathy. Wernicke encephalopathy consists of a triad of confusion, ataxia, and ophthalmoplegia (i.e., sixth nerve palsy). Subsequent Korsakoff psychosis is characterized by anterograde amnesia with confabulation. Early recognition and treatment with thiamine can minimize these organic brain injuries. Chronic alcohol use often leads to laboratory abnormalities, including increased serum liver biochemical tests, uric acid, and triglycerides with decreased serum potassium and magnesium. At-risk drinking should be recognized and addressed early with an initial goal of harm reduction and an ultimate goal of total abstinence. Alcoholics Anonymous, religious counseling, and other social resources can aid the patient in reducing use and abstaining from alcohol. Abstinence is the recommended approach. Medical approaches include disulfiram and naltrexone to discourage alcohol use. Naltrexone can be initiated while the patient is still drinking while disulfiram causes unpleasant side effects such as flushing and palpitations if used concomitantly with alcohol. Acamprosate may also be used once abstinence is achieved in order to reduce the symptoms of withdrawal. Acarbose (A) is an alpha-glucosidase inhibitor that blocks the breakdown of starches into sugar and thereby helps to control blood glucose levels in patients with diabetes. Chlordiazepoxide (B) is a long-acting benzodiazepine that is commonly used to treat alcohol withdrawal syndrome. It is not an appropriate agent for the management of alcohol dependence. Naloxone (D) is a potent opioid antagonist that is most frequently used to reverse opioid overdose. It has no known beneficial role in alcohol use disorder.

A 19-year-old woman presents to the primary care clinic with decreased pleasure in usually pleasurable activities for the past 3 months. She also reports hypersomnia, decreased appetite, difficulty concentrating, and fatigue. Which of the following is more prevalent for the suspected diagnosis? A. Asian ancestry B. Cigarette smoking C. Divorced marital status D. High socioeconomic status

C. Divorced marital status Major depressive disorder is a condition marked by depressed mood or dysphoria that lasts at least 2 weeks and is accompanied by at least five of the following features: anhedonia, changes in appetite, changes in weight, sleep disturbances (insomnia or hypersomnia), fatigue, neurocognitive dysfunction, feelings of guilt, and suicidal ideation. Depression is most common in young adults and is more common in women than men. Risk factors for depression include a family history of first-degree relatives having depression; low socioeconomic status; marital status of divorced, separated, or widowed; lack of interpersonal relationships; recent difficult life events; early childhood trauma (e.g., childhood physical abuse, childhood emotional abuse, or neglect); and postpartum status. The diagnostic criteria for major depressive disorder are listed in the fifth edition of the Diagnostic and Statistical Manual of Mental Disorders and require that the symptoms last at least 2 weeks and cause clinically significant impairment. The first-line treatment for major depressive disorder is antidepressants. Selective serotonin reuptake inhibitors, such as sertraline or escitalopram, are typically the antidepressant of choice due to their safety profile. Individuals of American Indian descent have a higher prevalence of depression compared to other ethnicities in the United States, and individuals of Asian ancestry (A) have the lowest prevalence. Cigarette smoking (B) is not a known risk factor for major depressive disorder. High socioeconomic status (D) is incorrect because the prevalence of major depressive disorder is higher in individuals with low socioeconomic status.

A 21-year-old man presents to the emergency department after his parents found him wandering the neighborhood at night. He reports that a voice told him to look for treasure that was buried for him underneath a house in the neighborhood. The patient is demanding that he be discharged so he can find the treasure. His parents have noticed behavioral changes over the past 6 months. He has had less emotional expression and been less motivated. His urine drug screen result is negative. You decide to start him on ziprasidone. Which of the following should be checked prior to starting the patient on this medication? A. Chest X-ray B. Complete blood count C. Electrocardiogram D. Thyroid-stimulating hormone

C. Electrocardiogram Chest X-ray (A) does not need to be checked prior to starting an antipsychotic medication. Complete blood count (B) should be checked in patients who take clozapine to monitor for agranulocytosis. Thyroid-stimulating hormone (D) is not checked in patients prior to starting an antipsychotic

A 45-year-old man with a history of chronic alcohol use presents to the emergency department with confusion. You are concerned for a condition caused by thiamine deficiency. Which of the following is the classic triad associated with this condition? A. Dysarthria, encephalopathy, and vertigo B. Dysarthria, facial numbness, and gait ataxia C. Encephalopathy, oculomotor dysfunction, and gait ataxia D. Encephalopathy, papilledema, and vertigo

C. Encephalopathy, oculomotor dysfunction, and gait ataxia Dysarthria and vertigo (A) are not typical findings of Wernicke encephalopathy. Facial numbness (B) is not caused by Wernicke encephalopathy. However, patients may have paresthesias in the extremities due to peripheral neuropathy. Papilledema (D) is not a typical finding of Wernicke encephalopathy. Wernicke Encephalopathy Associated with chronic alcohol use Ataxia and confusion PE will show nystagmus, lateral rectus palsy Most commonly caused by thiamine (B1) deficiency Treatment is aggressive thiamine repletion Replace thiamine BEFORE glucose Korsakoff (irreversible memory loss)

A 36 year-old patient presents requesting something to help him sleep. He reports that he has always had a problem sleeping, admits to feeling nervous most days for the last 2 years, and that he has always been "uptight" and a "worry wart." During the previous eight months he has frequently felt tense, shaky, sweaty, with palpitations and frequent headaches. He reports being irritable with his 5 year-old son. Which of the following is the best treatment option for this patient? A. Alprazolam (Xanax) B. Haloperidol (Haldol) C. Escitalopram (Lexapro) D. Diphenhydramine (Benadryl)

C. Escitalopram (Lexapro) Certain antidepressants, including SSRIs (eg, escitalopram, starting dose of 10 mg po once/day) and serotonin-norepinephrine reuptake inhibitors (eg, venlafaxine extended-release, starting dose 37.5 mg po once/day) are effective but typically only after being taken for at least a few weeks. Benzodiazepines in small to moderate doses are also often and more rapidly effective, although sustained use may lead to physical dependence. One strategy involves starting with concomitant use of a benzodiazepine and an antidepressant. Once the antidepressant becomes effective, the benzodiazepine is tapered. Buspirone is also effective; the starting dose is 5 mg po bid or tid. However, buspirone can take at least 2 wk before it begins to help.

A 26-year-old woman presents to a plastic surgery clinic concerned about the appearance of her nose. She reports that her nose is long and causes her to have an ugly face. She frequently asks her close friends if her nose looks too long and spends several hours each day looking at her nose in the mirror, which temporarily makes her feel better. Physical exam reveals a relatively normal-appearing nose. You review her chart and notice she has previously had two plastic surgeries on her nose. Which of the following is an acceptable first-line treatment for the suspected diagnosis? A. Aripiprazole B. Buspirone C. Fluoxetine D. Venlafaxine

C. Fluoxetine Aripiprazole (A) is a second-generation antipsychotic that can be used to augment an SSRI in the treatment of body dysmorphic disorder. However, it is not a first-line agent. Buspirone (B) is an anxiolytic. It may be added in patients who are resistant to treatment with one medication, and if combined with SSRIs, it is important to monitor for serotonin syndrome. Venlafaxine (D) is a serotonin and norepinephrine reuptake inhibitor, which is another agent that can be used to augment an SSRI. Body Dysmorphic Disorder Concerned with perceived flaws or defects in physical appearance Commonly presents for cosmetic procedures rather than for mental health concerns Cosmetic procedures contraindicated, as likely to result in dissatisfaction and litigation or threats toward clinician Interventions include psychoeducation, modified CBT, SSRI

A 24-year-old woman presents to the emergency department after taking 20 acetaminophen tablets. She reports that her mother died recently, and she has had a depressed mood for months. She says she took the pills to "try to end it all." Which of the following risk factors is the most predictive of future suicide? A. Chronic pain B. Depression C. History of suicide attempts D. Identifying as LGBTQ

C. History of suicide attempts Chronic pain (A), depression (B), and identifying as LGBTQ (D) are all risk factors for suicide, but a history of suicide attempts is the most predictive risk factor of future suicide. True or false: ketamine has proven efficacy in decreasing suicidal ideation in suicidal patients. -True

An 8-year-old boy presents to the clinic with his mother, who reports he frequently daydreams, makes careless mistakes, and has difficulty with organization while doing schoolwork. She says this has led to him struggling academically, and she spends several hours each night helping him with homework. You discuss that the first-line treatment for the suspected condition is methylphenidate. Which of the following are the three core findings associated with the suspected diagnosis? A. Aggression, lying, and stealing B. Defiant behavior, hostility, and inattention C. Hyperactivity, impulsivity, and inattention D. Poor eye contact, repetitive behaviors, and social communication impairment

C. Hyperactivity, impulsivity, and inattention Aggression, lying, and stealing (A) are classic findings in conduct disorder. Conduct disorder is a childhood condition marked by a repetitive and persistent pattern of behavior that violates the basic rights of others or major societal norms and rules. Other common findings are destruction of property and serious violation of rules. Defiant behavior, hostility, and inattention (B) is incorrect because defiant behavior and hostility are classic findings in oppositional defiant disorder. Oppositional defiant disorder is a childhood condition marked by a maladaptive pattern of defiance. Poor eye contact, repetitive behaviors, and social communication impairment (D) are common findings in autism spectrum disorder. Autism spectrum disorder is a neurodevelopmental abnormality in children that is frequently diagnosed between 2 and 3 years of age.

A 24-year-old man presents to the clinic with depressed mood, feelings of worthlessness, extreme guilt, difficulty concentrating, decreased sleep, decreased appetite, and lack of interest in any activity. These symptoms have been present for more than 1 month and are worsening in severity. The patient states that recently he has felt that life is not worth living. Which of the following is an indication for hospitalization? A. Allergy to selective serotonin reuptake inhibitors B. Firearm in the home C. Inability to discuss safety planning D. Multiple concerned family members in the home

C. Inability to discuss safety planning Major depressive disorder is a mood disorder that is characterized by depressed mood and anhedonia. Five or more depressive symptoms must be present most of the day, most days of the week, for a period of 2 weeks or more for a diagnosis. These symptoms include depressed mood, anhedonia, changes in sleep pattern, changes in appetite, feelings of excessive guilt, thoughts of worthlessness, suicidal thoughts, psychomotor agitation, fatigue, and indecisiveness. Risk factors for the development of major depressive disorder include certain medications, chronic pain, comorbid psychiatric disorders, childhood trauma, adverse life events, low socioeconomic status, neurotic personality traits, and comorbid medical disorders, such as cardiac disease or parkinsonism. Women have higher rates of major depressive disorder, but men have higher rates of suicide. Suicide is a significant complication of major depressive disorder. Treatment of major depressive disorder is with cognitive behavioral therapy and medication. First-line medications include selective serotonin reuptake inhibitors and serotonin and norepinephrine reuptake inhibitors. Patients at risk of suicide may require hospitalization for stabilization of depressive symptoms. Indications for hospitalization in a depressed patient who expresses suicidal ideation include a recent suicide attempt, a detailed suicide plan, inability to discuss safety planning, comorbid psychiatric disorders (such as borderline personality disorder, schizophrenia, or bipolar mood disorder), agitation, impulsivity, severe hopelessness, and poor social support. Allergy to selective serotonin reuptake inhibitors (A) does not necessitate inpatient treatment. Many other classes of antidepressants are available, such as tricyclic antidepressants, atypical antidepressants, and monoamine oxidase inhibitors. Some patients respond sufficiently with psychotherapy alone. The presence of a firearm in the home (B) does not automatically require a depressed patient to be hospitalized if the patient has no intent to use the firearm to harm themselves or others. However, a safety plan should be discussed with the patient and put in place that takes into account the presence of a firearm in the home. Most depressed patients should be counseled to store the weapon with a friend or relative so as not to have easy access to it. Multiple concerned family members in the home (D) are a deterrent to suicide, not a risk factor. Poor social support and living alone may be indications for hospitalization in a patient who expresses suicidal intent.

A 22-year-old man presents to the clinic complaining of sweating, tremors, and nausea. He states he feels this way if he goes 1 day without drinking alcohol. He has been drinking about six beers per day and more on the weekends for the past 2 years. Which of the following signs and symptoms is consistent with a diagnosis of mild alcohol withdrawal? A. Delirium tremens B. Hallucinations C. Insomnia D. Sinus bradycardia

C. Insomnia Delirium tremens (A) is considered a severe manifestation of alcohol withdrawal, as are hallucinations (B). Sinus bradycardia (D) is not a symptom of alcohol withdrawal syndrome. Sinus tachycardia is a common sign of alcohol withdrawal syndrome. Beta-blockers may control the tachycardia of alcohol withdrawal in the short-term but do not prevent the development of delirium tremens or seizures and should not be used in place of benzodiazepines. Patients with cardiac disease may be administered cardiac medications after initial volume repletion and sedation. Alcohol Withdrawal Syndrome Early symptoms: anxiety, nausea, restlessness Risk of seizure Delirium tremens may start after 48 hours Treat with benzodiazepines Correct nutritional deficiencies Offer alcohol cessation support and resources

A 37-year-old man presents to the office with concerns of increasing stress and "difficulty controlling urges." He notes for the past few years he has found himself sexually aroused by young children. This has manifested through sexual fantasies. He reports he has not acted on these fantasies (he has not watched child pornography or sexually abused a prepubescent child), but he is finding his fantasies are becoming more intense and more difficult to control, and they are causing him significant distress. He recently contacted a young boy over the internet. The patient requests pharmacologic therapy that can reduce the frequency of his fantasies and his sex drive. What should be recommended at this time? A. Flibanserin B. Fluoxetine C. Leuprolide acetate D. Surgical castration

C. Leuprolide acetate Flibanserin (A) is a 5-hydroxytryptamine type 1A agonist that is used in the treatment of female hyposexual desire disorders. Fluoxetine (B) is incorrect. While selective serotonin reuptake inhibitors can be used in paraphilic disorders to reduce compulsive sexual urges, a gonadotropin-releasing hormone agonist, such as leuprolide acetate, is the correct option to reduce the frequency of sexual fantasies and also address the patient's sex drive. Surgical castration (D) would not be considered a pharmacologic intervention. While historically it has been considered successful, it is not recommended in the United States because of ethical implications.

A 25-year-old man presents to the clinic at the urging of his family members, who state that he does not have social relationships with family or close friends. The patient reports that he does not desire relationships. He enjoys his job as a zoologist and likes hiking and bird-watching by himself. Which of the following additional findings is consistent with the suspected personality disorder? A. Anxiety about social interactions B. Impulsivity C. Little interest in sexual activity D. Nonbizarre delusions

C. Little interest in sexual activity Anxiety about social interactions (A) is seen in avoidant personality disorder. However, individuals with schizoid personality disorder have limited emotional response and are content with their reclusive lifestyle. Impulsivity (B) is a core feature of borderline personality disorder. Nonbizarre delusions (D) are delusions that are within the realm of possibility. They are often seen in schizotypal personality disorder. Schizoid personality disorder is a cluster A personality disorder characterized by a lack of interest in social interactions, which leads to a reclusive lifestyle. These individuals are not anxious about social interactions, as seen in avoidant personality disorder, but simply prefer not to have them. People with schizoid personality disorder frequently have solitary occupations and activities, little interest in sexual activity, and few close friends. They are indifferent to praise or criticism. The diagnosis of schizoid personality disorder is made according to the criteria in the fifth edition of the Diagnostic and Statistical Manual of Mental Disorders and is based on a pattern of voluntary social withdrawal that begins during early adulthood and occurs in a variety of settings. Individuals with schizoid personality disorder frequently do not seek treatment without the urging of family members. Pharmacotherapy has not proven to be effective. Group and family therapy can help the individual develop social skills and allow the family to give feedback.

A 25-year-old man presents to the emergency department, brought by emergency medical services, after getting into an altercation at a nightclub. He is agitated and yelling at the staff in the emergency department. His vital signs are notable for a HR of 135 bpm and blood pressure of 208/115 mm Hg. On exam, you notice that he is diaphoretic and has dilated pupils. You are able to obtain further information from his roommate who reports he is likely intoxicated with a substance made from pseudoephedrine in a home laboratory. Which of the following medications is considered first-line treatment for his elevated blood pressure? A. Esmolol B. Haloperidol C. Lorazepam D. Nitroprusside

C. Lorazepam Esmolol (A) is a beta-blocker. Beta-blockers should not be used in patients with sympathomimetic intoxication, such as with cocaine or amphetamines, because of the risk for unopposed alpha-agonist stimulation. Alpha-1 receptors are located on blood vessels and increase blood pressure when stimulated. Haloperidol (B) is a first-generation antipsychotic. It is considered a second-line treatment for agitation in patients with amphetamine intoxication. However, it has no role in the treatment of elevated blood pressure from amphetamine intoxication. Nitroprusside (D) is a vasodilator. It is second-line treatment to control elevated blood pressure in the setting of amphetamine intoxication. However, the preferred pharmacologic agent to control blood pressure is a benzodiazepine. Which antidepressant medication can cause a false positive for amphetamines on a urine drug assay? -Bupropion

A 20-year-old gymnast presents with amenorrhea. Her body mass index is 16 kg/m2. She says she feels fat and exercises for hours every day in addition to practicing gymnastics. She reports she eats one small meal per day and limits the calories she eats because she is afraid of gaining weight. Which of the following is included in the female athlete triad? A. Eating food more rapidly than normal B. Eating nonfood substances C. Loss of bone mineral density D. Tooth enamel loss

C. Loss of bone mineral density Eating food more rapidly than normal (A) is characteristic of binge eating disorder and bulimia nervosa. Eating nonfood substances (B) is typical of pica eating disorder. Tooth enamel loss (D) can be a result of frequent regurgitation or vomiting associated with bulimia nervosa or rumination disorder.

A 23-year-old woman presents to the clinic with decreased pleasure in usually enjoyable activities for the past month. She also reports hypersomnia, weight gain, decreased concentration, and excessive guilt. Which of the following is the most likely diagnosis? A. Bipolar disorder type 2 B. Cyclothymic disorder C. Major depressive disorder D. Persistent depressive disorder

C. Major depressive disorder Bipolar disorder type 2 (A) is diagnosed in patients who meet the criteria for a major depressive episode and a hypomanic episode. Hypomania is defined by an abnormal and persistently elevated or irritable mood, an increase in energy or goal-directed activity that lasts at least 4 days, and at least four of the symptoms that characterize mania. Hypomanic episodes are less severe and have a shorter duration than manic episodes. Cyclothymic disorder (B) is defined by recurrent episodes of hypomanic or depressive symptoms that do not meet the criteria for bipolar disorder type 2 or major depressive disorder, respectively. The diagnostic criteria for cyclothymic disorder require that symptoms be present at least half the time for a period of 2 years. Persistent depressive disorder (D) is defined by depressed mood for at least 2 consecutive years with at least two of the following symptoms: decreased or increased appetite, insomnia or hypersomnia, low energy or fatigue, low self-esteem, impaired concentration or decision making, and hopelessness.

A 35-year-old woman presents with hearing loss and tinnitus that she says are secondary to working in a loud environment without the proper hearing protection. She is currently involved in a lawsuit against her former employer. An audiogram is performed and shows inconsistent results. An otoacoustic emissions test and an electronystagmography are performed and are within reference range. What is the most likely diagnosis? A. Conversion disorder B. Factitious disorder imposed on self C. Malingering D. Somatic symptom disorder

C. Malingering Malingering is characterized by an individual creating false medical or psychiatric symptoms for external reward or gain. This behavior is intentional deception for the purpose of obtaining a reward, such as money, medications (e.g., opiates), or time off from work. Individuals may tamper with medical instruments or specimens, forge medical records, remain noncompliant with medical treatment to worsen an existing condition, or fabricate symptoms. Malingering patients will often try to avoid invasive tests or painful medical procedures. These patients will also try to avoid testing that will expose the factitious disorder Conversion disorder (A) involves neurologic symptoms without any obvious neurological pathophysiology and without evidence of deception or falsification. In patients who have a factitious disorder imposed on self (B), individuals may report factitious symptoms or stimulate symptoms in the absence of external reward or gain. Somatic symptom disorder (D) is characterized by somatic symptoms and treatment-seeking behaviors without evidence of deception or falsifying information. These patients become preoccupied with their symptoms.

A 48-year-old man presents to the clinic complaining of fatigue. He reports drinking 6 to 10 beers per day for the past 15 years. He currently takes lisinopril for hypertension. Physical exam reveals rhinophyma, facial erythema, hepatomegaly, an abdominal fluid wave, and external hemorrhoids. Which of the following laboratory abnormalities would be most likely in this patient? A. Alanine aminotransferase to aspartate aminotransferase ratio of 4:1 B. Gamma-glutamyltransferase 20 IU/L C. Mean corpuscular volume 105 fL D. Serum albumin 5.7 g/dL

C. Mean corpuscular volume 105 fL Alcohol use disorder can be diagnosed when a patient exhibits recurrent drinking leading to neglecting obligations, recurrent drinking in high-risk situations, drinking that leads to interpersonal problems, evidence of tolerance to alcohol, evidence of alcohol withdrawal, alcohol craving, desire to stop drinking with unsuccessful quit attempts, and a great deal of time spent surrounding alcohol-related pursuits. Patients at increased risk for alcohol use disorder are those under age 24, and men. Health problems attributable to alcohol use disorder include hypertension, anxiety, depression, gastrointestinal reflux, sleep disturbance, liver disease, bone marrow suppression, macrocytosis (mean corpuscular volume over 96 fL), cardiomyopathy, neuropathy, and trauma due to falls. Facial erythema, rhinophyma, superficial spider hemangiomas, varices, and hemorrhoids can also occur with chronic alcohol use. Treatment of alcohol use disorder depends on disease severity. Patients with mild disease benefit from psychosocial intervention alone, while those with moderate-to-severe disease often require an initial period of medically managed detoxification with benzodiazepines and antihypertensives. After initial withdrawal, patients with moderate-to-severe disease may benefit from psychosocial interventions plus naltrexone. Liver function tests are often abnormal in patients with alcohol use disorder. In general, a ratio of aspartate aminotransferase to alanine aminotransferase of 2:1 is indicative of alcohol use disorder, not an alanine aminotransferase to aspartate aminotransferase ratio of 4:1 (A). Gamma-glutamyltransferase 20 IU/L (B) is within normal limits. Patients with alcohol use disorder will often have gamma-glutamyltransferase levels well above the upper limits of normal (over 30 IU/L). Serum albumin 5.7 g/dL (D) is slightly above the normal range. Patients with alcohol use disorder are often undernourished and may have serum albumin levels under the lower limits of normal (3.4 g/dL).

A 7-year-old girl presents to the pediatrician multiple times for hematuria until it is discovered that her mother is contaminating the urine samples with her own blood. Which of the following is the suspected diagnosis? A. Conversion disorder B. Factitious disorder C. Medical child abuse D. Somatic symptom disorder

C. Medical child abuse Conversion disorder (A) is a condition marked by neurologic symptoms that are not consistent with known neurologic or general medical conditions. Factitious disorder (B) occurs when patients intentionally fabricate symptoms, with the goal being to play the role of the patient. Medical child abuse is when the caregiver fabricates symptoms in a child. Somatic symptom disorder (D) is a condition marked by the presence of longstanding somatic symptoms, such as pain or fatigue, that cause significant and excessive worrying and time devotion. The symptoms may or may not be explained by a general medical condition. Medical child abuse, which was formerly known as Munchausen syndrome by proxy, is diagnosed when a child receives unnecessary medical care that is harmful or potentially harmful because of a caregiver's actions. The caregiver may exaggerate symptoms, lie about symptoms, fabricate exam findings, or intentionally produce illness in a child. The clinical manifestations of medical child abuse vary significantly and include almost any presentation. However, it is common for the symptoms to be unwitnessed. Symptoms may include vomiting, diarrhea, hematuria, seizures, apneic episodes, and allergic reactions. The presentation of medical child abuse can be divided into mild, moderate, or severe based on the consequences for the child. In mild cases of medical child abuse, the caregiver may seek unnecessary diagnostic tests or treatment, but significant disruption in the child's life does not occur. Moderate cases of medical child abuse are marked by significant disruption in the child's life without the child's life being threatened. Examples include the caregiver causing rashes by painting on the skin or insisting on a wheelchair for a child who does not have a medical need for one. Severe cases of medical child abuse are potentially life-threatening either from the induction of illness or the medical care received. One example is a child having multiple episodes of polymicrobial sepsis due to a parent injecting feces in a central venous catheter. Clinicians must have a low index of suspicion for medical child abuse to make the diagnosis. In cases in which medical child abuse is suspected, particularly moderate or severe abuse, the clinician should consult with a multidisciplinary child abuse team.

A 34-year-old woman presents to the clinic for a mammogram. She reports that she is worried she will develop breast cancer because one of her friends developed breast cancer at a young age. She does not report any breast pain or lumps but does report that she had a transient tingling sensation on her left breast that occurred once and lasted 5 seconds. She has had eight mammograms in the past year and does self-breast examinations twice daily. She reports that her concern about having breast cancer has led to her spending excessive money on health care usage and hours each day reading about preventing breast cancer on the internet. Which of the following is a core feature of the suspected diagnosis? A. Desire for an external reward B. Fabrication of symptoms C. Minimal somatic symptoms D. Neurologic symptoms without a known medical explanation

C. Minimal somatic symptoms Desire for an external reward (A) is an important feature of malingering. Fabrication of symptoms (B) is the intentional production of symptoms. It is an important feature of factitious disorder and malingering. Neurologic symptoms without a known medical explanation (D) is the core feature of functional neurologic symptom disorder, which is also called conversion disorder. What is the most common known psychiatric comorbidity of illness anxiety disorder? -Generalized Anxiety Disorder

A 25-year-old woman presents to the emergency department with altered mental status. Her husband tells you he thinks she is under the influence of drugs but is not sure what she has been taking. Urine drug screen is positive for opioids. Which of the following physical exam findings is likely to be seen? A. Increased bowel sounds B. Increased respiratory rate C. Miosis D. Mydriasis

C. Miosis Opioids are a class of drugs used for pain relief and are derived from the opium poppy plant. They can be divided into three groups: naturally occurring, semisynthetic, and synthetic. Six opium alkaloids occur naturally, including morphine and codeine. Semisynthetic opioids use compounds that are derived from natural sources as the starting materials. Heroin, hydrocodone, and oxycodone are all semisynthetic opioids. Synthetic opioids are made using a chemical process where large molecules are synthesized from a combination of smaller materials. Buprenorphine, fentanyl, methadone, and tramadol are synthetic opioids. Regardless of the type, all opioids have a high potential for misuse and dependence and cause significant morbidity and mortality. Individuals experiencing acute opioid intoxication present with slurred speech, sedation, and miosis, also referred to as constricted or pinpoint pupils. If the patient is injecting opioids, injection sites may be seen on physical exam and sometimes show signs of infection, such as cellulitis or abscess. Examination of the nasal septum should be done to rule out perforation that may occur with repeated nasal use of opioids, especially when mixed with cocaine. A heart murmur can indicate the presence of bacterial endocarditis. Treatment in the acute setting consists of naloxone, which should be administered for patients experiencing opioid overdose. First-line, long-term treatment of opioid use disorder consists of medication-assisted treatment with an opioid agonist or antagonist, such as methadone or buprenorphine, together with psychosocial interventions Opioids affect gastrointestinal motility and generally cause constipation, bloating, and sometimes abdominal pain. Physical exam findings related to these gastrointestinal effects include decreased bowel sounds, not increased bowel sounds (A). Opioids depress respiration, and this sign is part of the opioid overdose triad in addition to pinpoint pupils and unconsciousness. Increased respiratory rate (B) is more commonly seen with stimulant misuse. Mydriasis (D) is the term used to describe dilated pupils. It can be a sign of drug use and is commonly seen in individuals using stimulants, dissociative agents such as phencyclidine, or psychedelic drugs such as lysergic acid diethylamide or psilocybin mushrooms.

A 19-year-old girl presents to the clinic for an annual physical. She states she runs at least 45 miles per week to maintain her figure. The patient has no significant medical history and has been amenorrheic for 6 months. Physical examination reveals a thin patient with peripheral edema and fine, downy hair on her face and arms. Vital signs include body mass index of 16.52 kg/m2, HR of 52 bpm, RR of 17 breaths per minute, BP of 98/52 mm Hg, T of 97.2°F, and SpO2 of 97% on room air. Which of the following is the best diagnosis? A. Extreme anorexia nervosa B. Mild anorexia nervosa C. Moderate anorexia nervosa D. Severe anorexia nervosa

C. Moderate anorexia nervosa Extreme anorexia nervosa (A) is diagnosed as a body mass index < 15 kg/m2, while mild anorexia nervosa (B) is characterized by a body mass index > 17 kg/m2. Patients with severe anorexia nervosa (D) have a body mass index between 15 and 15.99 kg/m2. Anorexia nervosa is an eating disorder that commonly presents during the adolescent and early adult years of life and most frequently affects women. This condition is characterized by self-imposed starvation due to a distorted perception of body weight and shape. Core diagnostic features of this disorder include a persistent restriction in energy intake that results in low body weight, an intense fear of gaining weight, and a distorted view of body image. Patients may engage in binge eating and purging practices or practice restriction via fasting, excessive exercise, or extreme dieting. The severity of anorexia nervosa is classified by body mass index, with a value ≥ 17 kg/m2 indicating mild disease, values between 16 and 16.99 kg/m2 representing moderate disease, and a body mass index of 15-15.99 kg/m2 defined as severe disease. Extreme anorexia nervosa is diagnosed in patients with a body mass index < 15 kg/m2. Anorexia commonly manifests with signs of starvation and malnutrition, including emaciation, significant hypotension, bradycardia, hypothermia, lanugo, peripheral edema, and amenorrhea or delayed menarche in prepubescent girls. Patients who practice self-induced vomiting may also have salivary gland hypertrophy, dental enamel erosion, and calluses or scars on the back of the hand and fingers (Russell sign). Patients may report exertional fatigue, dizziness, weakness, cold intolerance, palpitations, constipation, abdominal pain and bloating, and swelling of the feet. Laboratory results outside the reference range that can be due to chronic starvation include leukopenia, hypokalemia, hypochloremia, low thyroid hormone levels, and elevated liver enzymes, cortisol, and serum bicarbonate. Additionally, women often have low estrogen concentrations. Patients with anorexia nervosa who demonstrate medical instability (e.g., severe bradycardia or hypotension, cardiac dysrhythmia, body mass index < 15 kg/m2) require inpatient treatment with assessment and appropriate management of medical complications. Standard treatment procedures for patients with anorexia who do not require medical stabilization include nutritional rehabilitation and psychotherapy. Pharmacotherapy using antipsychotics, specifically olanzapine, can be added to an established regimen to promote weight gain. Distorted thoughts of body image or food are generally not amenable to pharmacotherapeutic approaches. Patients with comorbid depression or anxiety may benefit from selective serotonin reuptake inhibitor therapy (e.g., sertraline, fluoxetine), but tricyclic antidepressants (e.g., amitriptyline, nortriptyline) should be avoided in patients with anorexia, and bupropion is contraindicated in patients with eating disorders.

A 27-year-old man presents to the emergency department in a coma after being found on the side of a street in a parked car. Vital signs are significant for a respiratory rate of 10 breaths per minute. You examine him and notice pinpoint pupils. A standard urine drug test is performed. Which of the following drugs is most likely to return a positive result for the drug class that is suspected to have caused this patient's presentation? A. Fentanyl B. Methadone C. Morphine D. Oxycodone

C. Morphine Fentanyl (A) and methadone (B) are synthetic opioids. Therefore, they typically do not cause a positive result for opioids on standard urine drug testing. These drugs require an additional specific test. Oxycodone (D) is a semisynthetic opioid and is typically not detected by standard urine drug tests.

A 23-year-old woman presents to the emergency department for evaluation of new-onset psychosis. She is accompanied by a close friend who has been worried about her. Her friend reports that, for several months, there has been a change in the patient's behavior. The patient has been paranoid that someone has been following her and that someone can see her through her cell phone. The friend found the patient's phone in the garbage disposal. The friend was contacted by the patient's job after she had not shown up to work in several days. Today, the patient walked into oncoming traffic in the middle of a busy intersection. The patient said, "They told me to," and reports hearing commanding, unfamiliar voices. The patient reports no drug use or a family history of schizophrenia, but she does report a family history of major depression. She constantly looks over her shoulder during the exam but otherwise has a flat affect and demonstrates apathy. What in her history is a poor prognostic factor for her suspected condition? A. Acute onset B. Female sex C. Negative symptoms D. Paranoid behavior

C. Negative symptoms Acute onset (A) is associated with a better outcome in patients with schizophrenia. Patients with a more insidious onset of symptoms typically have a poorer outcome. Female sex (B) is associated with a good prognosis, while male sex is associated with a poor prognosis. Paranoid behavior (D), which typically involves more positive symptoms, is associated with a good prognosis, while patients with predominantly negative symptoms often have a poorer prognosis.

A 25-year-old man presents to the emergency department with altered mental status. Vital signs are significant for a temperature of 102.5°F, heart rate of 135 bpm, and blood pressure of 195/122 mm Hg. You examine him and find a stuporous man with profuse diaphoresis, rigidity in his extremities through all ranges of movement. You discuss his presentation with his mother, who notes he recently started taking risperidone for an unknown psychiatric condition. CT scan of his head is negative, and lumbar puncture reveals cerebrospinal fluid of 5 white blood cells/µL. Which of the following diagnoses is suspected? A. Bacterial meningitis B. Malignant hyperthermia C. Neuroleptic malignant syndrome D. Serotonin syndrome

C. Neuroleptic malignant syndrome Bacterial meningitis (A) presents with fever, headache, neck stiffness, and altered mental status. The diagnosis is supported by a cerebrospinal fluid white blood cell count of more than 1,000 white blood cells/µL. The diagnosis is confirmed by bacterial growth on a culture of cerebrospinal fluid. Malignant hyperthermia (B) is a rare genetic disorder that presents with similar clinical features as neuroleptic malignant syndrome. It is distinguished by the clinical setting since malignant hyperthermia occurs with use of potent halogenated anesthetic agents and succinylcholine. Serotonin syndrome (D) occurs in individuals who are taking serotonergic agents, such as selective serotonin reuptake inhibitors. It also causes similar symptoms to neuroleptic malignant syndrome. However, nausea, vomiting, and diarrhea are more common in serotonin syndrome. Typically patients with serotonin syndrome have hyperreflexia while neuroleptic malignant syndrome rarely has hyperreflexia. Neuroleptic malignant syndrome is a life-threatening condition associated with the use of dopamine antagonist agents or withdrawal from dopamine agonists. Common dopamine antagonists are antipsychotic medications and the antiemetics metoclopramide, prochlorperazine, and promethazine. The hallmark clinical tetrad of neuroleptic malignant syndrome is mental status changes, rigidity, fever, and dysautonomia. Dysautonomia consists of tachycardia, labile or high blood pressure, tachypnea, and diaphoresis. Lead-pipe rigidity is a classic exam finding and is defined by rigidity that produces smooth and steady resistance to passive movement of the limbs. Neuroleptic malignant syndrome develops most often within 2 weeks of starting a new antidopaminergic medication, but it can also develop after one dose or in a patient who has been on a stable dosing regimen for years. Common laboratory findings include elevated creatine kinase and leukocytosis. The diagnosis is supported by the presence of at least two of the four findings in the tetrad in a patient who takes a medication known to be associated with neuroleptic malignant syndrome. Neuroimaging and lumbar puncture for cerebrospinal fluid analysis must be obtained to rule out central nervous system infection and structural brain disease. Patients with neuroleptic malignant syndrome should be treated in the intensive care unit. Stopping the causative agent is the most important intervention. Otherwise, supportive care is the mainstay of treatment. Supportive care includes maintaining a euvolemic state via intravenous fluids, treating hyperthermia with cooling blankets, lowering blood pressure when it is markedly elevated, preventing venous thromboembolism with anticoagulation, and treating agitation with benzodiazepines. Some medications (including dantrolene, bromocriptine, and amantadine) are possibly effective in moderate or severe cases.

A 30-year-old man presents to the clinic with recurrent episodes of abrupt fear that last for about an hour. He often experiences palpitations and paresthesias during these episodes. The patient states he has stopped going to public places because he does not know when the episodes will occur. Which of the following is a risk factor for the suspected diagnosis? A. Advanced age B. Male sex C. Neuroticism personality trait D. Obesity

C. Neuroticism personality trait The risk factors include neuroticism personality trait, childhood physical or sexual abuse, and family history of panic disorder. Panic disorder is diagnosed according to the criteria in the fifth edition of the Diagnostic and Statistical Manual of Mental Disorders (DSM-5). The diagnostic criteria include at least two panic attacks and one or more of the following: concern about future attacks, worry about the implication of the attack, and significant changes in behavior related to the attacks. It is important to exclude substance use and underlying medical conditions prior to diagnosing panic disorder. The first-line treatment for panic disorder is cognitive behavioral therapy, pharmacologic therapy with a selective serotonin reuptake inhibitor (sertraline or fluoxetine), or both. Patients who show a partial response to selective serotonin reuptake inhibitors can have treatment augmented with a benzodiazepine, gabapentin, pregabalin, or mirtazapine. Panic disorder most often begins in adolescence or young adults, not advanced age (A). Panic disorder is more common in the female sex, not male sex (B). Obesity (D) has not been shown to increase the risk of panic disorder.

A 26-year-old woman with no significant medical history presents to the clinic for irregular menstrual cycles. She reports her menstrual cycles have been absent for the past 5 months, during which time she reports no new medications or supplements but reports increased exercise daily. Vital signs indicate HR of 57 bpm, RR of 18 breaths per minute, BP of 99/56 mm Hg, T of 97.7°F, SpO2 of 99% on room air, and a body mass index of 17.1 kg/m2. A urine human chorionic gonadotropin test is negative. Which of the following is the best initial treatment for this patient? A. Cognitive behavioral therapy B. Hospital admission for advanced therapy C. Nutritional counseling and rehabilitation D. Olanzapine

C. Nutritional counseling and rehabilitation Cognitive behavioral therapy (A) or another form of psychotherapy is usually involved in the treatment plan for patients with anorexia nervosa, but it is not the most important initial treatment. Hospital admission for advanced therapy (B) is not warranted for the patient in the vignette, as she does not meet the criteria for invasive nutritional rehabilitation, has a body mass index > 15 kg/m2, and does not have vital signs suggestive of instability (e.g., HR < 40 bpm, BP < 80/60 mm Hg). Olanzapine (D) is not a first-line treatment strategy for patients with anorexia nervosa and is instead reserved for those who have undergone nutritional rehabilitation and psychotherapy and have not gained weight.

A 20-year-old woman presents to the office reluctantly with her roommates, who express concerns about recent changes in the patient's behavior. The patient has recently had a hard time leaving the apartment for fear she will leave her hair straightener plugged in. She has to check multiple times before leaving and is over an hour late to her first class every day. The patient is quiet and reluctant to speak. On physical examination, you notice small patches of hair missing from her scalp. What is the most likely diagnosis? A. Generalized anxiety disorder B. Major depressive disorder C. Obsessive-compulsive disorder D. Obsessive-compulsive personality disorder

C. Obsessive-compulsive disorder Generalized anxiety disorder (A) is a mental disorder characterized by persistent and excessive worrying that lasts at least 6 months. The worrying is hard to control and causes significant distress to the patient involving real-life concerns. Major depressive disorder (B) is characterized by five or more of the following for at least 2 weeks: depressed mood, anhedonia, changes in appetite or weight, changes in sleep, feelings of worthlessness or guilt, decreased ability to concentrate, and recurrent thoughts of death. At least one of the symptoms must be either depressed mood or anhedonia to make a diagnosis. Patients may have ruminative thoughts, but they are not related to obsessions or compulsions. Obsessive-compulsive personality disorder (D) does not involve obsessions but is characterized by a pattern of perfectionism and control leading to repetitive behaviors. Obsessive-compulsive disorder (OCD) is a mental health disorder in which the patient experiences recurring, distressing thoughts (obsessions) or repetitive mental or behavioral actions (compulsions). OCD initially presents in childhood or adolescence and persists throughout the patient's lifetime. Patients may experience both or only one symptom. Obsessions are distressing, disturbing, and unwanted by the patient. They may present as recurrent thoughts such as the fear of germ exposure or contamination, images of violent scenes, or urges such as the urge to harm someone. Patients develop compulsions as a way to suppress the obsession or prevent it from coming true. Compulsions can be a physical or mental action, such as repetitively checking, washing, or counting. The obsessions and compulsions a person suffers from are specific to that person, but there are several themes that they usually follow, including cleaning, symmetry (e.g., repeating, ordering, counting), forbidden thoughts (e.g., aggressive or sexual thoughts), and harm. To make a diagnosis according to the DSM-5 criteria, a patient may have obsessions, compulsions, or both. The obsessions or compulsions last > 1 hour per day or cause significant distress to the patient, are not due to other substance use or medication, and are not explained by another mental disorder.

A 26-year-old man presents to the psychiatry clinic for treatment of anxiety predominantly related to his landscaping business. He reports that he spends hours at work focused on orderliness, which sometimes comes at the expense of efficiency. He says he has difficulty delegating work to others because he believes this will compromise the quality of the work, which leads to him working long hours and experiencing increased stress. His wife, who accompanies him to the visit, reports that it is difficult for him to get rid of things, and he is rigid in the way he runs his business. Which of the following is the most likely diagnosis? A. Generalized anxiety disorder B. Obsessive-compulsive disorder C. Obsessive-compulsive personality disorder D. Paranoid personality disorde

C. Obsessive-compulsive personality disorder Generalized anxiety disorder (A) is an anxiety disorder marked by excessive and persistent worrying about many aspects of life. Patients with generalized anxiety disorder often experience poor sleep, fatigue, difficulty relaxing, headaches, and pain in the neck, shoulders, and back. The patient in this vignette has anxiety related to work, but the vignette did not describe persistent worrying. Obsessive-compulsive disorder (B) is marked by obsessions and compulsions that are time-consuming, distressing, and impairing. Obsessions are recurrent, intrusive, and undesired thoughts, and compulsions are behaviors that patients perform to relieve the obsessions. Patients with obsessive-compulsive personality disorder do not have distressing obsessions and compulsions. Paranoid personality disorder (D) is a cluster A personality disorder marked by a pervasive distrust and suspiciousness of others. Although the patient in the vignette struggles to trust others with work, the overall picture is more consistent with obsessive-compulsive personality disorder than paranoid personality disorder.

A 24-year-old woman presents to the clinic for evaluation of amenorrhea. Her body mass index is 16 kg/m2, pulse is 60 beats per minute, blood pressure is 90/60 mm Hg, and temperature is 97.7°F. She has fine lanugo covering her body, and a midsystolic click is auscultated at the cardiac apex. She reports calorie restriction for the past 3 years because she feels fat and is intensely fearful of gaining weight. Which of the following personality disorders is most common in patients with her presumed diagnosis? A. Histrionic personality disorder B. Narcissistic personality disorder C. Obsessive-compulsive personality disorder D. Paranoid personality disorder

C. Obsessive-compulsive personality disorder Histrionic personality disorder (A) is characterized by excessive emotionality, theatricality, easy suggestibility, and attention-seeking behaviors. Anorexia nervosa is not associated with histrionic personality disorder. Patients with histrionic personality disorder are overly obsessed with their outward appearance and use their physical appearance to draw attention to themselves, whereas patients with anorexia nervosa are obsessed with their physical appearance but do not seek to attract attention to themselves. Narcissistic personality disorder (B) is characterized by grandiosity, a need for admiration, and a lack of empathy. Psychological disorders associated with narcissistic personality disorder include depression, bipolar disorder, anxiety, and substance use disorder. Paranoid personality disorder (D) is characterized by a pervasive distrust of others and their motives and is not commonly associated with anorexia nervosa.

A 23-year-old woman reports to the clinic with pain, swelling, and instability in her right knee. She states her symptoms arose while playing soccer 2 days ago when she felt a "pop" while cutting and had subsequent swelling. She reports severe anxiety when discussing first-line diagnostic testing. Which of the following is most appropriate? A. Applied tension technique B. Diagnosis without imaging C. Oral diazepam D. Referral for cognitive behavioral therapy

C. Oral diazepam Applied tension technique (A) is used to reduce the risk of syncopal episode during blood draw in patients with blood-injection-injury phobia and does not help patients with MRI claustrophobia. Diagnosis without imaging (B) is not an appropriate treatment strategy, as the suspected condition in the vignette (anterior cruciate ligament tear) often requires surgical intervention following diagnostic imaging with MRI. Referral for cognitive behavioral therapy (D) is not generally warranted in patients with MRI claustrophobia unless they require repeat imaging or have medical contraindications to benzodiazepines and positional adjustment is not an option. Specific phobias surrounding medical, dental, or surgical procedures cause acute anxiety and distress secondary to excessive fear and worry that interferes with normal functioning. Magnetic resonance imaging (MRI) claustrophobia is one of the most common procedural related specific phobias and is associated with the fear of suffocating during an MRI scan. Regardless of the specific phobia, excessive fears related to clinical procedures require the establishment of a trusting clinician-patient relationship, education about the condition and why the procedure is recommended, and education about the procedure. Patients should be given as much control as possible during the procedure and should be made comfortable. Primary intervention for MRI claustrophobia involves adjusting the patient's position within the scanner. If this is not feasible or does not resolve anxiety, a benzodiazepine (e.g., diazepam, lorazepam, alprazolam) is the first-line medication used for MRI claustrophobia. Dental phobia, which involves the fear and avoidance of going to the dentist, can be treated with psychotherapeutic interventions, such as cognitive behavioral therapy, in vivo exposure therapy, cognitive restructuring, and hypnotherapy, among others. Medical treatment for dental phobia involves sedation using nitrous oxide, general anesthesia, or benzodiazepines. Patients who require a blood draw but have a specific blood-injection-injury phobia should first be treated with applied tension to counteract the vasovagal fainting response associated with this phobia. This technique involves repeatedly tensing muscles to increase blood pressure and prevent syncope. Patients who have a known history of fainting or who cannot appropriately perform the applied tension technique should be informed to lie down, look away from the site of blood draw, and sit up slowly following resolution of any dizziness or fainting.

An 18-year-old woman presents with concerns for anxiety. She reports that symptoms mostly occur in social situations and fears that her peers will notice her anxiety resulting in embarrassment. Which of the following is a clinical manifestation of the most likely diagnosis? A. Delusions B. Inattention C. Palpitations D. Poor social skills

C. Palpitations Delusions (A) are associated with psychotic disorders and can cause a fear of social situations because of perceived harm rather than embarrassment or scrutiny. Inattention (B) is a symptom of attention-deficit/hyperactivity disorder, which can be associated with social anxiety secondary to fears that inattention will cause social issues. Inattention is not a typical symptom of social anxiety disorder. Poor social skills (D) may be associated with mild forms of autism spectrum disorder and can cause some social anxiety from the fear of missing or misinterpreting social cues. This is not a typical symptom of social anxiety disorder.

Which of the following clinical therapeutics would be most sedating? A. Citalopram B. Fluoxetine C. Paroxetine D. Sertraline

C. Paroxetine Citalopram (A) should be avoided in those with known QT prolongation but is not as commonly associated with sedation as paroxetine. Fluoxetine (B) is more commonly associated with increased activation. Sertraline (D) is commonly associated with diarrhea in the first few months of administration. Selective serotonin reuptake inhibitors (SSRIs) are generally safe and effective medications for treating depressive and anxiety disorders. In addition, these medications have been used to treat obsessive-compulsive disorder, post-traumatic stress disorder, and somatoform disorders. These drugs work by decreasing the action of the presynaptic serotonin reuptake pump, thereby increasing the amount of available serotonin in the synapse. This action eventually causes increased amounts of neuroprotective proteins. SSRIs are absorbed in the gastrointestinal tract and metabolized by the liver. Common side effects include sexual dysfunction, drowsiness, weight gain, insomnia, and headache, although these may be worse when beginning the medication and may subside over time. Sedation is more commonly associated with paroxetine than with other SSRIs.

A 55-year-old man who takes medications for hypertension, hyperlipidemia, and major depressive disorder presents to the clinic to discuss sexual dysfunction. He also mentions he has been experiencing nausea recently as well as sedation. Which of the following medications is likely responsible for his symptoms? A. Bupropion B. Mirtazapine C. Paroxetine D. Venlafaxine

C. Paroxetine Selective serotonin reuptake inhibitors (SSRIs) are the most common class of medications used to treat major depressive disorder. Gastrointestinal (GI) upset and sleep disturbances are common side effects of these medications. Paroxetine is a potent SSRI, and its mechanism of action is to inhibit the uptake of serotonin by the central nervous system, thus increasing available serotonin levels. Paroxetine is known to cause sexual dysfunction, nausea, sedation, and sometimes weight gain. Sexual dysfunction, GI upset, and sleep disturbances can be common side effects among SSRI medications. Bupropion (A) is a norepinephrine and dopamine reuptake inhibitor used to treat major depressive disorder. Bupropion is known to cause gastrointestinal side effects such as nausea, but it is not known to cause sexual side effects. Mirtazapine (B) is a tetracyclic antidepressant medication that can cause sedation but does not cause sexual side effects. Mirtazapine also causes weight gain, hypercholesterolemia, and constipation. Venlafaxine (D) is a serotonin and norepinephrine reuptake inhibitor, and common side effects include gastrointestinal upset, weight loss, and sleep disturbance. Sexual dysfunction is not a common side effect of venlafaxine.

A 41-year-old man presents to the office with a 1-month history of trouble sleeping, loss of appetite, decreased energy, and difficulty concentrating at work. He states that he used to go for daily walks with his neighbor which helped to clear his mind, but he no longer has an interest in doing this anymore. Which of the following tools has the highest specificity and sensitivity for screening for the suspected diagnosis? A. 5-item Geriatric Depression Scale B. Patient Health Questionnaire-2 C. Patient Health Questionnaire-9 D. World Health Organization Well-Being Index

C. Patient Health Questionnaire-9 The 5-item Geriatric Depression Scale (A) is used only in older patients and is often used as an additional screening tool. The Patient Health Questionnaire-2 (B) is convenient due to its short length but is less specific and sensitive than the PHQ-9, which inquires about several more possible symptoms of depression. The World Health Organization Well-Being Index (D) has high sensitivity but is less specific than other screenings.

A 35-year-old woman presents due to depressed mood. She states she has felt depressed for 3 years, feels the need to sleep all the time, has poor concentration at work, and feels hopeless her symptoms will ever get better. She has tried cognitive behavioral therapy and several antidepressants at times without significant relief. Which of the following is the most likely diagnosis? A. Bipolar major depression B. Major depressive disorder C. Persistent depressive disorder D. Seasonal affective disorder

C. Persistent depressive disorder Persistent depressive disorder or dysthymia is diagnosed in patients with a depressed mood for at least 2 years accompanied by at least two of the following: low energy, poor self-esteem, decreased or increased appetite, insomnia or hypersomnia, hopelessness, or poor concentration. At times, patients may meet the criteria for a major depressive episode, and underlying dysthymia worsens the course of major depression. Patients typically do not recover as well from dysthymia as from major depression. Recommended treatment modalities include pharmacotherapy and cognitive behavioral therapy. Selective serotonin reuptake inhibitors are the recommended first-line treatment, but other antidepressant classes can be effective. For patients who respond to pharmacotherapy, it should be continued for 6-12 months after symptom improvement or resolution. Cognitive behavioral therapy is recommended for patients who have mild symptoms, do not respond to medication, or have incomplete resolution of symptoms with medication.

A 35-year-old man with a history of previously diagnosed schizophrenia presents to the office with worsening symptoms of his mental illness. He tells you last year he was sent to an inpatient drug treatment facility and has been sober since then. Which of the following antipsychotic medications should be avoided due to the potential for misuse and side effects of sedation? A. Clozapine B. Olanzapine C. Quetiapine D. Risperidone

C. Quetiapine Clozapine (A) is a second-generation antipsychotic that is closely regulated due to the risk of serious side effects, particularly agranulocytosis. It is reserved for patients with severe and refractory symptoms, requires regular laboratory monitoring, and should be prescribed by a psychiatrist who is familiar with its use. Olanzapine (B) has a higher risk of misuse potential than risperidone (D) or aripiprazole but has been found to be less of a risk than quetiapine. If there is a question about which agent to use in a patient with a history of substance use, risperidone or aripiprazole would be better choices than either quetiapine or olanzapine.

A 27-year-old man with a history of attention-deficit/hyperactivity disorder presents to primary care for an annual physical. During the initial encounter with a physician assistant, the patient displays pressured speech and begins recounting his accomplishments over the past year. He reports a recent promotion to creative director at a large advertising firm but adds that he is also thinking about investing in a marijuana dispensary or maybe becoming a neurosurgeon. Without impetus or explanation, the patient declares that he would "never get stuck" in a career as a PA. When questioned by a physician, the patient explains that his excitement is due to a "hot streak" at work and reports forgoing sleep for the past 10 days. Conversely, the patient notes being "uninspired" and "miserable" when last seen by the physician. While "uninspired," the patient experiences fatigue, sluggishness, difficulty concentrating, decreased appetite, and feelings of worthlessness. However, the patient and family report no current impairment, social dysfunction or need for hospitalization. Which of the following is recommended as an initial treatment for the patient's current condition? A. Lithium B. Methylphenidate C. Quetiapine D. Sertraline

C. Quetiapine Monotherapy with a second-generation antipsychotic such as quetiapine is recommended as a first-line treatment for the hypomanic episodes of bipolar II disorder. Other atypical antipsychotics (such as aripiprazole, olanzapine, risperidone, or ziprasidone) may be used in the acute setting, but only quetiapine, aripiprazole, and ziprasidone are approved as maintenance therapies. Further, quetiapine is also approved for use during major depressive episodes. Unlike Bipolar I Disorder, patients with bipolar II disorder must meet DSM-V for major depressive disorder, and do not experience manic or "mixed" episodes that markedly impair function or require hospitalization. "Mixed" episodes are mood disturbances that simultaneously exhibit both manic and depressive features; while bipolar II patients experience hypomanic and depressive episodes at separate times. The patient above is actively hypomanic. He has been experiencing elevated mood ("excitement") for over four days and displays (1) pressured speech, (2) grandiosity, and (3) decreased need for sleep, but does not report impairment, and reports a previous major depressive episode. Therefore, the patient meets the DSM-V criteria for Bipolar II Disorder (but not bipolar I disorder). Although no functional impairment is noted, bipolar II disorder is associated with an increased risk for suicide, and warrants pharmacotherapy. Quetiapine covers hypomania and major depressive episodes, and is an appropriate choice for bipolar II disorder. Lithium (A) has many adverse effects and is only recommended for use in bipolar I disorder. Further, mood stabilizers such as lithium and valproate are often used in combination with antipsychotics, and are not recommended as first-line monotherapy in bipolar disorders. The patient above has not experienced functional impairment and does not require a mood stabilizer. Methylphenidate (B) is a stimulate and recommended for use in ADHD. The patient above does not report inattention or difficulty completing tasks. Therefore, stimulants are not indicated. Sertraline (D) is a selective serotonin reuptake inhibitor (SSRI) and approved to treat major depressive episodes but not mania or hypomania. Debate exists on the propensity of SSRIs to induce the "rapid cycling" of mood disturbances. Until more evidence is available, SSRI monotherapy is not recommended for manic, hypomanic, or emotionally labile patients.

A 2-year-old girl is admitted to the hospital for recurrent seizures. Her mother reports that the seizures were occurring daily prior to admission, but the patient has not had any seizures in the 3 days since being admitted to the hospital. On exam, the patient appears healthy without any neurologic abnormalities. The patient has had an MRI of the brain and an electroencephalogram, both of which were unremarkable. The mother reports that the patient has an older sibling who also had seizures prior to dying of an unknown medical illness at 6 years of age. Which of the following is a common finding in the suspected diagnosis? A. Caregiver refusal of medical procedures B. Consistently seeing the same health care clinician C. Repeat visits for similar symptoms in the absence of objective abnormal findings D. Two involved parents in the child's life

C. Repeat visits for similar symptoms in the absence of objective abnormal findings Caregiver refusal of medical procedures (A) is incorrect because caregivers who inflict medical child abuse often insist on painful procedures or hospitalizations despite clinicians not thinking they are necessary. Patients experiencing medical child abuse frequently see multiple health care clinicians, not the same health care clinician (B). The perpetrator of medical child abuse is typically a woman, and the father is often absent from the child's life, thus two involved parents in the child's life (D) is incorrect.

A 25-year-old man presents to the emergency department in police custody for aggressive behavior. The police report he has a history of violent acts, including killing his neighbor's dog, and does not have remorse for his actions. You suspect a personality disorder. Which of the following is the recommended daily pharmacologic treatment for individuals with aggressive behavior due to this condition? A. Alprazolam B. Fluoxetine C. Risperidone D. Sertraline

C. Risperidone Alprazolam (A) is a benzodiazepine, which has the potential for misuse. All medications with the potential for misuse should be avoided in individuals with antisocial personality disorder. Fluoxetine (B) and sertraline (D) are selective serotonin reuptake inhibitors. They are considered second-line agents if second-generation antipsychotics are ineffective or not tolerated.

A 32-year-old woman presents to the psychiatry clinic to discuss treatment for a psychiatric condition. She reports she is unable to maintain a job because she has recurring thoughts she has left a door unlocked at her apartment. These thoughts lead to her returning to her apartment multiple times per day to make sure the apartment doors are locked. She prefers to attempt pharmacologic therapy rather than cognitive behavioral therapy. Which of the following is the most appropriate first-line agent? A. Duloxetine B. Risperidone C. Sertraline D. Venlafaxine

C. Sertraline Duloxetine (A), which is a serotonin and norepinephrine reuptake inhibitor, has not been studied in the treatment of obsessive-compulsive disorder. Serotonin and norepinephrine reuptake inhibitors do not have FDA approval to treat obsessive-compulsive disorder. Risperidone (B) is an atypical antipsychotic medication that can be used as an augmenting agent in patients with obsessive-compulsive disorder who have a partial response to selective serotonin reuptake inhibitors or clomipramine. Venlafaxine (D) is another serotonin and norepinephrine reuptake inhibitor. Therefore, it does not have FDA approval to treat obsessive-compulsive disorder. How long should patients with obsessive-compulsive disorder and a positive response to pharmacologic therapy remain on medication? -at least 1 year

A 27-year-old woman presents to the clinic with a depressed mood for the past 3 years. She reports that during the same period she has slept more than usual and had difficulty concentrating. The symptoms occur most days, and during the past 3 years, she has never gone more than 1 week without symptoms. Which of the following is a common side effect of the first-line pharmacologic class used to treat the suspected diagnosis? A. Constipation B. Hair loss C. Sexual dysfunction D. Weight loss

C. Sexual dysfunction Constipation (A) is incorrect. Diarrhea is a known side effect of sertraline but changes in bowel habits, such as diarrhea and constipation, are not common adverse effects of selective serotonin reuptake inhibitors. Hair loss (B) is incorrect. Weight loss (D) is incorrect. Weight gain occurs in approximately 12% of patients treated with selective serotonin reuptake inhibitors.

A 26-year-old veteran presents with recurrent thoughts about an event that occurred while he was in the military. He reports that he avoids crowds and has flashbacks of the event occurring, which are disturbing to him. Considering the most likely diagnosis, which of the following would be the most common clinical manifestation? A. Amnesia B. Depersonalization C. Sleep disturbance D. Somatic symptoms

C. Sleep disturbance Amnesia (A) may be present in post-traumatic stress disorder but is more common in the dissociative subtype of the disorder. Depersonalization (B) is the feeling that one is disconnected from their own body and is more common with the dissociative subtype of post-traumatic stress disorder. Somatic symptoms (D) can be present in patients with post-traumatic stress disorder but are not as common as sleep disturbance and are not a criterion for diagnosis.

A 34-year-old man presents to a therapist for couples therapy at the insistence of his wife. He reports the problem with their relationship is that she does not show him the respect he deserves as the most prestigious attorney in town. His wife reports he lacks empathy and has a constant need to be admired. Which of the following additional features supports a diagnosis of the suspected personality disorder? A. Difficulty making everyday decisions without excessive advice B. Preoccupation with details, rules, lists, and organization C. Superficial and exploitative relationships D. Taking pleasure in few activities

C. Superficial and exploitative relationships Difficulty making everyday decisions without excessive advice (A) is a core feature of dependent personality disorder. Preoccupation with details, rules, lists, and organization (B) is a core feature of obsessive-compulsive personality disorder. Taking pleasure in few activities (D) is a core feature of schizoid personality disorder.

A 30-year-old woman who is obese and has chronic hepatitis C presents to the clinic with episodes of deep depression followed by weeks of expansive mood, flight of ideas, risk-seeking behavior, and insomnia. Which of the following would need to be monitored frequently if the patient is started on the most appropriate therapy? A. Fasting plasma glucose B. Liver enzymes C. Thyroid function D. Urine beta-human chorionic gonadotropin

C. Thyroid function Fasting plasma glucose (A) does not need to be monitored as diabetes mellitus is not a complication related to chronic lithium ingestion. Liver enzymes (B) should be monitored in patients who are treated with valproate. Although valproate is a common first-line therapy for bipolar disease, it is not the most appropriate choice for the patient in the above vignette because she is of childbearing age. Valproate can cause major congenital malformations and is contraindicated in women of childbearing age unless the patient is refractory to other medications. Lithium carries a slight risk of congenital malformations when administered during the first trimester but is not contraindicated during pregnancy because the risk is slight and the benefits often outweigh the risk. For this reason, urine beta-human chorionic gonadotropin (D) does not necessarily need to be monitored on a regular basis, although patients should be informed of the risk.

A 21-year-old woman presents with concerns for fatigue and abdominal discomfort. She reports her symptoms began 3 months ago when she started dieting. She feels she has no control over the amount of food she eats at times, and in an effort to compensate for the amount of food she eats, she resorts to vomiting after meals or using laxatives. Which of the following may be found on physical exam when considering the most likely diagnosis? A. Hypertension B. Sinus bradycardia C. Tooth enamel erosion D. Unilateral parotitis

C. Tooth enamel erosion Hypertension (A) is not consistent with bulimia nervosa, as typically patients with this disorder are volume depleted and thus exhibit hypotension. Sinus bradycardia (B) is not as likely as sinus tachycardia for the same reason. Volume depletion causes an increased heart rate. Unilateral parotitis (D) is not associated with bulimia nervosa. However, hypertrophy of the parotid gland bilaterally is more common and is secondary to self-induced vomiting. Bulimia nervosa is characterized by episodes of binge eating followed by inappropriate compensatory behavior that occurs at least once per week for at least 3 months. This behavior is secondary to an unhealthy perception of body image. During the periods of binge eating, individuals often feel they have no control over the amount of food they are consuming. Caloric restriction is a type of compensatory behavior that typically propagates the cycle of binge eating and vomiting. Compensatory behavior is most often demonstrated by self-induced vomiting but can include the inappropriate use of laxatives or emetics. Medical symptoms of bulimia nervosa commonly include lethargy, irregular menses, abdominal pain, or constipation. Physical exam findings may include tachycardia, hypotension, xerosis, sialadenosis, and tooth enamel erosion. The tooth enamel erosion is caused by frequent vomiting and contact with gastric acid. Laboratory studies should include a complete metabolic panel, complete blood count with differential, and urinalysis.

A 22 year-old female presents to the emergency department with rapid heart rate. She appears quite thin and dehydrated. She denies that she is thin, stating "I am so fat that I can hardly stand myself! That is why I exercise every day." She runs twelve to fifteen miles a day, and on weekends also bicycles forty to fifty miles. Her LMP was six months ago. On exam, she is 5' 6" tall and weighs 98 pounds. Temp 98 degrees F, pulse 100, respirations 18, BP 98/60. EKG shows sinus tachycardia. Laboratory findings include Na 138 mEq/L, K 2.8 mEq/L, Cl 91 mEq/L, BUN 35 mg/dL, Creatinine 1.1 mg/dL. Which of the following is the next most appropriate treatment? A. Propylthiouracil (PTU) and individual psychotherapy B. Hormone replacement and cognitive therapy C. Weight restoration and family therapy D. IV hydration and antidepressant therapy

C. Weight restoration and family therapy Anorexia nervosa requires a comprehensive, multidisciplinary approach to treatment that integrates medical management, individual psychotherapy, and family therapy. Currently, the best results have been shown with weight restoration accompanied by family therapy for patients with adolescent-onset anorexia nervosa and individual therapy for patients with onset after 18 years of age. Inpatient treatment is often required.

A 48-year-old male with alcohol use disorder, whose last drink was 6 hours ago, presents to the clinic complaining of palpitations and intermittent abdominal pain. On physical examination, blood pressure is 170/110 mm Hg, pulse 124/min, respirations 22/min, and temperature 100.4 degrees F. The patient is agitated and excitable. Cardiovascular examination reveals tachycardia without murmurs, gallops, or rubs and is otherwise normal. Which of the following is the most appropriate intervention? A. start antihypertensive medication B. begin a cardiac work-up for angina C. admit to the hospital for alcohol withdrawal D. consult social services for appropriate disposition

C. admit to the hospital for alcohol withdrawal This patient is exhibiting the signs and symptoms of alcoholic withdrawal characterized by delirium, autonomic hyperactivity, perceptual distortions, and fluctuating levels of psychomotor activity. Seizures are a common occurrence prior to DTs, but the delirium may occur without preceding seizures. This is a medical emergency, and if untreated, it has a mortality rate of 20%.

Rates of alcohol use in the adolescent are reportedly higher in: A. families with rigorous parental monitoring B. Japanese families C. adoptive children whose biological parent is an alcoholic D. mood disorders in a grandparent

C. adoptive children whose biological parent is an alcoholic There is a three-to fourfold increase in risk for alcohol dependence in adopted children whose biological parents are alcohol dependents.

A 26 year-old female has a long history of sexual promiscuity and substance abuse. She frequently expresses anger when she feels abandoned. She also has difficulty in controlling her anger at times. Her past relationships have been intense and short-lived. She has attempted suicide twice in the past 18 months. This patient exhibits which of the following disorders? A. bipolar B. avoidant personality C. borderline personality D. dissociative

C. borderline personality Borderline personality disorder is characterized by instability of interpersonal relationships, marked impulsivity that is potentially self-damaging, inappropriate, intense anger or control of anger, recurrent suicidal attempts, gestures or threats, and identity disturbances. All of these are exhibited in this patient.

A 33 year-old hypertensive patient presents with increasing feelings of worthlessness and hopelessness over the past several months. Other symptoms include frequent crying episodes, loss of appetite, weight loss, insomnia with daytime sleepiness, and thoughts of suicide without any specific plans in place at this time. Which of the following would be the most appropriate first-line medication for this patient? A. amitriptyline (Elavil) B. bupropion (Wellbutrin) C. fluoxetine (Prozac) D. phenelzine (Nardil)

C. fluoxetine (Prozac) Selective serotonin reuptake inhibitors are the drugs of first choice for treating major depression.

A 19 year-old woman has been consuming up to six beers daily since she was 16. She is now pregnant with her first child, has had little prenatal care, and is due to deliver in four weeks. Of the following, which neonatal problems should you anticipate? A. kernicterus B. hydrocephalus C. low birth weight D. teeth discoloration

C. low birth weight Fetal alcohol syndrome is a common cause of low birth weight.

A 28 year-old male patient is being treated for depression and has been taking paroxetine (Paxil) for the past two and a half months with a marked improvement in symptoms. The patient reports problems with sexual functioning which he believes is related to the medication. Which of the following is an immediate concern with abrupt discontinuation of the medication? A. increased risk of suicide B. diminished sexual functioning C. risk of drug withdrawal symptoms D. worsened depressive symptoms

C. risk of drug withdrawal symptoms The greatest risk of abrupt withdrawal of short acting SSRIs is a withdrawal syndrome or withdrawal delirium which represents cholinergic rebound.

What is the name of the criteria that can be used to diagnose Wernicke encephalopathy in individuals with chronic alcohol use disorder?

Caine criteria.

For what medical indications has cannabis been approved?

Chronic pain (including cancer, neuropathy), glaucoma, epilepsy, spasticity (as in multiple sclerosis), nausea and vomiting (refractory to medication), and anorexia.

Which selective serotonin reuptake inhibitor increases the QTc interval the most?

Citalopram.

Which antipsychotic drug has a black box warning of neutropenia?

Clozapine

What is considered first-line treatment for patients with binge eating disorder?

Cognitive Behavioral Therapy

A mother brings her 6 year-old boy for evaluation of school behavior problems. She says the teacher told her that the boy does not pay attention in class, that he gets up and runs around the room when the rest of the children are listening to a story, and that he seems to be easily distracted by events outside or in the hall. He refuses to remain in his seat during class, and occasionally sits under his desk or crawls around under a table. The teacher told the mother this behavior is interfering with the child's ability to function in the classroom and to learn. The mother states that she has noticed some of these behaviors at home, including his inability to watch his favorite cartoon program all the way through. Which of the following is the most likely diagnosis? A. Antisocial disorder B. Dysthymic mood disorder C. Obsessive-compulsive disorder D. Attention deficit hyperactivity disorder

D. Attention deficit hyperactivity disorder Attention deficit hyperactivity disorder is characterized by inattention, including increased distractibility and difficulty sustaining attention; poor impulse control and decreased self-inhibitory capacity; and motor overactivity and motor restlessness, which are pervasive and interfere with the individual's ability to function under normal circumstances.

A 20-year-old woman is seeing a therapist for a condition marked by repetitive excessive food intake followed by self-induced vomiting. These behaviors have been happening several times per week for 6 months, and the patient reports an intense fear of gaining weight. Her body mass index is 29.2 kg/m2. Which of the following is the most likely diagnosis? A. Anorexia nervosa B. Binge eating disorder C. Borderline personality disorder D. Bulimia nervosa

D. Bulimia nervosa Anorexia nervosa (A) is an eating disorder marked by restricted caloric intake leading to an abnormally low body mass index, intense fear of gaining weight, and a distorted perception of body weight and shape. The body mass index is the main distinguishing factor between anorexia nervosa and bulimia nervosa, as patients with bulimia nervosa have a normal or high body mass index. Binge eating disorder (B) is an eating disorder marked by binge eating episodes. It is distinguished from bulimia nervosa by the lack of compensatory behaviors that are seen in bulimia nervosa. Borderline personality disorder (C) is a cluster B personality disorder that is marked by emotional impulsivity. The other core features of borderline personality disorder are unstable interpersonal relationships and unstable self-image, suicidal behavior, and inappropriate intense anger.

A 26-year-old woman presents for psychotherapy. You notice she seems to use physical appearance to draw attention to herself and considers relationships to be more intimate than they actually are. Which of the following additional findings is most consistent with the suspected personality disorder? A. Cognitive and perceptual disturbances B. Disregard for the rights of others C. Distrust and suspiciousness of others D. Excessive emotionality and attention-seeking behavior

D. Excessive emotionality and attention-seeking behavior Histrionic personality disorder is a cluster B personality disorder marked by excessive emotionality and attention-seeking behavior. It is more common in women. Histrionic personality disorder is diagnosed according to the criteria in the fifth edition of the Diagnostic and Statistical Manual of Mental Disorders. The criteria include a pervasive pattern of excessive emotionality and attention-seeking that is present in a variety of contexts by early adulthood. Furthermore, at least five of the following symptoms must be present: discomfort with not being the center of attention, seductive behavior, quickly shifting and superficial emotional expression, use of physical appearance to draw attention to oneself or evade unwanted responsibilities, impressionistic style of speech that lacks in detail, exaggerated expression of emotion, influenced easily by others, and tendency to consider relationships more intimate than they are. The preferred treatment is psychotherapy, specifically cognitive behavioral therapy or group therapy. Cognitive and perceptual disturbances (A) are features that mark schizotypal personality disorder. Disregard for the rights of others (B) is a feature that marks antisocial personality disorder. Distrust and suspiciousness of others (C) are features that mark paranoid personality disorder.

Which of the following is effective in the long-term treatment of panic attacks? A. Triazolam (Halcion) B. Propranolol (Inderal) C. Lorazepam (Ativan) D. Fluoxetine (Prozac)

D. Fluoxetine (Prozac) SSRI's, such as fluoxetine, are the initial drugs of choice for the long-term treatment of panic disorders.

A 25-year-old man presents to his therapist to discuss his difficulty maintaining romantic relationships. He reports that he is sexually aroused by his partners' toes, which has ended several of his past relationships. Which of the following is another common focus of sexual arousal in the suspected condition? A. Bracelets B. Ears C. Eyes D. Hair

D. Hair Fetishistic disorder is a paraphilic disorder that is diagnosed in patients with recurrent and intense sexual arousal from either nonliving objects or nongenital body parts. The nonliving object or nongenital body part that leads to the sexual arousal is called the fetish. Common fetishes include women's underpants, bras, shoes, feet, toes, and hair. Fetishistic disorder can manifest with fantasies, urges, or behaviors about the fetish. It is more common in men. The diagnostic criteria for fetishistic disorder from the fifth edition of the Diagnostic and Statistical Manual of Mental Disorders require a symptom duration of at least 6 months and the symptoms must cause significant distress. However, most patients who have a fetish do not have clinically significant distress and thus do not meet the diagnostic criteria. There is limited available data regarding the treatment of fetishistic disorder. Pharmacologic treatments that have been attempted include antidepressants and antiandrogen medications. Psychotherapies that may be used include cognitive behavioral therapy and individual psychodynamic therapy.

A 21-year-old woman with anorexia nervosa is admitted to an inpatient treatment facility for nutritional rehabilitation. She develops lower extremity edema and orthopnea after a week of treatment. Her creatine kinase is 5,000 U/L. Which of the following electrolyte abnormalities is the primary cause of tissue hypoxia? A. Hyperkalemia B. Hyperphosphatemia C. Hypokalemia D. Hypophosphatemia

D. Hypophosphatemia Hyperkalemia (A) and hyperphosphatemia (B) do not occur in refeeding syndrome. Hypokalemia (C) is a common laboratory finding in refeeding syndrome since the increase in insulin will move potassium into the cells. However, hypophosphatemia is the primary cause of complications in refeeding syndrome.

A 32-year-old woman presents with depressed mood, loss of interest in daily activities, and weight gain. She reports difficulty sleeping and concentrating and feels she would be "better off dead," although she reports no specific suicidal ideation or plan. She has been feeling this way for the past month since her divorce was finalized. She is given a prescription for amitriptyline. What is the mechanism of action of the class of this medication? A. Antagonism of presynaptic alpha-2 adrenoreceptors B. Antagonism of the 5-HT2A receptor C. Increase of vesicular stores of norepinephrine and serotonin D. Inhibition of reuptake of norepinephrine and serotonin

D. Inhibition of reuptake of norepinephrine and serotonin Antagonism of the 5-HT2A receptor (A) is the major antidepressant action of nefazodone and trazodone. Antagonism of presynaptic alpha-2 adrenoreceptors (B) is the mechanism of action of mirtazapine, a heterocyclic antidepressant. Monoamine oxidase inhibitors work by increasing brain amine levels through increasing vesicular stores of norepinephrine and serotonin (C) and allowing a larger release when neuronal activity discharges these neurotransmitter In what pregnancy risk factor category are tricyclic antidepressants? -Cat C

A 31-year-old woman presents to the emergency department after ingesting a full bottle of sertraline 100 mg tablets. Her pulse is 120 beats per minute, blood pressure is 160/100 mm Hg, and temperature is 104.1°F. On exam, she has ocular clonus, hyperreflexia, agitation, dilated pupils, and diaphoresis. Which of the following represents the best first-line therapy for this patient? A. Acetaminophen 1,000 mg PO every 8 hours B. Benztropine 2 mg IV once every 24 hours C. Cyproheptadine 12 mg PO once, then 6 mg PO every 6 hours D. Lorazepam 2 mg IV every 4 hour

D. Lorazepam 2 mg IV every 4 hour Acetaminophen 1,000 mg every 8 hours (A) should be avoided in patients with serotonin syndrome because they may already have abnormalities in liver function and because they may be acidotic. Benztropine 2 mg IV once every 24 hours (B) will make the symptoms of serotonin syndrome worse because benztropine is an anticholinergic agent. Anticholinergic agents have many of the same side effects as serotonin excess, such as hyperpyrexia, tachycardia, and dry mucous membranes. Cyproheptadine 12 mg PO once, then 6 mg PO every 6 hours (C) is second-line therapy for patients with serotonin syndrome who do not respond well enough to benzodiazepines. Cyproheptadine is a histamine-1 antagonist with serotonin antagonist properties.

A patient with advanced AIDS complicated by toxoplasmosis presents with altered mental status, recent onset of seizures, and focal neurologic deficits. Which of the following diagnostic studies is most helpful? A. Toxoplasma gondii antibody titers B. CT scan of the brain C. Lumbar puncture D. MRI of brain

D. MRI of brain An MRI showing multiple isodense or hypodense ring-enhancing mass lesions is the most useful test for such a patient.

A 5-year-old boy presents to the emergency department with his mother for evaluation of abdominal pain. A chart review reveals this patient has been to the emergency department several times for a variety of symptoms with no specific diagnoses given. Abdominal exam is unremarkable, and while the mother is out of the room, the patient seems much improved, though when the mother returns, she insists something is wrong with her child. Which of the following is the most likely diagnosis? A. Child neglect B. Conversion disorder C. Malingering D. Medical child abuse

D. Medical child abuse Child neglect (A) is defined as any action or inaction that would put a child at serious risk for physical or emotional harm or death. Neglect can be further divided into several subtypes, including physical, supervisory, emotional, nutritional, educational, medical, and dental. Child neglect is the most common form of child abuse. Child medical neglect is the failure to seek medical care for a child when necessary or the failure to follow prescribed treatment and intervention when given. Conversion disorder (B) is a somatic symptom disorder in which the patient experiences neurological symptoms as a manifestation of psychological conflict. Conversion disorder commonly occurs in the setting of other psychological disorders, such as depression or anxiety. The patient is unconsciously exhibiting these symptoms, and there is no desire for any outside benefit on the part of the patient. Malingering (C) is a disorder in which the patient purposefully produces or fabricates symptoms in themself to get an external benefit, such as time off work, money, or medications. Malingering is differentiated from factitious disorders by the goal of the patient to obtain external reward, while a patient with a factitious disorder does not have a specific external reward in mind. Factitious Disorder Imposed on Another Context: usually a mother falsifying illness in her child DSM-5 Dx applies to perpetratorChild's Dx would typically be medical child abuse Red flags include discrepant Hx, recurrent illness unresponsive to treatment as expected, multiple medical visits or interventions, and clinical improvement in absence of caregiver Diagnosis or intervention often requires videotaping, consultation with hospital ethics or legal

A 10-year-old boy presents to the clinic with his mother, who is concerned about his poor school performance. She reports he makes careless mistakes, has difficulty maintaining attention, and is easily distracted during class. Which of the following medications for treating the suspected condition has a black box warning for misuse and dependence? A. Atomoxetine B. Clonidine C. Guanfacine D. Methylphenidate

D. Methylphenidate Atomoxetine (A) is a nonstimulant norepinephrine reuptake inhibitor that is used to treat ADHD. It has a black box warning regarding an increased risk of suicidal ideation in children and adolescents. Atomoxetine is typically used in patients who are refractory to stimulants, prefer to avoid stimulants, or have a history of misuse of substances. Clonidine (B) and guanfacine (C) are alpha-2 adrenergic agonists that may be used as monotherapy or as adjuncts to stimulants in the treatment of ADHD. They are less effective than stimulants or atomoxetine. Clonidine and guanfacine are used in patients who have not had an adequate response to stimulants or atomoxetine. The adverse effects associated with clonidine and guanfacine include somnolence, sedation, and hypotension. The somnolence and sedation tend to improve with continued use.

A 10-year-old boy presents after being referred by his teacher. He is having difficulty concentrating and sitting still while in school. He often interrupts his teacher and has problems completing in-class assignments. His parents report he has the same problems at home, with frequent outbursts and impulsive behaviors. Which of the following appropriate therapeutics works by blocking the reuptake of norepinephrine and dopamine into presynaptic neurons? A. Atomoxetine B. Clonidine C. Guanfacine D. Methylphenidate

D. Methylphenidate Atomoxetine (A) is a selective norepinephrine reuptake inhibitor. Clonidine (B) and guanfacine (C) are alpha-2 adrenergic agonists. Attention-deficit/hyperactivity disorder (ADHD) is a common neuropsychiatric disorder characterized by inattention and impulsivity that causes significant impairment. The diagnosis is made when a patient has a persistent pattern of inattention and hyperactivity or impulsivity, symptoms are present in two or more settings, and symptoms interfere with daily activities and performance. First-line treatment is generally with amphetamines or methylphenidate. Methylphenidate has an onset of action of about 30 to 60 minutes after administration and peaks at 1 to 2 hours. The mechanism of action is through blocking the reuptake of norepinephrine and dopamine into presynaptic neurons. Adverse effects include insomnia, weight loss, loss of appetite, and nervousness. Fatigue and irritability can occur as a rebound effect when the medication begins to wear off.

A 24-year-old man is found unconscious on a sidewalk and presents to the emergency department via ambulance. Vital signs include an HR of 55 bpm, BP of 100/68 mm Hg, RR of 8 breaths/minute, T of 98.6°F, and oxygen saturation of 90% on room air. Physical examination reveals a comatose man with miotic pupils and no signs of external trauma. His point-of-care glucose is 75 mg/dL. After supplemental oxygen administration, which of the following is the recommended treatment? A. Activated charcoal B. Flumazenil C. Methadone D. Naloxone

D. Naloxone Activated charcoal (A) is a form of gastrointestinal decontamination that can bind to opioids. However, it is not recommended for treating opioid intoxication due to the risk of aspiration, particularly in patients with a decreased level of consciousness. Flumazenil (B) is an antidote that can be used in patients with benzodiazepine intoxication. Methadone (C) is a long-acting opioid agonist that can be used in the treatment of opioid use disorder. If used in the treatment of opioid use disorder, it is administered regularly and helps reduce the euphoric high or craving associated with opioid use. Why is hemodialysis not an effective treatment for opioid intoxication? -The volume of distribution of opioids is too wide

A 10-year-old boy presents to the clinic for evaluation after his teacher voiced concerns about his behavior in school. She reports the patient has difficulty remaining seated during class, talks excessively, interrupts others, and blurts out answers too quickly. His parents state that, since he was 4 years old, the patient gets angry when waiting for his turn and enjoys boisterous play. His family voices concerns about the first-line medication and would like to try an alternative agent. Which of the following side effects is most likely with this alternative medication? A. Blurred vision B. Growth suppression C. Motor tic development D. Nausea

D. Nausea Blurred vision (A), growth suppression (B), and motor tic development (C) are known adverse effects of stimulant medications. Alternative medications, such as atomoxetine and guanfacine, do not result in as these side effects as frequently as stimulants, but they have less efficacy than stimulant medications in the treatment of attention-deficit/hyperactivity disorder.

A 57-year-old man presents to the clinic requesting help with smoking cessation. He has smoked one pack of cigarettes per day for 20 years and has multiple previous failed attempts at quitting on his own. After discussing all smoking cessation options, including gum, patches, and oral medications, he chooses to try varenicline. Which of the following symptoms is a common side effect that you should counsel the patient on? A. Cardiac dysrhythmia B. Changes in smell C. Hyperlipidemia D. Nausea

D. Nausea Cardiac dysrhythmia (A) is an uncommon adverse effect of varenicline, with a reported occurrence of < 1% of users. While changes in smell (B) have been reported and are sometimes helpful in smoking cessation by causing an aversion to the smell of cigarettes, it is not a common reaction and also occurs in < 1% of users. Hyperlipidemia (C) is also not a common adverse effect but can be seen more commonly in the use of atypical antipsychotics. Varenicline is a smoking cessation medication that agonizes and blocks nicotinic acetylcholine receptors. Varenicline stimulates dopamine activity to a smaller degree than nicotine does, resulting in decreased craving and withdrawal symptoms. Common side effects include nausea, vomiting, headache, insomnia, appetite changes, abnormal dreams, and altered mood, including irritability, depression, suicidality, and psychosis. Because of the changes in mood, close monitoring is advised when giving this medication to patients, especially those with a prior psychiatric history. Varenicline is given in a starter pack in which the patient takes one pill twice daily until the pack is complete.

A 22-year-old woman with a history of anxiety disorder presents to the emergency department with chest pressure, shortness of breath, and palpitations. Her vital signs initially show tachycardia and tachypnea but normalize after a few minutes. A workup is initiated that shows no acute cardiopulmonary etiology. Based on the suspected diagnosis, which of the following symptoms would most likely be present? A. Cough B. Diarrhea C. Headache D. Nausea

D. Nausea The differential diagnosis for a patient with shortness of breath should include pulmonary etiologies such as asthma or infection. A cough (A) would indicate a pulmonary cause, rather than a panic attack. Diarrhea (B) is not a symptom of a panic attack although some patients may report nausea or general gastrointestinal upset. Headache (C) in conjunction with episodic diaphoresis and palpitations could indicate a pheochromocytoma.

A 55-year-old man presents to the office with his wife for his annual physical exam. His wife pulls you aside to express concern about how much her husband is drinking. When asked, the patient reports drinking "a couple of beers after work." Which of the following is a diagnostic criterion from the fifth edition of the Diagnostic and Statistical Manual of Mental Disorders used to confirm the diagnosis of alcohol use disorder? A. Drinking more than three drinks per day B. Drinking to alleviate anxiety C. Increased drinking for the past 2 months D. Need for increased amounts of alcohol

D. Need for increased amounts of alcohol At-risk drinking is defined as drinking more than four drinks per day for men and more than three drinks per day (A) for women. One drink is defined as 12 ounces of beer, 8 ounces of malt liquor, 5 ounces of wine, or 1.5 ounces of hard liquor. Individuals who are engaging in at-risk drinking behaviors are at high risk of developing alcohol use disorder. At-risk drinking often occurs as an attempted means to alleviate anxiety (B) or solve other psychological problems. The DSM-5 diagnostic criteria for alcohol use disorder requires the presence of symptoms for at least 12 months. Increased drinking for the past 2 months (C) may fall into the at-risk drinking category depending on quantity but does not yet meet the criteria for alcohol use disorder.

A 32-year-old man presents to the clinic at the request of his family. He reports that it is difficult for him to maintain a job or friendships because he cannot trust anyone. He is also recently separated from his wife who he continuously thought was unfaithful. What are the core characteristics of the cluster that the suspected personality disorder is in? A. Absentminded and passive B. Anxious and fearful C. Dramatic, emotional, and erratic D. Odd and eccentric

D. Odd and eccentric Absentminded and passive (A) does not describe the core features of any cluster of personality disorders. Anxious and fearful (B) are the core features that describe cluster C personality disorders, which includes avoidant, dependent, and obsessive-compulsive. Dramatic, emotional, and erratic (C) are the core features that describe cluster B personality disorders, which includes antisocial, borderline, narcissistic, and histrionic.

Which of the following drugs is first-line therapy for schizophrenia? A. Chlorpromazine (Thorazine) B. Clozapine (Clozaril) C. Haloperidol (Haldol) D. Olanzapine (Zyprexa)

D. Olanzapine (Zyprexa) Initial pharmacologic therapy of schizophrenia should begin with one of the newer, "atypical" antipsychotic drugs, such as olanzapine, risperidone, quetiapine, etc. because their side effect profile is significantly better than the older drugs, and they may be more effective for negative psychotic symptoms.

A 24-year-old woman presents to the emergency department with generalized weakness. Her partner who is with her notes that she purges to avoid weight gain and has an intense fear of gaining weight. On exam, you note that she is emaciated and calculate her body mass index to be 15.1 kg/m2. Which of the following is an indication for admission in women with the suspected eating disorder? A. Blood pressure of 95/65 mm Hg B. Body mass index of 15.9 kg/m2 C. Heart rate of 48 bpm D. Orthostatic increase in pulse of 28 bpm

D. Orthostatic increase in pulse of 28 bpm An orthostatic increase in pulse of at least 20 bpm or decrease in systolic blood pressure of at least 20 mm Hg is considered unstable and is an indication for admission. Refeeding syndrome should be suspected in patients with marked edema or a serum phosphorus of < 2 mg/dL after beginning nutritional rehabilitation. The goal of treatment of anorexia nervosa is to restore normal weight. Treatment consists of nutritional rehabilitation and psychotherapy. Nutritional rehabilitation focuses on controlled and gradual weight gain by slowly increasing caloric intake. Psychotherapy may consist of cognitive behavioral therapy or family therapy. Pharmacologic therapy is sometimes used as an adjunct to nutritional rehabilitation and psychotherapy. Olanzapine, a second-generation antipsychotic, is the recommended pharmacologic agent.

Which of the following is the primary risk factor for the development of postpartum unipolar major depression? A. Age ≤ 40 years B. Married marital status C. Nulliparity D. Past history of perinatal depression

D. Past history of perinatal depression Age is a risk factor for the development of postpartum depression, with women age > 40 years or < 25 years the most at-risk ages. Age ≤ 40 years (A) is not a risk factor. Women who are single are at higher risk of developing postpartum depression. Women with a married marital status (B) are not at an increased risk. Multiparity is another risk factor. Nulliparous (C) women are not at higher risk of developing postpartum depression. While age, marital status, and parity status are all risk factors, they are not as great a risk factor as a past history of perinatal or nonperinatal depression. The postpartum period is defined as the 12 months after the birth of a child. Depression that occurs either during pregnancy or postpartum may be referred to as either peripartum or postpartum depression. It affects one in seven women, and if left untreated has significant maternal and neonatal morbidity. It is recommended to screen all pregnant and postpartum women for depression. The Edinburgh Postnatal Depression scale is the most commonly used instrument. This is a 10-item, self-reported scale that can be completed in 5 minutes or less. There are a number of risk factors for the development of postpartum unipolar major depression, including a history of domestic violence, unintended pregnancy, fear of childbirth, poor perinatal physical health, history of premenstrual syndrome, perinatal sleep disturbances, adverse neonatal outcomes, difficulty with breastfeeding, and childcare stress. The primary and greatest risk factor is a past history of perinatal or nonperinatal depression. The pathogenesis of postpartum depression is unclear, although genetic factors and hormonal changes are thought to play a role. Symptoms are similar in both postpartum depression and major depressive disorder. There are a number of adverse consequences of postpartum depression, including impairment in daily functioning, difficulty with breastfeeding, poor nutrition and health of the infant, disruption in care of other children in the home, and a negative impact on the woman's relationship with her partner. While untreated postpartum depression may resolve without treatment, it may also develop into persistent and chronic depressive disorder, making intervention important as a means to prevent long-term consequences. Suicide and infanticide are rare occurrences, but assessment of harm to self and the infant should be part of the evaluation of postpartum depression. Treatment for mild to moderate cases is with psychotherapy alone. Moderate to severe postpartum depression should be treated with antidepressant medication, and selective serotonin reuptake inhibitors are first line.

A 23-year-old woman presents to the clinic to discuss an episode of intense fear following a stressful life event. She reports that the episode was not incited by a trigger reminding her of a previous life event. Which of the following additional findings supports the suspected cause of her symptoms? A. Absence of somatic symptoms B. Duration of 4 hours C. Gradual onset D. Peak intensity within minutes

D. Peak intensity within minutes Absence of somatic symptoms (A) is incorrect. Panic attacks are accompanied by somatic symptoms, such as sweating, palpitations, trembling or shaking, shortness of breath, feelings of choking, chest pain, nausea, dizziness, and paresthesias. Duration of 4 hours (B) is incorrect. Panic attacks typically last less than an hour. Gradual onset (C) is incorrect because panic attacks typically have an abrupt onset.

A 27-year-old woman presents to the emergency department reporting dysphoria, muscle aches, yawning, diaphoresis, restlessness, rhinorrhea, lacrimation, nausea, vomiting, and diarrhea. Her symptoms began about 12 hours ago. On physical exam, she is tachycardic and hypotensive, with decreased skin turgor, increased bowel sounds, and several scabs and scars noted over both antecubital fossae. Which of the following signs and symptoms is consistent with the patient's most likely diagnosis? A. Altered mental status B. Hyperpyrexia C. Miosis D. Piloerection

D. Piloerection Acute opioid withdrawal is a syndrome caused by abrupt cessation of short-acting opioids, such as heroin. Opioid withdrawal symptoms usually begin within 12 to 24 hours of the last dose of the opioid, peak at day 3, and taper off by day 7. Classic opioid withdrawal signs and symptoms include piloerection, diaphoresis, yawning, restlessness, dysphoria, myalgia, arthralgia, rhinorrhea, lacrimation, nausea, vomiting, diarrhea, palpitations, mydriasis, insomnia, tremor, and irritability. Tachycardia and hypotension can result if the patient becomes dehydrated due to vomiting and diarrhea. Many patients with opioid use disorder continue to use opioids, not for the euphoria, but to avoid the withdrawal symptoms. Treatment of acute opioid withdrawal that is not secondary to opioid antagonist administration is with opioid agonists, such as methadone or buprenorphine, which decrease withdrawal symptoms without causing euphoria. Other nonopioid agents that treat the symptoms of acute opioid withdrawal include promethazine, loperamide, diazepam, and clonidine. Patients who desire continued assistance to become opioid-free can enter methadone programs and receive methadone on a prolonged and gradually diminishing basis, with the eventual goal of total abstinence from opioids.

A 30-year-old woman presents to the clinic reporting fatigue, excessive sleepiness, decreased appetite, depressed mood, increased guilt, lack of interest in enjoyable activities, and thoughts of suicide. She states these symptoms started 1 week after giving birth to her first child and have persisted for the past 7 weeks. Which of the following represents the most likely diagnosis? A. Peripartum bipolar disorder, manic episode with psychotic features B. Peripartum major depressive disorder with psychotic features C. Postpartum blues D. Postpartum unipolar major depression

D. Postpartum unipolar major depression Postpartum unipolar major depression is characterized by symptoms of major depressive disorder that present themselves during the 12 months after childbirth. The patient must have five or more symptoms that qualify as major depressive symptoms for the diagnosis. These symptoms include 2 or more weeks of depressed mood, excessive guilt, thoughts of suicide or self-harm, alterations in sleep, increased or decreased appetite, anhedonia, psychomotor agitation, fatigue, and diminished concentration. Patients at risk for postpartum depression are those with a history of a depressive episode, those with increased life stressors, patients with little social support, those with gestational diabetes, or patients with multiple gestation. Treatment of postpartum depression includes cognitive behavioral therapy and selective serotonin reuptake inhibitors. Sertraline is considered first-line treatment in nursing mothers with postpartum depression because its excretion into the breastmilk is low. Peripartum bipolar disorder, manic episode with psychotic features (A) presents in the pre- or postpartum period with signs of mania, such as increased energy, expansive mood, irritability, talkativeness, distractibility, and decreased need for sleep. These patients must also exhibit psychotic features, such as auditory or visual hallucinations, delusions, or paranoia. Peripartum major depressive disorder with psychotic features (B) is a diagnosis that may be made when criteria are met for major depressive disorder in the pre- or postpartum period and psychotic features are also present. Postpartum blues (C) refers to depressed mood, irritability, sleep disturbance, fatigue, and decreased concentration that occurs naturally 2 to 3 days after delivery. Postpartum blues are self-limited and mild, disappearing by 2 weeks postpartum.

A 28-year-old male presents with a recent onset of upsetting dreams, anxiety, and disturbing flashbacks. The patient reports that he returned from active duty in Iraq 3 months ago and was adjusting to life back at home until about 7 weeks ago when he began having an intense fear of loud noises and seeing sudden flashbacks to bombing attacks he endured during his time in Iraq. He had to take a leave from his army instructor duties as he was not able to handle the familiar settings of practice shooting ranges and sudden loud explosions during battalion training. After refusing to leave his house, he was finally persuaded by his wife to visit your clinic. What is the most likely diagnosis? A. Acute stress disorder B. Performance anxiety C. Schizophrenia D. Posttraumatic stress disorder E. Panic attack

D. Posttraumatic stress disorder A patient complaining of flashbacks, nightmares, and avoidance of specific stimuli is likely suffering from posttraumatic stress disorder (PTSD). PTSD is often seen in war veterans, rape victims and survivors of traumatic events. PTSD symptoms do not have to manifest right after the event, but they must last for longer than 1 month to qualify as PTSD. Acute stress disorder has the same symptom profile as PTSD but lasts from 3 days to a month. The most common symptoms of PTSD are flashbacks, nightmares, avoidance of stimuli, anhedonia, insomnia, and suicidal thoughts. The main course of treatment is psychotherapy and the use of selective serotonin reuptake inhibitors. Once diagnosed, benzodiazepines and atypical antipsychotics should be avoided due to the overwhelming number of side effects. Caretakers should be wary of substance abuse and mood disorders.

A 32-year-old woman presents to the clinic due to anxiety and insomnia. She states she has tried antidepressants, buspirone, antihistamines, and muscle relaxers, and none have controlled her anxiety in the past. Physical exam reveals tachycardia, tremor, and diaphoresis. Pharmacy reviews show she has had one particular medicine filled at five different pharmacies and prescribed by three different clinicians in the past year. Which of the following is the mechanism of action of the medicine she is most likely seeking? A. Increased release of catecholamines through inhibition of monoamine oxidase B. Inhibition of dopamine receptors C. Inhibition of serotonin reuptake D. Potentiation of endogenous gamma-aminobutyric acid type A

D. Potentiation of endogenous gamma-aminobutyric acid type A Increased release of catecholamines through inhibition of monoamine oxidase (A) is the mechanism of action of amphetamine. A patient seeking sleep and relief from anxiety would not seek an amphetamine. Inhibition of dopamine receptors (B) is the mechanism of action of antipsychotic medications. Antipsychotic medications may produce somnolence and help with insomnia but do not produce physical dependence. Inhibition of serotonin reuptake (C) is the mechanism of action of selective serotonin reuptake inhibitors such as fluoxetine and paroxetine. These medications are antidepressants, and the patient in the above vignette states antidepressants have not helped her in the past. This class of medication may cause short-lived withdrawal symptoms but is not considered habit-forming and does not produce physical dependence

A 35-year-old woman presents to a primary care clinic with chronic fatigue. She has been to several primary care providers and specialists and had numerous diagnostic tests performed, which have not revealed any clear etiology. She reports that she spends hours each day researching the cause of her fatigue. Which of the following additional findings would support the diagnosis of the condition that replaced hypochondriasis in the fifth edition of the Diagnostic and Statistical Manual of Mental Disorders? A. Enjoyment of playing the role of the patient B. External reward motivating symptoms C. Fear of a serious underlying illness D. Presence of multiple vague symptoms

D. Presence of multiple vague symptoms Somatic symptom disorder is a condition defined by the presence of long-standing somatic symptoms that cause devotion of excessive thoughts, anxiety, time, or energy. Common somatic symptoms include pain, fatigue, nausea, headaches, dyspnea, and paresthesias. The symptoms are real and may or may not be explained by a medical condition. However, it is the excessive thoughts, anxiety, time, and energy that lead to the diagnosis of somatic symptom disorder. Patients frequently have multiple somatic symptoms, although only one is required for the diagnosis. The estimated prevalence in the general population is about 4%, but the prevalence is likely higher in health care settings. Risk factors include female sex and concurrent psychiatric disorders. The diagnosis of somatic symptom disorder is made according to the criteria in the fifth edition of the Diagnostic and Statistical Manual of Mental Disorders. The duration of symptoms must be at least 6 months, although the somatic symptom of concern may change during this time. Furthermore, the condition must cause distress or psychosocial impairment. The management of somatic symptom disorder consists of acknowledging that the patient's symptoms are real and explaining that a psychiatric diagnosis does not negate the difficulty of the symptoms. It is best if patients with somatic symptom disorder see the same clinician consistently and avoid excessive diagnostic testing. Psychiatry referral may be indicated in refractory cases. Enjoyment of playing the role of the patient (A) is seen in factitious disorder. Factitious disorder is a condition in which one intentionally produces symptoms but there is not a clear external reward, as is seen in malingering. The goal of producing the symptoms is simply to play the role of the patient. External reward motivating symptoms (B) is seen in malingering. Malingering is a condition in which someone intentionally produces symptoms with the goal of an external reward, which may include a financial incentive or missed work. Fear of a serious underlying illness (C) is a feature that marks illness anxiety disorder. Patients with illness anxiety disorder may have somatic symptoms, but the primary feature is fear of a serious underlying medical condition.

A 28-year-old man with a history of substance use presents to the emergency department due to headache, nausea, vomiting, and restlessness. He is agitated upon arrival, making it difficult to obtain the history. Physical exam findings include mydriasis, pallor, tremors, and diaphoresis. Vital signs reveal blood pressure of 175/98 mm Hg and pulse of 110 bpm. Urine drug screen is positive for cocaine. Which of the following medications should be avoided in the acute phase of treatment? A. Aspirin B. Bromocriptine C. Nitroglycerin D. Propranolol

D. Propranolol Aspirin (A) is used to treat chest pain that often accompanies cocaine intoxication. Cocaine can cause thrombus formation, making aspirin an important part of the initial intervention. Aortic dissection can also occur due to cocaine use, so if there is any clinical suspicion for aortic dissection, then aspirin should not be used. Aspirin is also used along with statin therapy to prevent secondary cardiovascular disease in the long-term treatment of cocaine use disorder. The euphoria associated with cocaine intoxication is related to the blockage of dopamine reuptake. The dopamine agonist bromocriptine (B) can be used to relieve cravings that occur during acute cocaine withdrawal. Nitroglycerin (C) may be given by either sublingual or intravenous route to individuals with ischemic chest pain and ST elevation or depression. It is safe and typically part of the initial intervention for cocaine toxicity and chest pain when there is a normal ECG or minimal changes in the ST segment.

A 30-year-old man with no comorbidities and who takes no medications presents to the clinic with tachycardia that occurs while speaking to large crowds. This has not been a problem in the past, but he has recently been promoted and must address many coworkers on a weekly basis. When he gives his presentations, he sweats profusely, trembles, and feels as if his heart is beating out of his chest. Which of the following is the most appropriate therapy for this man's clinical disorder? A. Amitriptyline B. Clonidine C. Mirtazapine D. Propranolol

D. Propranolol Patients with symptoms of social anxiety disorder that only manifest themselves when performing one particular skill, such as speaking in front of large crowds or giving presentations at work, can be diagnosed with the performance-only subset of social anxiety disorder. These patients may only require treatment on an as-needed basis. Such treatment can include benzodiazepines, such as clonazepam or lorazepam, or beta-blockers, such as propranolol. Amitriptyline (A) is a tricyclic antidepressant. This class of medications has not been well studied in the treatment of social anxiety disorder, but in small trials, it does not appear to be very effective. Clonidine (B) is an antihypertensive medication that crosses the blood-brain barrier and can have anti-anxiolytic properties. Its use in social anxiety disorder is not well defined. Mirtazapine (C) is an atypical antidepressant that has not shown consistent effectiveness in the treatment of social anxiety disorder.

A 26-year-old woman presents for psychotherapy. Her condition is marked by unstable emotions, behaviors, and interpersonal relationships. She has a poorly formed identity and intense fear of abandonment from her close relationships. Which of the following additional findings would most support the suspected personality disorder? A. Avoidance of occupational activities that may involve criticism B. Inappropriate sexually seductive or provocative behavior C. Preoccupation with unjustified doubts about the trustworthiness of others D. Recurrent suicidal behaviors

D. Recurrent suicidal behaviors Avoidance of occupational activities that may involve criticism (A) is a core feature of narcissistic personality disorder. Inappropriate sexually seductive or provocative behavior (B) is a core feature of histrionic personality disorder. Preoccupation with unjustified doubts about the trustworthiness of others (C) is a core feature of paranoid personality disorder. Borderline personality disorder is a cluster B personality disorder that is marked by the following core clinical features: unstable emotions, behaviors, and interpersonal relationships; fear of abandonment; poorly formed identity; impulsiveness; and suicidal behaviors. Patients with borderline personality often begin relationships with intense attachments but end relationships with the slightest conflict. Individuals with borderline personality disorder are at increased risk for suicidal threats, gestures, or self-mutilating behavior. Similar to other personality disorders, the symptoms must be pervasive and begin during adolescence or early adulthood. The diagnostic criteria are listed in the fifth edition of the Diagnostic and Statistical Manual of Mental Disorders and are based on the core clinical features. Borderline personality disorder is diagnosed more often in women than men, and there are likely both genetic and environmental factors that contribute to its etiology. The best treatments for borderline personality disorder are psychotherapy and symptom-focused pharmacologic therapy. Mood stabilizers (lithium, valproate, and carbamazepine) tend to have more pharmacologic benefits than antidepressants.

A 25-year-old man presents to the clinic due to breast tenderness and milk discharge from bilateral nipples for the past 3 weeks. Two months ago he presented to the clinic with auditory and visual hallucinations, persecutory thoughts, and paranoia and was prescribed a medication that he believes is causing his current symptoms. Which of the following medications is the most likely culprit? A. Aripiprazole B. Divalproex C. Quetiapine D. Risperidone

D. Risperidone Aripiprazole (A) is a newer antipsychotic medication that has no effect on serum prolactin levels. In general, the side effect profile of aripiprazole is very favorable, although it is slightly less efficacious than risperidone in the treatment of schizophrenia. Divalproex (B) is an anticonvulsant used in the treatment of seizure disorder and bipolar disorder. Divalproex is not indicated in the treatment of schizophrenia or psychotic symptoms. Quetiapine (C) is an antipsychotic medication that is less likely than risperidone to cause hyperprolactinemia. Sedation and hyperlipidemia are major side effects of quetiapine.

A man presents to the emergency department, brought by emergency medical services after he was brandishing a knife in the city park. Upon arrival, emergency personnel noted bizarre behavior with self-inflicted cuts on the man's arms. He told first responders the voices told him he is invincible, and he resisted their initial attempts at treating his wounds. In the emergency department, the patient reports ingesting angel dust. Which of the following is most likely found on physical examination? A. Argyll Robertson pupil B. Miosis C. Mydriasis D. Rotatory nystagmus

D. Rotatory nystagmus An Argyll Robertson pupil (A) accommodates when the patient is asked to focus on near and distant objects alternatively but does not react to light exposure. This condition occurs most commonly with neurosyphilis. Miosis (B) defines a small, contracted pupil and is common with opioid use disorder, while mydriasis (C), a dilated pupil, is associated with cocaine use, among others. Which modern anesthetic is derived from phencyclidine? -ketamine

A 32-year-old man presents to the therapist at his family's request. He reports that his family is frequently concerned because he does not desire social relationships. He reports that he works as a scientist alone in a laboratory. His hobbies include hunting and fishing alone. Which of the following personality disorders is the most likely diagnosis? A. Antisocial B. Avoidant C. Paranoid D. Schizoid

D. Schizoid Antisocial (A) personality disorder is a cluster B personality disorder characterized by exploitative, rule-breaking, impulsive, and deceitful behaviors. Patients with antisocial personality disorder lack empathy for others. Although antisocial personality disorder cannot be diagnosed until 18 years of age, most patients who are eventually diagnosed with antisocial personality disorder are diagnosed with conduct disorder during childhood. Avoidant (B) personality disorder is a cluster C personality disorder that is similar to schizoid personality disorder in that individuals with each condition tend to be socially isolated. However, individuals with avoidant personality disorder have a desire for social relationships but have social anxiety limiting the pursuit of those relationships. This contrasts with schizoid personality disorder, in which individuals do not desire social relationships. Paranoid (C) personality disorder is a cluster A personality disorder marked by a pervasive distrust and suspiciousness of others. Although these individuals may be socially isolated, the social isolation is driven by their lack of trust in others.

A 24-year-old man presents to the psychiatry clinic with his wife for follow-up. He was brought to the emergency department 3 months ago after the police found him blocking traffic on the highway and telling drivers that the apocalypse was coming. At that time, he says he could hear voices telling him to save the world. His wife reports that these behaviors began abruptly 3 months ago after his father died, and he was subsequently treated in a psychiatric hospital, where his symptoms improved. However, he has been withdrawn with a flat affect since then and has not been able to function well enough to find a job. The patient had a negative medical workup near the onset of symptoms, and it included a negative drug test result. He reports no depressed mood. Which of the following is the most likely diagnosis? A. Bipolar disorder type I B. Brief psychotic disorder C. Schizophrenia D. Schizophreniform disorder

D. Schizophreniform disorder Bipolar disorder type I (A) is diagnosed in patients who have had at least one manic episode. However, most patients with bipolar disorder type I have depressive episodes. This patient had psychotic features, such as auditory hallucinations and the delusion that an apocalypse was coming. Psychotic features may be seen during both manic episodes and in patients with schizophrenia who do not meet the criteria for a manic episode. Manic episodes are defined by a persistently elevated mood lasting at least a week and including at least three of the following findings: distractibility, impulsive behavior, grandiosity, flight of ideas, increased activity, insomnia, and rapid or pressured speech. These criteria are not explicitly met in the vignette, thus schizophreniform disorder is the most correct answer. Brief psychotic disorder (B) is a condition in which the symptoms last more than a day but less than a month. The patient must have at least one of the following symptoms: hallucinations, delusions, disorganized speech, and disorganized behavior. Negative symptoms are less likely to occur than in schizophrenia and schizophreniform disorder. Schizophrenia (C) is a psychiatric condition with similar clinical symptoms to schizophreniform disorder. However, the symptoms must persist for at least 6 months to meet the diagnostic criteria.

A 35-year-old man presents with a persistent pattern of odd behavior since early adulthood. He often believes that people talking on the radio have special messages for him. In addition, he has few close friends, pervasive suspicion of others, and social anxiety. Which of the following personality disorders does this individual most likely have? A. Avoidant B. Paranoid C. Schizoid D. Schizotypal

D. Schizotypal Schizotypal personality disorder is a cluster A personality disorder marked by cognitive-perceptual abnormalities, oddness or disorganized behavior, and interpersonal difficulties. The cognitive-perceptual abnormalities include odd beliefs, unusual perceptual experiences, ideas of reference, and paranoia. Ideas of reference are the belief that irrelevant occurrences in the world are related directly to you. Oddness and disorganized behavior manifest predominantly in ways that can be seen, such as hygiene, attire, and social behaviors that are eccentric. Individuals with schizotypal personality typically have few close relationships and chronic social anxiety that does not improve with familiarity. Similar to other personality disorders, the features of schizotypal personality disorder must be present by early adulthood. The diagnosis of schizotypal personality disorder is made according to the criteria in the fifth edition of the Diagnostic and Statistical Manual of Mental Disorders, which are based on the manifestations described above. The recommended treatment of schizotypal personality disorder is psychotherapy with pharmacotherapy targeting specific symptoms. Cognitive-perceptual and psychotic symptoms causing significant distress are typically treated with atypical antipsychotics, such as quetiapine. Patients with prominent cognitive deficits may also be treated with stimulant medications, such as methylphenidate. Guanfacine is an alternative for patients with a history of substance use. Social anxiety in patients with schizotypal personality disorder may be treated with a daily dose of clonazepam. Selective serotonin reuptake inhibitors can be prescribed for patients with a history of substance use disorder. Avoidant (A) personality disorder is marked by social inhibition and fear of rejection. These individuals avoid social interactions and seek jobs with limited interaction with others because of their social anxiety. Despite this anxiety, these individuals desire companionship. The odd behavior and cognitive-perceptual abnormalities distinguish schizotypal personality disorder from avoidant personality disorder. Paranoid (B) personality disorder is marked by pervasive distrust and suspicion of others (without evidence). The odd behavior and cognitive-perceptual disturbance distinguish paranoid personality disorder from avoidant personality disorder. Schizoid (C) personality disorder is also marked by social withdrawal. However, these individuals do not have a desire for companionship.

A 32-year-old woman presents to the emergency department with mental status changes, including agitation and confusion. Her vital signs reveal tachycardia and hyperthermia. On a physical exam, the patient is flushed and dilated pupils are noted. She has hyperreflexia and an inducible clonus. Her medical history includes gastroparesis and generalized anxiety disorder for which she takes metoclopramide and sertraline. Which of the following is the most likely diagnosis? A. Anticholinergic toxicity B. Malignant hyperthermia C. Neuroleptic malignant syndrome D. Serotonin syndrome

D. Serotonin syndrome Anticholinergic toxicity (A) is associated with the use of anticholinergic medications. The similarities between anticholinergic toxicity and serotonin syndrome include hyperthermia, agitation, dry mucous membranes, and mental status changes. Muscular tone and reflexes are normal in anticholinergic toxicity, which differentiates the two syndromes. Malignant hyperthermia (B) presents with muscle rigidity, tachycardia, and hyperthermia after exposure to halogenated anesthetics or depolarizing muscle relaxants. Neuroleptic malignant syndrome (C) develops more gradually than serotonin syndrome, which typically develops over 24 hours. This syndrome is also associated with slow neuromuscular activity rather than neuromuscular hyperactivity.

A 25-year-old woman who is 5 weeks postpartum and breastfeeding presents to the clinic with decreased interest in activities that are usually pleasurable for the past 3 weeks. She also reports excessive feelings of guilt, decreased concentration, dysphoria, and fatigue. You offer to refer the patient for psychotherapy, and she declines. She has no prior history of psychiatric conditions. Which of the following is the most appropriate pharmacologic treatment for the suspected diagnosis? A. Bupropion B. Doxepin C. Escitalopram D. Sertraline

D. Sertraline Bupropion (A) is typically avoided in postpartum lactating women because there have been few studies performed, and it is possibly associated with seizures in infants. Doxepin (B) is not recommended for lactating patients because it has a long half-life and may accumulate in nursing infants. Escitalopram (C) is less studied than sertraline and paroxetine. There was one case report where a term infant developed necrotizing enterocolitis after being exposed to escitalopram during the first 5 days of life and had no apparent risk factors for necrotizing enterocolitis.

A 19-year-old woman presents to the psychiatry clinic to discuss anxiety. She reports that she becomes anxious when she has to talk to people in her activities of daily living because she fears scrutiny and embarrassment. This has led to her performing poorly during her first year of college and has limited her ability to make friends. She does not report excessive anxiety in other situations. Which of the following is the most appropriate treatment? A. Buspirone B. Lorazepam C. Propranolol D. Sertraline

D. Sertraline Buspirone (A) is considered a partial serotonin agonist and is sometimes used to augment selective serotonin reuptake inhibitors in the treatment of generalized anxiety disorder. It is not a first-line treatment for social anxiety disorder. Lorazepam (B) is a benzodiazepine that can be used to treat social anxiety disorder in patients who experience symptoms only in rare or infrequent performance-type situations. However, this patient has more generalized symptoms. Propranolol (C) is a beta-blocker that can be used as needed to treat social anxiety disorder in patients who have physical symptoms (such as tremors, palpitations, or sweating) in rare or infrequent performance-type situations. It is not a first-line treatment for patients with frequent symptoms of social anxiety disorder in multiple situations.

A 17 year-old patient presents to the emergency department with agitation and hallucinations, and has one seizure. He admits to using "some drugs" but does not know what they were. On physical examination, temperature is 103 degrees F, BP 140/90, pulse 120, respirations 20. Remainder of the examination is unremarkable. Which of the following diagnostic studies will be of most help in managing this patient? A. Drug screen B. Urine dipstick C. Complete blood count D. Serum creatinine kinase

D. Serum creatinine kinase Serum creatinine kinase is the most sensitive test to detect rhabdomyolysis, a serious complication of seizures and hyperthermia related to drug abuse

A 5-year-old boy presents to the pediatric clinic with his parents who are concerned about episodes that occur during his sleep. They describe the episodes as the patient waking up with a loud scream, facial flushing, and sweating. The episodes usually occur during the first 3 hours of nocturnal sleep. The patient is difficult to console during these episodes and does not remember them the next day. Which of the following is the most likely diagnosis? A. Confusional arousals B. Nightmares C. Panic attacks D. Sleep terrors

D. Sleep terrors Confusional arousals (A) are another example of a nonrapid eye movement sleep parasomnia. These episodes usually occur in the first half of nocturnal sleep since rapid eye movement sleep predominates during the second half of the night. During confusional arousals, the classic findings include a child sitting up in bed in distress. The child may cry or moan and usually cannot be comforted. However, episodes of confusional arousal do not cause the flushing, sweating, or tachycardia seen during sleep terrors. Nightmares (B) are distinguished from sleep terrors because individuals wake up immediately after nightmares and can recall the content of the nightmare. In contrast, individuals who experience sleep terrors do not wake up and typically do not remember the episode the next day. Behaviors, such as screaming and kicking, occur more often in sleep terrors than nightmares. Sleep terrors are a nonrapid eye movement parasomnia, whereas nightmares are a rapid eye movement parasomnia. This difference leads to sleep terrors typically occurring during the first few hours of sleep, whereas nightmares occur in the last few hours of sleep, which are typically predominantly rapid eye movement sleep. Panic attacks (C) may occur during sleep. However, the classic symptoms include abruptly awakening from nonrapid eye movement sleep with a sensation of choking or tightness in the chest. Furthermore, many children with nocturnal panic attacks have episodes of panic attacks or anxiety during the day.

A 19-year-old woman presents with concerns about attending a work event. She reports she is afraid she will do or say something embarrassing, and she has been worrying about attending the event since its announcement. She reports that, for the last 2 years, she has had difficulty making friends and avoids any gatherings because she is worried people will notice her nervousness. Which of the following is the most likely diagnosis? A. Agoraphobia B. Panic disorder C. Performance anxiety disorder D. Social anxiety disorder

D. Social anxiety disorder Agoraphobia (A) is characterized by the avoidance of social situations because of the fear of developing anxiety and not being able to control oneself or leave the situation that is causing the anxiety. Panic disorder (B) involves intense panic attacks that have a sudden, unexpected onset and are distressing for the patient. Performance anxiety disorder (C) is characterized by anxiety centered around doing an activity in front of others and includes the fear of judgment. Social anxiety disorder is a common psychiatric disorder characterized by the intense fear of embarrassment or humiliation within social situations. Diagnosis is made by the following criteria: marked anxiety about social situations, fear of showing anxiety in social situations, avoidance behaviors, and significant social impairment. Symptoms must be present for longer than 6 months to confirm the diagnosis. The anxiety causes avoidance behaviors and distress within social situations, which causes social impairment and impacts overall quality of life. Physical manifestations of social anxiety disorder may distress the individual experiencing the anxiety and cause them to fear the manifestations will be noticed by others. These manifestations include blushing, sweating, or trembling. Coexisting anxiety or phobic conditions are common. Onset is typically in adolescence, and emergence after age 30 is uncommon. Assessment should include screening for coexisting conditions and phobias. Typically, selective serotonin reuptake inhibitors are used as first-line therapy for social anxiety disorder.

A 33-year-old woman presents to the clinic with chronic fatigue and headaches. You review the patient's prior medical records and see that she has been evaluated by seven different clinicians for these symptoms and has had extensive laboratory testing, a magnetic resonance imaging of her brain, and a lumbar puncture that did not reveal any abnormalities. The headaches have been diagnosed as tension-type headaches. She reports that she spends hours each day researching these symptoms, which has made it difficult to keep a job or socialize. Which of the following is the most likely diagnosis? A. Factitious disorder B. Generalized anxiety disorder C. Illness anxiety disorder D. Somatic symptom disorder

D. Somatic symptom disorder Factitious disorder (A) is a condition in which one intentionally produces somatic symptoms to achieve the role of being the patient. This condition is similar to malingering because both conditions are marked by falsification of symptoms. However, individuals with factitious disorder are not motivated by an external reward but rather the desire to be a patient. There is no indication in this vignette that the patient is intentionally producing the symptoms. Generalized anxiety disorder (B) is a condition marked by persistent anxiety across a variety of settings, such as work, home, and social activities. The patient in this vignette only shows signs of anxiety about her somatic symptoms. Illness anxiety disorder (C) is a condition in which an individual experiences distress over the idea of having or developing a serious medical condition. Individuals with illness anxiety disorder may experience somatic symptoms, but the somatic symptoms are mild in severity compared to somatic symptom disorder where the somatic symptoms are the core finding Somatic Symptom Disorder (Formerly Somatoform Disorder) Genuine symptoms with no identifiable cause Precipitating emotional event often precedes somatic symptoms F > M Management involves regularly scheduled primary care visits and a combination of behavior modification, psychological and social therapy

A 25 year-old female graduate student presents to the student health center for the eighth time in three weeks to be sure she does not have meningitis. She read that there was a student on campus who had meningitis last month, and now she has headaches and is requesting to be tested to make sure she does not have meningitis. She has been evaluated at each visit, and physical examination has been completely normal each time. Which of the following is the most likely diagnosis? A. Conversion disorder B. Hypochondriasis C. Malingering D. Somatic symptom disorder

D. Somatic symptom disorder Some previously distinct somatic disorders—somatization disorder, undifferentiated somatoform disorder, hypochondriasis, and somatoform pain disorder—are now considered somatic symptom disorders. All have common features. The symptoms may or may not be associated with another medical problem; symptoms no longer have to be medically unexplained. Sometimes the symptoms are normal body sensations or discomfort that do not signify a serious disorder. Patients are commonly unaware of their underlying mental problem and believe that they have physical ailments, so they typically continue to pressure physicians for additional or repeated tests and treatments even after results of a thorough evaluation have been negative.

A 19-year-old woman presents to the clinic complaining of decreased interest in usually enjoyable activities for the past 6 weeks. She also reports that she has gained 10 pounds and has been sleeping 16 hours per day during this time period. She feels fatigued despite sleeping more than usual and has difficulty concentrating when she attempts to be productive. You decide to start her on pharmacologic therapy. Which of the following is a boxed warning of the first-line pharmacologic class used to treat the suspected diagnosis? A. Agranulocytosis B. Diarrhea C. Sexual dysfunction D. Suicidal thoughts

D. Suicidal thoughts Agranulocytosis (A) refers to a blood disorder characterized by a decreased number of granulocytes (neutrophils, eosinophils, and basophils). It is a possible adverse effect of clozapine, which is a second-generation (atypical) antipsychotic. Diarrhea (B) is a possible adverse effect of selective serotonin reuptake inhibitors, particularly sertraline. However, there is not a box warning about diarrhea. Sexual dysfunction (C) is a common adverse effect of selective serotonin reuptake inhibitors, but there is not a box warning about sexual dysfunction. The symptoms of sexual dysfunction may include erectile dysfunction and decreased libido in men and anorgasmia and decreased libido in women.

A 26-year-old woman with a past medical history of irritable bowel syndrome and fibromyalgia presents to the neurology clinic for follow-up. She initially presented 2 weeks ago to the emergency department with sudden onset of right lower extremity paralysis. There was no history of trauma. Physical examination revealed grade 0/5 strength in the right lower extremity with 2+ deep tendon reflexes. She underwent further diagnostic testing, including neuroimaging of the brain and spine, a nerve conduction study, and an electromyography, which were each unremarkable. Which of the following features is most consistent with the suspected diagnosis? A. Symptoms are eventually explained by a medical condition B. Symptoms are motivated by an external reward C. Symptoms are motivated by an underlying desire to play the role of the patient D. Symptoms are unintentionally produced

D. Symptoms are unintentionally produced Symptoms are eventually explained by a medical condition (A) is incorrect. The diagnosis of conversion disorder is made once general medical conditions have been ruled out. Patients with new or worsening symptoms should have appropriate medical tests performed to rule out any new possible conditions. Symptoms are motivated by an external reward (B) describes malingering, which is a condition in which patients intentionally produce symptoms to gain an external reward, such as opioid pain medications or time off from work. Symptoms are motivated by an underlying desire to play the role of the patient (C) describes factitious disorder. The symptoms of conversion disorder have an unclear etiology but are not intentionally produced.

A 13-year-old girl presents to the emergency department after being found unconscious in her room. Her parents walked into her room and saw her with a cloth against her mouth and an empty can of paint thinner next to her bed. You detect a glue odor on exam. Which of the following substances is the most likely intoxicant? A. Cannabis B. Lysergic acid diethylamide C. Nitrous oxide D. Toluene

D. Toluene Cannabis (A) intoxication rarely leads to medical presentation. Individuals with cannabis intoxication experience a high marked by a euphoric and pleasurable feeling with increased sociability and decreased anxiety. Conjunctival injection, dry mouth, and increased appetite are also common. In some cases, however, individuals can have a reaction marked by dysphoria and irritability. Lysergic acid diethylamide (B) is often referred to as LSD. It is the classic hallucinogen and is typically taken by mouth. The common manifestations include an increased perception of sensory input, euphoria, a perception of being outside one's body, feelings of expansiveness, and synesthesia (blending of the senses). However, unwanted effects can include fear, dysphoria, and frightening imagery. Nitrous oxide (C) is a type of inhalant. It is not in paint thinner and is most frequently inhaled from balloons. Nitrous oxide misuse can lead to polyneuropathy, ataxia, and psychosis. Which urinary metabolites may be identified after toluene exposure? -Hippuric acid Inhalant Intoxication Recent intended or unintended short-term, high-dose exposure to inhalant substances Problematic behavior or psychological changes during or shortly afterwards At least 2 of 13 clinical signs or symptoms: Dizziness, nystagmus, incoordination, slurred speech, unsteady gait, lethargy, depressed reflexes, psychomotor retardation, tremor, generalized muscle weakness, blurred vision or diplopia, stupor or coma, euphoria Not attributable to another medical condition or better explained by another mental disorder or substance

A 15-year-old girl presents to the clinic with her mother for her yearly well check. During the standard conversation alone with the patient, she reports she has experienced some stress at home recently and has been consuming large amounts of food at night twice weekly for the past 6 months without any compensatory behaviors, such as purging. What is a complication you should watch for, given the most likely diagnosis? A. Bradycardia B. Colonic dysmotility C. Hypokalemia D. Type 2 diabetes

D. Type 2 diabetes Bradycardia (A), defined as a heart rate < 60 bpm, is a complication of anorexia nervosa. It is thought to be caused by increased parasympathetic tone without changed sympathetic tone. Colonic dysmotility (B) can be seen commonly in bulimia nervosa due to purging via misuse of laxatives or enemas. Hypokalemia (C) can also be seen in bulimia nervosa due to loss of potassium through self-induced vomiting.

A 34-year-old man presents to the psychiatry clinic for a 3-month follow-up visit. You suspect, based on his flat affect and apparent reacting to internal stimuli, that he has been noncompliant with his prescribed medication regimen. When asked about his medications, he verbalizes words repeatedly and does not answer the question. Which of the following most accurately describes this speech pattern? A. Circumstantial B. Loose association C. Tangential D. Verbigeration

D. Verbigeration In circumstantial speech (A), the patient discusses unrelated topics before eventually arriving at an answer. In Loose association (B), a patient will rapidly shift between topics that have no connection. In tangential speech (C), the patient discusses unrelated topics and never arrives at an answer.

A 45-year-old man presents to therapy for unusual sexual behavior. He reports being sexually aroused by using binoculars to watch women who live in the complex across the street from him undress. He has been doing this for 12 months. Which of the following paraphilic disorders does this patient most likely have? A. Exhibitionistic disorder B. Fetishistic disorder C. Frotteuristic disorder D. Voyeuristic disorder

D. Voyeuristic disorder Voyeuristic disorder is a paraphilic disorder characterized by sexual arousal from observing an unsuspecting nude or undressing person. Paraphilic disorders require engagement in unusual sexual activities or preoccupation with unusual sexual urges for at least 6 months. Paraphilic disorders are more common in men than women. Paraphilic disorders are difficult to treat, but cognitive behavioral therapy may be helpful.

Which of the following effects result from cigarette smoking and contributes to atherogenesis? A. transient decrease in blood pressure B. polycythemia with relative hyperoxemia C. decreased blood viscosity D. chronic inflammation

D. chronic inflammation

A patient presents within one hour of ingesting 30 tablets of diazepam (Valium). Which of the following is the most appropriate intervention? A. IV naloxone B. alkalinization of the urine C. cardiac pacing D. gastric lavage

D. gastric lavage Gastric lavage, along with the administration of activated charcoal and monitoring of vital signs and CNS status is the mainstay of therapy in the person who has overdosed on benzodiazepines. Vomiting should be induced in the person who is not comatose. Flumazenil (Romazicon), a specific benzodiazepine antagonist, might be used with caution in certain patients.

A divorced female patient presents for an employment physical. She states she has had a "run of bad luck" with jobs and has not been able to hold any job for longer than 2-3 months. She also states she has been arrested several times for getting into fights when she is out with the girls. She states she drinks an occasional beer, but denies any significant problems with alcohol. Which of the following laboratory findings would support your suspected diagnosis? A. decreased triglycerides B. decreased serum uric acid C. increased LDL cholesterol D. increased mean corpuscular volume

D. increased mean corpuscular volume This patient most likely has alcohol abuse as evidence by her social, occupational and legal issues. Laboratory tests will reveal the presence of an elevated mean corpuscular volume, triglycerides, serum uric acid and liver function tests.

A 56 year-old man is admitted to a hospital unit for evaluation of rectal bleeding and weight loss. He has a strong family history of cancer. Soon after admission, a barium enema is scheduled. The patient refuses the "prep" because he fears x-ray radiation. He states he has had previous x-rays, but becomes frightened at the thought of an x-ray and "can't face it." The most likely diagnosis is A. hysterical personality. B. dissociative state. C. conversion reaction. D. phobic neurosis

D. phobic neurosis Phobic neurosis is a phobic ideation of displacement where the patient transfers feelings of anxiety from the object to one that can be avoided.

What are the adverse effects of stimulant medications?

Decreased appetite, poor growth or weight loss, insomnia, social withdrawal, headache, increased heart rate, increased blood pressure, and exacerbation of tics disorders.

What is the most common comorbid mental health illness associated with delusional disorder?

Depression

What psychiatric comorbidities are most commonly associated with post-traumatic stress disorder?

Depressive disorders, anxiety disorders, and substance use disorders.

What are two examples of long-acting benzodiazepines?

Diazepam and clonazepam.

How long do hypomanic episodes typically last?

Episodes typically resolve within several weeks.

What is the pathophysiology behind amenorrhea and osteoporosis within the female athlete triad?

Estrogen deficiency.

What are the common adverse effects of first-generation antipsychotics?

Extrapyramidal side effects (akathisia, rigidity, bradykinesia, tremor, acute dystonia) and tardive dyskinesia.

True or false: anticoagulation is an absolute contraindication to electroconvulsive therapy.

FALSE

True or false: antidepressants are indicated in the treatment of a complicated grief reaction.

FALSE -CBT

True or false: bupropion is recommended for tobacco cessation in pregnant patients

False. Behavioral interventions should be the cornerstone of tobacco cessation efforts in pregnant women.

True or false: women tend to experience their first psychotic break at a younger age than men.

False. In men, it tends to occur between 18 and 25 years of age, while in women, it tends to occur between 21 and 30 years of age.

True or false: lithium is primarily excreted by the liver.

False. The kidneys primarily excrete lithium.

What selective serotonin reuptake inhibitor is associated more commonly with nausea and vomiting?

Fluvoxamine

What selective serotonin reuptake inhibitors are best to use in breastfeeding women?

Fluvoxamine, paroxetine, or sertraline.

Which paraphilic disorder refers to an individual receiving intense sexual arousal from rubbing against a nonconsenting individual?

Frotteuristic disorder.

Which condition presents with neurologic symptoms that cause clinically significant distress but do not have a known medical or neurologic cause after workup?

Functional neurologic symptom disorder (previously conversion disorder)

How are hallucinations different from delusions?

Hallucinations are sensory perceptions without physical stimuli, whereas delusions have real sensory stimuli.

What is crack lung?

Hemorrhagic alveolitis from inhaled cocaine.

What is the classic electrolyte abnormality associated with refeeding syndrome?

Hypophosphatemia.

What are the causes of false positives for opioids on a urine drug test?

Ingestion of poppy seeds, rifampin, and fluoroquinolones (ciprofloxacin, moxifloxacin, and levofloxacin).

Which nonintravenous benzodiazepine and route of administration combination provides the fastest onset of anxiolytic effects?

Intranasal midazolam.

What chronic problem may be noted in patients with bulimia nervosa who use ipecac syrup to purge?

Ipecac-induced myopathy, including cardiomyopathy that may be irreversible.

Which other psychiatric disorder, in addition to narcissistic personality disorder, can cause grandiosity?

Manic or hypomanic episodes of bipolar disorder.

What can be used for sleep in a patient who has insomnia and is being tapered off benzodiazepines?

Melatonin

What is the most common adverse effect of lithium?

Nephrogenic diabetes insipidus

What dietary deficiency, in addition to zinc and iron, has been shown to be a factor in attention-deficit/hyperactivity disorder?

Omega-3 Fatty Acids

What is the mechanism of action of buspirone?

Partial serotonin agonist

What is the difference between persistent depressive disorder and dysthymic disorder?

Persistent depressive disorder is a Diagnostic and Statistical Manual V term that is essentially identical to dysthymic disorder as described in the Diagnostic and Statistical Manual IV.

What is the first-line therapy for treatment of bipolar major depression?

Quetiapine or lurasidone

A 28-year-old woman presents to the clinic reporting ongoing intermittent abdominal pain, fatigue, and back pain for the past 7 months. The patient notes she has been evaluated in the emergency department multiple times and has seen other health care clinicians, none of whom have found a cause of her pain. She is constantly worried about her symptoms. On physical exam, the patient appears anxious. Otherwise, there are no significant findings. Labs and imaging results that she had completed recently are all within normal range. Which of the following is the most appropriate initial treatment for this patient? A. Antidepressant therapy B. Cognitive behavior therapy C. Dialectical behavior therapy D. Regularly scheduled visits in the office

Regularly scheduled visits in the office The patient in the above vignette has somatic symptom disorder, which is when an individual has one or more somatic symptoms that are not explained by a physical or medical condition. These symptoms cause significant distress or dysfunction on a day-to-day basis. It is important to suspect this disorder when a patient presents with vague symptoms and an inconsistent history. According to the fifth edition of the Diagnostic and Statistical Manual of Mental Disorders (DSM-5), the diagnostic criteria for somatic symptom disorder must include the following: 1) one or more somatic symptoms that cause the patient distress or psychosocial impairment; 2) excessive thoughts, feelings, or behaviors associated with the somatic symptoms; and 3) although the symptom may change, the disorder must be present for 6 or more months. Initial treatment includes regularly scheduled visits with a health care clinician.

Which atypical second-generation antipsychotics are most likely to cause hyperprolactinemia?

Risperidone and paliperidone.

What tool can be used to assess the side effects of medications used to treat attention-deficit/hyperactivity disorder?

Side effects rating scale.

Which defense mechanism is classically associated with borderline personality disorder?

Splitting, which is the tendency to view individuals in dichotomous categories, such as entirely good or evil.

Which pharmacologic agents used in the treatment of bipolar I disorder cause weight gain?

The antipsychotics, especially olanzapine, risperidone, and haloperidol.

What are ideas of reference?

The belief that irrelevant and innocuous things in the world have great personal significance.

What are neologisms?

The creation of new, idiosyncratic words.

Which laboratory tests should be run in a patient presenting with symptoms of major depressive disorder to rule out medical causes of depression?

Thyroid-stimulating hormone, complete metabolic panel, complete blood cell count, and urine toxicology

True or false: there is a strong link between suicidal ideation and obsessive-compulsive disorder.

True

Which second-generation antipsychotics cause the least amount of weight gain?

Ziprasidone, aripiprazole, and lurasidone.


Related study sets

Chapter 2: Causes of Abnormal Behavior

View Set

EMSU: Quiz 15, EMT Chapter 15 Respiratory Emergencies

View Set

APHUG Unit 4 - Chapter 8 Political Geography Key Issues #2

View Set

Cognition, Vision, Somatosensory Competency

View Set